HESI Fundamental practice

Lakukan tugas rumah & ujian kamu dengan baik sekarang menggunakan Quizwiz!

9. The nurse is administering the 0900 medications to a client who was admitted during the night. Which client statement indicates that the nurse should further assess the medication order?

"This is a new pill I have never taken before."

69. An older adult who recently began self-administration of insulin calls the nurse daily to review the steps that should be taken when giving an injection. The nurse has assessed the client's skills during two previous office visits and knows that the client safely administered the injections. What is the nurse's best response?

"When I watched you give yourself the injection, you did it correctly."

27. The nurse is performing an intake interview for a newly admitted client to the rehabilitation unit. Which questions will the nurse include in the interview? (Select all that apply.)

"When do you usually go to bed? And, when do you usually wake up?" "Do you usually bathe/shower in the morning or in the evening?" "Do you have any intolerance to food that we need to know about?" "How long do you think you will be here on the rehabilitation unit?"

**While instructing a community group regarding risk factors for coronary artery disease, the nurse provides a list of risk factors that cannot be modified. What should be included on the list? a. Heredity b. Hypertension c. Cigarette smoking d. Diabetes mellitus

( A) Heredity refers to genetic makeup and cannot be changed. Cigarette smoking is a lifestyle habit that involves behavior modification. Hypertension and diabetes mellitus are risk factors of coronary artery disease that can be controlled with diet, medication, and exercise.

**A client has been in a coma for two months and is maintained on bed rest. At what angle should the nurse adjust the head of the bed to prevent the effects of shearing force? a. 30 degrees b. 45 degrees c. 60 degrees d. 90 degrees

(A) Shearing force occurs when two surfaces move against each other; when the bed is at an angle greater than 30 degrees, the torso tends to slide and cause this phenomenon. Fortyfive, 60, and 90 degrees will raise the head of the bed too high, and the client will slide down in the bed, causing shearing forces.

**The nurse discovers several palpable elevated masses on a client's arms. Which term most accurately describes the assessment findings? a. Erosions b. Maculesct3 c. Papules d. Vesicles

( C) Papules are superficial and elevated up to 0.5 cm. Nodules and tumors are masses similar to papules but are elevated more than 0.5 cm and may infiltrate deeper into tissues. Erosions are characterized as loss of the epidermis layer; macules are nonpalpable, flat changes in skin color less than 1 cm in diameter; and vesicles are usually transparent, filled with serous fluid, and are a blisterlike elevation.

18. To avoid nerve injury, what location should the nurse select to administer a 3 mL IM injection? a. Ventrogluteal b. Outer upper quadrant of the buttock c. Two inches below the acromion process d. Vastus lateralis

(a)--2-3mL IM injections should always be made in the ventrogluteal area to minimize discomfort. this muscle is the deepest muscle in body

**A preschooler is admitted with a diagnosis of acute glomerulonephritis. The child's history reveals a 5-lb weight gain in 1 week and periorbital edema. How can the nurse obtain the most accurate information on the status of the child's edema? a. Weighing daily b. Observing body changes c. Measuring intake and output d. Measuring intake and output

(A) Weight monitoring is the most useful means of assessing fluid balance and changes in the edematous state; 1 L of fluid weighs about 2.2 lb. Visual inspection is subjective and generally inaccurate. Measuring intake and output is not as accurate as daily weights; fluid may be trapped in the third compartment. Monitoring of electrolyte values is unreliable; they may or may not be altered with fluid shifts.

**What is the best nursing intervention to minimize perineal edema after an episiotomy? a. Applying ice packs b. Offering warm sitz baths c. Administering aspirin prn d. Elevating the hips on a pillow

(A)Cold causes vasoconstriction and reduces edema by lessening the accumulation of blood and lymph at the episiotomy site; cold also deadens nerve endings and lessens the pain. Heat therapy alone does not resolve perineal edema. Aspirin is contraindicated in the early postpartum period because of the risk for hemorrhage. Elevating the hips provides little or minimal perineal relief.

**A home care nurse is teaching a client of the Orthodox Jewish faith about food supplements that can promote weight gain. Which client statement supports the nurse's conclusion that the teaching is successful? a. "Supper is the best time to add caffeine to my diet."correct2 b. "I should put a slice of cheese on my turkey sandwich." c. "Dinner should include both beef or veal and a glass of milk." d. "I will have cookies and a milkshake as a midafternoon snack."

(B) A supplement between meals helps a client gain weight. Drinking milk without consuming meat at the same time is an acceptable practice in the Orthodox Jewish faith. Caffeine is unrelated to Jewish dietary practices. Late in the day, caffeine should be avoided because it may keep the client awake at night. Orthodox Jews do not eat dairy and meat or fowl at the same meal.

**A nurse is caring for a client with a diagnosis of renal calculi secondary to hyperparathyroidism. Which type of diet should the nurse explore with the client when providing discharge information? a. Low purine b. Low calcium c. High phosphorus d. High alkaline ash

(B) Calcium and phosphorus are components of these stones; foods high in calcium and phosphorus should be avoided. Low purine and high alkaline ash diets are indicated for clients with gout. Foods high in phosphorus must be avoided.

**A hospitalized 7-year-old boy wakes up crying because he has wet his bed. It is most appropriate for the nurse to: a. Allow the child to change his bed and pajamas b. Change the child's bed while he changes his pajamas c. Take the child to the bathroom and change his pajamas d. Remind the child to call the nurse next time to avoid the need to change his pajamas

(B) Changing the child's bed while the child changes his pajamas will not call attention to the accident and will minimize the child's embarrassment. The child would probably be unable to change the bed without assistance; failure to complete the task might add to his embarrassment. Taking the child to the bathroom to change his pajamas and reminding the child to call a nurse next time will only add to the child's embarrassment.

**A nurse in the health clinic is counseling a college student who recently was diagnosed with asthma. On what aspect of care should the nurse focus? Incorrect1 a. Teaching how to make a room allergy-free. b. Referring to a support group for individuals with asthma. c. Arranging with the college to ensure a speedy return to classes. d. Evaluating whether the necessary lifestyle changes are understood.

(D ) Understanding the disorder and the details of care are essential for the client to be selfsufficient. Although teaching is important, a perceived understanding of the need for specific interventions must be expressed before there is a readiness for learning. Referring to a support group is premature; this may be done eventually. Although ensuring a speedy return to classes is important, involving the college should be the client's decision.

86. The nurse is preparing to administer 10 mL of liquid potassium chloride through a feeding tube, followed by 10 mL of liquid acetaminophen. Which action should the nurse include in this procedure?

Administer water between the doses of the two liquid medications.

An obese male client discusses with the nurse his plans to begin a long-term weight loss regimen. In addition to dietary changes, he plans to begin an intensive aerobic exercise program 3 to 4 times a week and to take stress management classes. After praising the client for his decision, which instruction is most important for the nurse to provide? A. Be sure to have a complete physical examination before beginning your planned exercise program. B. Be careful that the exercise program doesn't simply add to your stress level, making you want to eat more. C. Increased exercise helps to reduce stress, so you may not need to spend money on a stress management class. D. Make sure to monitor your weight loss regularly to provide a sense of accomplishment and motivation.

Answer: (A)Be sure to have a complete physical examination before beginning your planned exercise program.

When evaluating a client's plan of care, the nurse determines that a desired outcome was not achieved. Which action will the nurse implement first? A. Establish a new nursing diagnosis. B. Note which actions were not implemented. C. Add additional nursing orders to the plan. D. Collaborate with the healthcare provider to make changes.

Answer: (B)Note which actions were not implemented.

Which action is most important for the nurse to implement when donning sterile gloves? A. Maintain thumb at a ninety degree angle. B. Hold hands with fingers down while gloving. C. Keep gloved hands above the elbows. D. Put the glove on the dominant hand first.

Answer: (C)Keep gloved hands above the elbows

When conducting an admission assessment, the nurse should ask the client about the use of complimentary healing practices. Which statement is accurate regarding the use of these practices? A. Complimentary healing practices interfere with the efficacy of the medical model of treatment. B. Conventional medications are likely to interact with folk remedies and cause adverse effects. C. Many complimentary healing practices can be used in conjunction with conventional practices. D. Conventional medical practices will ultimately replace the use of complementary healing practices.

Answer: (C)Many complementary healing practices can be used in conjunction with conventional practices.

The nurse is evaluating client learning about a low-sodium diet. Selection of which meal would indicate to the nurse that this client understands the dietary restrictions? A. Tossed salad, low-sodium dressing, bacon and tomato sandwich. B. New England clam chowder, no-salt crackers, fresh fruit salad. C. Skim milk, turkey salad, roll, and vanilla ice cream. D. Macaroni and cheese, diet Coke, a slice of cherry pie.

Answer: (C)Skim milk, turkey salad, roll, and vanilla ice cream.

The healthcare provider prescribes furosemide (Lasix) 15mg IV stat. On hand is Lasix 20 mg/2ml. How many milliliters should the nurse administer.

Answer: 1.5ml

During a physical assessment, a female client begins to cry. Which action is best for the nurse to take?

Answer: Acknowledge the client's distress and tell her it is all right to cry.

A hospitalized male client is receiving nasogastric tube feeding via a small-bore tube and a continuous pump infusion. He reports that he had a bad bout of severe coughing a few minutes ago, but feels fine now. What action is best for the nurse to take?

Answer: After clearing the tube with 30ml of air, check the pH of fluid withdrawn from the tube.

Buddhism forbids use of

Answer: Alcohol in any form

A client with pneumonia has a decrease in O2 Sats from 94% to 88% while ambulating. Based on these findings, which intervention should the nurse implement first?

Answer: Assist the ambulating client back to the bed

Judaism prohibits

Answer: Autopsy

Which assessment data provides the most accurate determination of proper placement of a nasogastric tube?

Answer: Chest x-ray obtained after insertion

The nurse plans to obtain health assessment information from a primary source. Which option is a primary source for the completion of the health assessment?

Answer: Client

During the admission interview, which technique is most efficient for the nurse to use when obtaining info about signs and symptoms of a clients primary health problem?

Answer: Closed Ended Questions

A client with chronic kidney disease (CKD) selects a scrambled egg for his breakfast. What action should the nurse take?

Answer: Commend the client for selecting a high biological value protein.

The nurse notices that the mother of a 9 year old Vietnamese child always looks at the floor when she talks to the nurse. What action should the nurse take?

Answer: Continue asking the mother questions about the child.

A client who has been NPO for 3 days is receiving an infusion of D5W 0.45 normal saline (NS) with KCL 20mEq at 83ml/hr. The client's eight-hr urine output is 400ml, blood urea nitrogen (BUN) is 15mg/dl, lungs are clear bilaterally, serum glucose is 120mg/dl, and serum K is 3.7 mEq/L. Which action is most important for the nurse to implement?

Answer: Document in the medical record that these normal findings are expected Outcomes

The nurse is teaching a client proper use of an inhaler. When should the client administer the inhaler delivered medication to demonstrate correct use of the inhaler.

Answer: During the inhalation

A client with acute hemorrhagic anemia is to receive four units of packed RBCs as rapidly as possible. Which intervention is most important for the nurse to implement?

Answer: Ensure the accuracy of the blood type match.

The nurse is administering medications through a nasogastric tube (NGT) which is connected to suction. After ensuring correct tube placement, what action should the nurse take next?

Answer: Flush the tube with water.

The nurse is completing a mental assessment for a client who is demonstrating slow thought processes, personality changes, and emotional lability. Which area of the brain controls these neuro-cognitive functions?

Answer: Frontal lobe

A nurse is using a genogram while conducting a client's health assessment and past medical history. What information should the genogram provide?

Answer: Genetic and Familial Disorders

A client who is in hospice care complains of increasing amounts of pain. The healthcare provider prescribes an analgesic every four hours as needed. Which action should the nurse implement?

Answer: Give an around the clock schedule for administration of analgesics.

A client is in the radiology department at 0900 when the prescription Lovofloxacin 500mg IV q24hr is scheduled to be administered. The client returns to the unit at 1300. What is the best intervention for the nurse to implement?

Answer: Give the missed dose at 1300 and change the schedule to administer daily at 1300

The nurse notices that the Hispanic parents of a toddler who returns from surgery offer the child only the broth that comes on the clear liquid try. Other liquids, including gelatin, Popsicles, and juices , remain untouched. What explanation is most appropriate for this behavior?

Answer: Hot remedies restore the balance after surgery, which is considered a cold Condition

The nurse is instructing a client with high cholesterol about diet and lifestyle modification. what comment from the client indicated that the teaching has been effective

Answer: I will limit my intake of beef to 4 ounces per week

At the beginning of the shift the nurse assesses a client who is admitted from the PACU. When should the nurse document the client's findings?

Answer: Immediately after the assessments are completed.

The nurse is caring for a client who is receiving 24hr total parenteral nutrition (TPN) via a central line at 54ml/hr. When initially assessing the client, the nurse notes that the TPN solution has run out and the next TPN solution is not available. What immediate action should the nurse take?

Answer: Infuse 10% dextrose and water at 54ml/hr

What action should the nurse implement when accessing an implanted infusion port for a client who receives long term IV medications

Answer: Insert a Huber-point needle into the port.

Three days following surgery, a male client observes his colostomy for the first time. He becomes quite upset and tells the nurse that it is much bigger than he expected. What is the best response by the nurse?

Answer: Instruct the client that the stoma will become smaller when the initial swelling Diminishes

A male client tells the nurse that he does not know where he is or what year it is. What data should the nurse document that is most accurate?

Answer: Is disorientated to place and time

A nurse is preparing to give meds through an NG tube. Which nursing action should prevent complications during administration?

Answer: Mix each medication individually.

The UAPs working on a chronic neuro unit ask the nurse to help then determine the safest way to transfer an elderly client with a left sided weakness from the bed to the chair. What method describes the correct transfer procedure for this client?

Answer: Move the chair parallel to the right side of the bed, and stand the client on the right foot.

A young mother of three children complains of increased anxiety during her annual physical exam. What info should the nurse obtain first?

Answer: Nutritional History

The nurse is performing nasotracheal suctioning. After suctioning the clients trachea for fifteen seconds, large amounts of thick yellow secretions return. What action should the nurse implement next.

Answer: Re-oxygenate the client before attempting to suction again.

The nurse observes an UAP taking a clients blood pressure with a cuff that is too small, but the blood pressure reading obtained is within the client's usual range. What action is most important for the nurse to implement.

Answer: Reassess the clients BP using a larger cuff

A nurse assigns a UAP to obtain vital signs from a very anxious client. What instructions should the nurse give the UAP?

Answer: Report the results of the vital signs to the nurse

The nurse witnesses the signature of a client who has signed an informed consent. Which statement best explains this nursing responsibility?

Answer: The client voluntarily signed the form.

125. The nurse is teaching a client how to perform progressive muscle relaxation techniques to relieve insomnia. A week later the client reports, "I am still unable to sleep, despite following the same routine every night." Which action should the nurse take next?

Ask the client to describe the routine he is currently following.

38. A client is laughing at a television program when the evening nurse enters the room. The client states, "My foot is hurting. I would like a pain pill." How should the nurse respond?

Ask the client to rate the pain using a 1 to 10 scale.

120. During a routine assessment, an obese 50-year-old client states, "I feel so unlovable because of my weight." Which is the best response by the nurse?

Ask the client to talk about specific concerns.

21. Ten minutes after signing an operative permit for a fractured hip, an older client states, "The aliens will be coming to get me soon!" and falls asleep. Which action should the nurse take next?

Assess the client's neurologic status.

124. The client 12 hours after a laparotomy reports to the nurse a pain rating of 7 to 10. The nurse reviews the medication orders and it is another hour before the client can have another dose of pain medication. What actions can the nurse take to assist the client? (Select all that apply.)

Assist the client into side-lying, curled position. Obtain a warm pack to apply to the site of the incision. Suggest to the client taking 10 deep breaths, in through the nose and out through the mouth. Help the client with sustained concentration of a personally pleasant topic

44. For the client with a sodium level of 128 mEq/L, which meal selections should the nurse suggest to the client? (Select all that apply.)

Bacon, egg, and cheese biscuit Chinese chicken and vegetables, with rice and soy sauce Grilled hot dog on a bun with ketchup and mustard

14. The nurse determines that a postoperative client's respiratory rate has increased from 18 to 24 breaths/min. Based on this assessment finding, what is the priority nursing action?

Determine if pain is causing the client's tachypnea.

105. After receiving written and verbal instructions from a clinic nurse about a newly prescribed medication, a client asks the nurse what to do if questions arise about the medication after getting home. How should the nurse respond?

Encourage the client to call the clinic nurse or health care provider if any questions arise

67. While conducting an intake assessment of an adult client at a community mental health clinic, the nurse notes that the client's affect is flat, responds to questions with short answers, and reports problems with sleeping. At the end of the intake assessment, the client reveals the loss of a life partner 1 month ago. What is the nurse's best action for this client?

Encourage the client to see the clinic's grief counselor.

13. A nurse is assigned to care for a close friend in the hospital setting. Which action should the nurse take first when given the assignment?

Explain the relationship to the charge nurse and ask for reassignment.

85. The nurse is assisting a client to the bathroom. When the client is 5 feet from the bathroom door, he states, "I feel faint." Before the nurse can get the client to a chair, the client starts to fall. Which is the priority action for the nurse to take?

Gently lower the client to the floor.

Which action is most important for the nurse to implement when donning sterile gloves? A. Maintain thumb at a ninety degree angle. B. Hold hands with fingers down while gloving. C. Keep gloved hands above the elbows. D. Put the glove on the dominant hand first.

Gloved hands held below waist level are considered unsterile (C). (A and B) are not essential to maintaining asepsis. While it may be helpful to put the glove on the dominant hand first, it is not necessary to ensure asepsis (D). Correct Answer: C

52. The nurse is evaluating measures implemented for the non-responsive client. Which findings indicate the effectiveness of the care delivered? (Select all that apply.)

Heals without redness bilaterally Skin intact on the back Elbow joint fully flexes and extends. Ankle joint rotates 360 degrees freely.

29. The client states to the nurse, "This medication makes my mouth so dry." What are the nurse's suggestions to quench the client's thirst? (Select all that apply.)

Infuse your water with fresh citrus fruits to quench your thirst. Freeze strawberries and water together in popsicle mold. Keep a few pieces of hard candy with you to suck on.

87. The nurse is drawing a blood sample from the client's basilic vein. Multiple attempts were made prior to obtaining the sample with the tourniquet in place for nearly 5 minutes. Which laboratory finding would the nurse suspect is inaccurate related to the prolonged tourniquet placement?

K 5.3 mEq/L

61. By rolling contaminated gloves inside-out, the nurse is affecting which step in the chain of infection?

Mode of transmission

115. The nurse manager of a skilled nursing (chronic care) unit is instructing UAPs on ways to prevent complications of immobility. Which action should be included in this instruction?

Perform range-of-motion exercises to prevent contractures.

103. The nurse prepares to insert a nasogastric tube in a client with hyperemesis who is awake and alert. Which nursing actions are correct? (Select all that apply.)

Place the client in a high Fowler position. Instruct the client to swallow after the tube has passed the pharynx.

7. The nurse is working at a community-based clinic. Which client's spiritual well-being concerns the nurse the most?

Roman Catholic woman considering an abortion

95. When emptying 350 mL of pale yellow urine from a client's urinal, the nurse notes that this is the first time the client has voided in 4 hours. Which action should the nurse take next?

Record the amount on the client's fluid output record.

The nurse is teaching a client with numerous allergies how to avoid allergens. Which instruction should be included in this teaching plan? A. Avoid any types of sprays, powders, and perfumes. B. Wearing a mask while cleaning will not help to avoid allergens. C. Purchase any type of clothing, but be sure it is washed before wearing it. D. Pollen count is related to hay fever, not to allergens.

The client with allergies should be instructed to reduce any exposure to pollen, dust, fumes, odors, sprays, powders, and perfumes (A). The client should be encouraged to wear a mask when working around dust or pollen (B). Clients with allergies should avoid any clothing that causes itching; washing clothes will not prevent an allergic reaction to some fabrics (C). Pollen count is related to allergens (D), and the client should be instructed to stay indoors when the pollen count is high. Correct Answer: A

Which snack food is best for the nurse to provide a client with myasthenia gravis who is at risk for altered nutritional status? A. Chocolate pudding. B. Graham crackers. C. Sugar free gelatin. D. Apple slices.

The client with myasthenia gravis is at high risk for altered nutrition because of fatigue and muscle weakness resulting in dysphagia. Snacks that are semisolid, such as pudding (A) are easy to swallow and require minimal chewing effort, and provide calories and protein. (C) does not provide any nutritional value. (B and D) require energy to chew and are more difficult to swallow than pudding. Correct Answer: A

The nurse witnesses the signature of a client who has signed an informed consent. Which statement best explains this nursing responsibility? A. The client voluntarily signed the form. B. The client fully understands the procedure. C. The client agrees with the procedure to be done. D. The client authorizes continued treatment.

The nurse signs the consent form to witness that the client voluntarily signs the consent (A), that the client's signature is authentic, and that the client is otherwise competent to give consent. It is the healthcare provider's responsibility to ensure the client fully understands the procedure (B). The nurse's signature does not indicate (C or D). Correct Answer: A

An elderly client with a fractured left hip is on strict bedrest. Which nursing measure is essential to the client's nursing care? A. Massage any reddened areas for at least five minutes. B. Encourage active range of motion exercises on extremities. C. Position the client laterally, prone, and dorsally in sequence. D. Gently lift the client when moving into a desired position.

To avoid shearing forces when repositioning, the client should be lifted gently across a surface (D). Reddened areas should not be massaged (A) since this may increase the damage to already traumatized skin. To control pain and muscle spasms, active range of motion (B) may be limited on the affected leg. The position described in (C) is contraindicated for a client with a fractured left hip. Correct Answer: D

A male client tells the nurse that he does not know where he is or what year it is. What data should the nurse document that is most accurate? A. demonstrates loss of remote memory. B. exhibits expressive dysphasia. C. has a diminished attention span. D. is disoriented to place and time.

The client is exhibiting disorientation (D). (A) refers to memory of the distant past. The client is able to express himself without difficulty (B), and does not demonstrate a diminished attention span (C). Correct Answer: D

49. After a needle stick occurs while removing the cap from a sterile needle, which action should the nurse take next?

Select another sterile needle.

89. The clinic nurse is conducting an assessment of a 2-year-old. The nurse asks the mother, "What is your child playing with now?" Which response indicates to the nurse that further teaching is needed? (Select all that apply.)

"A marble run race track is set up in the playroom." "We got a golf set because my other children play golf."

17. A 65-year-old client who attends an adult daycare program and is wheelchair mobile has redness in the sacral area. Which instruction is most important for the nurse to provide?

"Change positions in the chair frequently"

72. The nurse is instructing a client in the proper use of a metered-dose inhaler. Which instruction should the nurse provide the client to ensure the optimal benefits from the drug?

"Compress the inhaler while slowly breathing in through your mouth."

30. A client with frequent urinary tract infections (UTIs) asks the nurse to explain a friend's advice about drinking a glass of juice daily to prevent future UTIs. Which response is best for the nurse to provide?

"Cranberry juice stops pathogens' adherence to the bladder."

73. The nurse is teaching an obese client, newly diagnosed with arteriosclerosis, about reducing the risk of a heart attack or stroke. Which health promotion brochure is most important for the nurse to provide to this client?

"Decreasing Cholesterol Levels Through Diet"

24. The nurse is at a teen event. Which teen's statement would cause the nurse to input some safety tips? (Select all that apply.)

"My boyfriend and I fool around on occasion, but he never comes when he is inside me." "I never use my seatbelt while I am driving. I hate the way it feels."

81. The nurse plans to administer diazepam, 4 mg IV push, to a client with severe anxiety. How many milliliters should the nurse administer?_(Round to the nearest tenth.)

0.8mL

8. The nurse is preparing to administer 0.32 mL of medication subcutaneously. What supplies will the nurse need to deliver the medication? (Select all that apply.)

A 1 mL syringe Alcohol prep pads A 24-gauge ¾″ needle

75. The nurse is orienting a new graduate to the reporting regulations often seen in the emergency department. Which clients will the nurse need to report to the nurse manager/supervisor to alert the proper authorities? (Select all that apply.)

A 7-year-old who states, "I get beat up by my parents all the time." The child has bruising on the back in various stages of healing. A 40-year-old who states, "I was in an argument with my sibling and the next thing I knew I was shot in the shoulder." A 30-year-old who states, "The brawl was worth the stab wound I got. My family has never liked that family. It is just that way."

**The nurse recognizes that a common conflict experienced by the older adult is the conflict between: a. Youth and old age b. Retirement and work c. Independence and dependence d. Wishing to die and wishing to live

A common conflict confronting the older adult is between the desire to be taken care of by others and the desire to be in charge of one's own destiny. The conflict between the young and old age may occur but is not common. The conflict between the retired and working may occur but is not common. The conflict between those wishing to die and those wishing to live may occur but is not common.

The nurse is assessing the nutritional status of several clients. Which client has the greatest nutritional need for additional intake of protein? A. A college-age track runner with a sprained ankle. B. A lactating woman nursing her 3-day-old infant. C. A school-aged child with Type 2 diabetes. D. An elderly man being treated for a peptic ulcer.

A lactating woman (B) has the greatest need for additional protein intake. (A, C, and D) are all conditions that require protein, but do not have the increased metabolic protein demands of lactation. Correct Answer: B

116. The nurse is preparing an IV solution containing 10 mEq of potassium in 100 mL of normal saline. Which findings would concern the nurse? (Select all that apply.)

A red and swollen peripheral IV site Starting the infusion without an infusion devise The solution is a lemon-yellow color

A postoperative client will need to perform daily dressing changes after discharge. Which outcome statement best demonstrates the client's readiness to manage his wound care after discharge? The client A. asks relevant questions regarding the dressing change. B. states he will be able to complete the wound care regimen. C. demonstrates the wound care procedure correctly. D. has all the necessary supplies for wound care.

A return demonstration of a procedure (C) provides an objective assessment of the client's ability to perform a task, while (A and B) are subjective measures. (D) is important, but is less of a priority prior to discharge than the nurse's assessment of the client's ability to complete the wound care. Correct Answer: C

Heparin 20,000 units in 500 ml D5W at 50 ml/hour has been infusing for 5½ hours. How much heparin has the client received? A. 11,000 units. B. 13,000 units. C. 15,000 units. D. 17,000 units.

Answer: (A)11000 units

1. A community hospital is opening a mental health services department. Which document should the nurse use to develop the unit's nursing guidelines?

ANA's Scope and Standards of Nursing Practice

30. The nurse retrieves hydromorphone 4mg/mL from the Pyxis MedStation, an automated dispensing system, for a client who is receiving hydromorphone 3 mg IM 6 hours PRN for severe pain. How many mL should the nurse administer to the client? (Enter the numerical value only. If rounding is required, round to the nearest tenth)

Ans: 0.8 4mg/1 mL = 3mg/ X mL : 3mg/4mg = 0.75 (READ THE ROUNDING CAREFULLY, here is says to the nearest tenth. Therefore, the answer is 0.8 mL).

45. The postoperative client is placed on a clear liquid diet. Which selections will the nurse select for the client? (Select all that apply.)

Apple juice Popsicles Gelatin Black coffee

94. When performing sterile wound care in the acute care setting, the nurse obtains a bottle of normal saline from the bedside table that is labeled "opened" and dated 48 hours prior to the current date. Which is the best action for the nurse to take?

Discard the saline solution and obtain a new unopened bottle.

59. The mental health nurse plans to discuss a client's depression with the health care provider in the emergency department. There are two clients sitting across from the emergency department desk. Which nursing action is best?

Discuss the client another time.

A client with chronic renal failure selects a scrambled egg for his breakfast. What action should the nurse take? A. Commend the client for selecting a high biologic value protein. B. Remind the client that protein in the diet should be avoided. C. Suggest that the client also select orange juice, to promote absorption. D. Encourage the client to attend classes on dietary management of CRF.

Foods such as eggs and milk (A) are high biologic proteins which are allowed because they are complete proteins and supply the essential amino acids that are necessary for growth and cell repair. Although a low-protein diet is followed (B), some protein is essential. Orange juice is rich in potassium, and should not be encouraged (C). The client has made a good diet choice, so (D) is not necessary. Correct Answer: A

82. In completing a client's preoperative routine, the nurse finds that the operative permit is not signed. The client begins to ask more questions about the surgical procedure. Which action should the nurse take next?

Inform the surgeon the client has questions about the surgery.

36. The health care provider diagnoses metastatic cancer and recommends a gastrostomy for an elderly client in stable condition. The client's adult child is concerned and states to the nurse, "I don't think my parent 'can handle' the cancer diagnosis." What information will guide the nurse's response?

If informed consent is withheld from a client, health care providers could be found guilty of negligence.

50. Urinary catheterization is prescribed for a postoperative female client who has been unable to void for 8 hours. The nurse inserts the catheter, but no urine is seen in the tubing. Which action will the nurse take next?

Leave the catheter in place and reattempt with another catheter.

58. The nurse is aware that malnutrition is a common problem among clients served by a community health clinic for the homeless. Which laboratory value is the most reliable indicator of chronic protein malnutrition?

Low serum albumin level

28. Which fluid will the nurse select to administer with the prescribed blood transfusion?

Normal saline

20. The nurse is preparing an older client for discharge. Which method is best for the nurse to use when evaluating the client's ability to perform a dressing change at home?

Observe the client change the dressing unassisted.

83. The nurse is providing care to clients at a day treatment center. One of the clients who is usually talkative and eats well is now confused and did not eat lunch. The nurse learns these are new findings as of today. What are the next nursing actions? (Select all that apply.)

Obtain a clean catch urine sample. Take the client's vital signs. Assess for the initiation of any new medications. Obtain an oxygen saturation.

**A nurse anticipates that a hospitalized client will be transferred to a nursing home. When should the nurse begin preparing the client for the transfer? a. At the time of admission b. After a relative gives permission c. When the client talks about future plans d. As soon as the client's transfer has been approved

Preparation of clients for discharge to their own home or to a nursing home should begin on the day of admission. The client gives permission for transfer to a nursing home. Intervention includes talking to the family members, including them in plans, and helping them understand the importance of early preparation. The client may never talk about future plans. Waiting until the client's transfer has been approved will make the adjustment more difficult than if the client had adequate preparation time

The nurse is caring for a client who is receiving 24-hour total parenteral nutrition (TPN) via a central line at 54 ml/hr. When initially assessing the client, the nurse notes that the TPN solution has run out and the next TPN solution is not available. What immediate action should the nurse take? A. Infuse normal saline at a keep vein open rate. B. Discontinue the IV and flush the port with heparin. C. Infuse 10 percent dextrose and water at 54 ml/hr. D. Obtain a stat blood glucose level and notify the healthcare provider.

TPN is discontinued gradually to allow the client to adjust to decreased levels of glucose. Administering 10% dextrose in water at the prescribed rate (C) will keep the client from experiencing hypoglycemia until the next TPN solution is available. The client could experience a hypoglycemic reaction if the current level of glucose (A) is not maintained or if the TPN is discontinued abruptly (B). There is no reason to obtain a stat blood glucose level (D) and the healthcare provider cannot do anything about this situation. Correct Answer: C

106. The clinic nurse is reviewing an antibiotic medication prescribed to a client with a urinary tract infection. What instructions will the nurse include in the client's teaching? (Select all that apply.)

Take all of the medication as prescribed, especially when you start feeling better. Take the medication with 8 ounce/240 mL of water. Keep this medication out of the reach of small children, preferably in a locked cabinet.

The nurse is administering medications through a nasogastric tube (NGT) which is connected to suction. After ensuring correct tube placement, what action should the nurse take next? A. Clamp the tube for 20 minutes. B. Flush the tube with water. C. Administer the medications as prescribed. D. Crush the tablets and dissolve in sterile water.

The NGT should be flushed before, after and in between each medication administered (B). Once all medications are administered, the NGT should be clamped for 20 minutes (A). (C and D) may be implemented only after the tubing has been flushed. Correct Answer: B

5. The nurse observes a UAP taking a client's blood pressure in the lower extremity. Which observation of this procedure requires the nurse to intervene with the UAP's approach?

The UAP auscultates the popliteal pulse with the cuff on the lower leg.

The nurse prepares a 1,000 ml IV of 5% dextrose and water to be infused over 8 hours. The infusion set delivers 10 drops per milliliter. The nurse should regulate the IV to administer approximately how many drops per minute? A. 80 B. 8 C. 21 D. 25

The accepted formula for figuring drops per minute is: amount to be infused in one hour × drop factor/time for infusion (min)= drops per minute. Using this formula: 1,000/8 hours = 125 ml/ hour 125 × 10 (drip factor) = 1,250 drops in one hour. 1,250/ 60 (number of minutes in one hour) = 20.8 or 21 gtt/min (C). Correct Answer: C

37. The nurse evaluates the insertion site of an IV catheter and suspects the IV is infiltrated. Which findings support the evaluation? (Select all that apply.)

The area around the insertion site is swollen. The insertion site is cool to the touch. The client complains of a burning pain at the site. Redness is noted in the area of the insertion site.

The nurse observes that a male client has removed the covering from an ice pack applied to his knee. What action should the nurse take first? A. Observe the appearance of the skin under the ice pack. B. Instruct the client regarding the need for the covering. C. Reapply the covering after filling with fresh ice. D. Ask the client how long the ice was applied to the skin.

The first action taken by the nurse should be to assess the skin for any possible thermal injury (A). If no injury to the skin has occurred, the nurse can take the other actions (B, C, and D) as needed. Correct Answer: A

A client who is in hospice care complains of increasing amounts of pain. The healthcare provider prescribes an analgesic every four hours as needed. Which action should the nurse implement? A. Give an around-the-clock schedule for administration of analgesics. B. Administer analgesic medication as needed when the pain is severe. C. Provide medication to keep the client sedated and unaware of stimuli. D. Offer a medication-free period so that the client can do daily activities.

The most effective management of pain is achieved using an around-the-clock schedule that provides analgesic medications on a regular basis (A) and in a timely manner. Analgesics are less effective if pain persists until it is severe, so an analgesic medication should be administered before the client's pain peaks (B). Providing comfort is a priority for the client who is dying, but sedation that impairs the client's ability to interact and experience the time before life ends should be minimized (C). Offering a medication-free period allows the serum drug level to fall, which is not an effective method to manage chronic pain (D). Correct Answer: A

The nurse observes an unlicensed assistive personnel (UAP) taking a client's blood pressure with a cuff that is too small, but the blood pressure reading obtained is within the client's usual range. What action is most important for the nurse to implement? A. Tell the UAP to use a larger cuff at the next scheduled assessment. B. Reassess the client's blood pressure using a larger cuff. C. Have the unit educator review this procedure with the UAPs. D. Teach the UAP the correct technique for assessing blood pressure.

The most important action is to ensure that an accurate BP reading is obtained. The nurse should reassess the BP with the correct size cuff (B). Reassessment should not be postponed (A). Though (C and D) are likely indicated, these actions do not have the priority of (B). Correct Answer: B

An obese male client discusses with the nurse his plans to begin a long-term weight loss regimen. In addition to dietary changes, he plans to begin an intensive aerobic exercise program 3 to 4 times a week and to take stress management classes. After praising the client for his decision, which instruction is most important for the nurse to provide? A. Be sure to have a complete physical examination before beginning your planned exercise program. B. Be careful that the exercise program doesn't simply add to your stress level, making you want to eat more. C. Increased exercise helps to reduce stress, so you may not need to spend money on a stress management class. D. Make sure to monitor your weight loss regularly to provide a sense of accomplishment and motivation.

The most important teaching is (A), so that the client will not begin a dangerous level of exercise when he is not sufficiently fit. This might result in chest pain, a heart attack, or stroke. (B, C, and D) are important instructions, but are of less priority than (A). Correct Answer: A

55. The nurse is reviewing a client's lab results from 2 hours ago. The sodium level is 128 mEq/L. The nurse should be alert for which findings? (Select all that apply.)

Weakness in the hands and feet +1 reflexes to the patella Headache Nausea

31. When assisting a client from the bed to a chair, which procedure is best for the nurse to follow?

With the nurse's feet spread apart and knees aligned with the client's knees, stand and pivot the client into the chair.

104. The nurse is making an initial daily assessment at 0715 and notes 550 mL of LR running at 75 mL an hour. At what time, in military time, will the nurse hang the next bag of IV fluid?

_1435____ Rationale: 60 min × 0.33333 = 19.99 min = 20 min 7 hr 20 min + 0715 = 1435

64. How many mL will the nurse document on the client's intake and output record from the items listed?

_____ Ml 2155 1200 mL water 4 ounce container of gelatin 8 ounces of orange juice 355 mL can of soda 1 cup of soup

Nursing assistant cannot do anything with:

assessing, change dressing or wounds, no teaching

The healthcare provider prescribes 1,000 ml of Ringer's Lactate with 30 Units of Pitocin to run in over 4 hours for a client who has just delivered a 10 pound infant by cesarean section. The tubing has been changed to a 20 gtt/ml administration set. The nurse plans to set the flow rate at how many gtt/min? A. 42 gtt/min. B. 83 gtt/min. C. 125 gtt/min. D. 250 gtt/min.

gtt/min = 20gtts/ml X 1000 ml/4hrs X 1 hr/60 min Correct Answer: B

113. The nurse is evaluating the chart of a client scheduled for surgery in 1 hour. When viewing the consent form, the nurse notes the surgeon's signature, but not the client's signature. What steps must the nurse take? (Select all that apply.)

Ask the client, "Did your surgeon explain the procedure to you? Ask the client, "Do you have any questions?" Witness the signature.

A client drank 7.5 oz of orange juice, 6 oz of tea, and 8 oz of eggnog. How many milliliters of fluid were consumed by the client? Record your answer using a whole number. __________ mL

One ounce (oz) equals approximately 30 mL. The client drank a total of 21.5 oz; multiply 21.5 × 30, which yields 645 mL.

68. The postoperative client states to the nurse, "When I had surgery last year I got constipated. It was miserable. What can I do to avoid constipation after this surgery this time?" (Select all that apply.)

"Drink approximately 3000 mL of non-caffeinated fluid per day." "I will make sure that you get out of bed an walk for 10 minutes, six times per day." "I will ask your healthcare provider for a prescription of docusate." "When you are on a regular diet, make sure you order plenty of fruits and vegetables."

121. The client reports to the clinic nurse, "I sleep for about 2 hours and then I have to get up to use the bathroom. I repeat that pattern about three to four times per night." What questions will the nurse include in this client's assessment? (Select all that apply.)

"How much fluid do you drink after 8:00 in the evening?" "What time of day do you take your water pill?" "Do you drink any alcoholic beverages in the evening?" "When did this pattern of urination start?" "Do you have any itching or burning when you urinate?"

39. While reviewing the side effects of a newly prescribed medication, a 72-year-old client notes that one of the side effects is a reduction in sexual drive. Which is the best response by the nurse?

"How will this affect your present sexual activity?"

109. In the middle of running a resuscitation for a cardiac arrest the LPN states to the nurse, "What can I do for your other patients?" The nurse says to the LPN, "Go ahead and start that blood on the client in 434B. It is primed and ready to go. You have seen me do it a million times. You can do it." What is the LPN's best response?

"I can take over compressions so you can start the blood."

18. An 89-year-old client is admitted to the rehabilitation unit after a hip fracture. When reviewing the client's pre-fracture routine, the client states, "I usually get up around 0800 and have breakfast by 0900; I say my daily prayers between 1000 and 1030. I like lunch around 1300; then a nap from 1400 to 1600. I generally eat supper around 1900." What is the nurse's best response to the client's schedule?

"Is there any way you could say your prayers between 1230 and 1300?"

78. One week after being told that she has terminal cancer with a life expectancy of 3 weeks, the client tells the nurse, "I think I will plan a big party for all my friends." How should the nurse respond?

"Planning a party and thinking about all your friends sounds like fun."

**A nurse is teaching a group of parents about child abuse. What definition of assault should the nurse include in the teaching plan? a. Assault is a threat to do bodily harm to another person. b. It is a legal wrong committed by one person against the property of another. c. It is a legal wrong committed against the public that is punishable by state law. d. Assault is the application of force to another person without lawful justification.

( A) Assault is a threat or an attempt to do violence to another. Assault implies harm to persons rather than property. A legal wrong committed against the public that is punishable by state law is too broad to describe assault. Application of force to another person without lawful justification is the definition of battery.

**A client with a leg prosthesis and a history of syncopal episodes is being admitted to the hospital. When formulating the plan of care for this client, the nurse should include that the client is at risk for: a. Falls b. Impaired cognition c. Imbalanced nutrition d. Impaired gas exchange

( A) The client is at risk for falls related to the leg prosthesis and history of syncope. There is no evidence or contributing factors in the patient scenario of the other nursing problems.

**A nurse who is working on a medical-surgical unit receives a phone call requesting information about a client who has undergone surgery. The nurse observes that the client requested a do not publish ("DNP") order on any information regarding condition or presence in the hospital. What is the best response by the nurse? a. "We have no record of that client on our unit. Thank you for calling." b. "The new privacy laws prevent me from providing any client information over the phone." c. "The client has requested that no information be given out. You'll need to call the client directly." d. "It is against the hospital's policy to provide you with any information regarding any of our clients."

( A) The response "We have no record of that client on our unit. Thank you for calling." conforms to the request that no information be given regarding the client's condition or presence in the hospital. HIPAA laws do not prohibit the provision of information to others as long as the client consents. The response "The client has requested that no information be given out. You'll need to call the client directly." implies that the client is admitted to the facility; this violates the client's request that no information should be shared with others. Hospital policies do not prohibit the provision of information to others as long as the client consents. The response "It is against the hospital's policy to provide you with any information regarding any of our clients." also implies that the client is admitted to the facility.

**A toddler screams and cries noisily after parental visits, disturbing all the other children. When the crying is particularly loud and prolonged, the nurse puts the crib in a separate room and closes the door. The toddler is left there until the crying ceases, a matter of 30 or 45 minutes. Legally, how should this behavior be interpreted? a. Limits had to be set to control the child's crying.rect2 b. The child had a right to remain in the room with the other children. c. The child had to be removed because the other children needed to be considered. d. Segregation of the child for more than half an hour was too long a period of time.

( B )Legally, a person cannot be locked in a room (isolated) unless there is a threat of danger either to the self or to others. Limit setting in this situation is not warranted. This is a reaction to separation from the parent, which is common at this age. Crying, although irritating, will not harm the other children. A child should never be isolated.

**The nurse teaching a health awareness class identifies which situation as being the highest risk factor for the development of a deep vein thrombosis (DVT)? a. Pregnancyrrect2 b. Inactivity c. Aerobic exercise d. Tight clothing

( B) A DVT, or thrombus, may form as a result of venous stasis. It may lodge in a vein and can cause venous occlusion. Inactivity is a major cause of venous stasis leading to DVT. Pregnancy and tight clothing are also risk factors for DVT secondary to inactivity. Aerobic exercise is not a risk factor for DVT.

**The nurse is caring for a client with a closed soft tissue injury. The nurse describes the injury as a/an: a. Abrasion b. Contusion c. Laceration d. Avulsion

( B) Closed wounds are considered contusions and hematomas because the skin is not broken. Abrasions, lacerations, and avulsions are considered open because there is a break in the skin integrity.

**A client expresses concern about being exposed to radiation therapy because it can cause cancer. What should the nurse emphasize when informing the client about exposure to radiation? a. The dosage is kept at a minimum. b. Only a small part of the body is irradiated. c. The client's physical condition is not a risk factor. d. Nutritional environment of the affected cells is a risk factor.

( B) Current radiation therapy accurately targets malignant lesions with pinpoint precision, minimizing the detrimental effects of radiation to healthy tissue. The dose is not as significant as the extent of tissue being irradiated. When radiation therapy is prescribed, the health care provider takes into consideration the ability of the client to tolerate the therapy, determining that the benefit outweighs the risk. Nutritional environment of the affected cells does not influence radiation's effect.

**Which nursing intervention is most appropriate for a client in skeletal traction? a. Add and remove weights as the client desires.rect2 b. Assess the pin sites at least every shift and as needed. c. Ensure that the knots in the rope are tied to the pulley. d. Perform range of motion to joints proximal and distal to the fracture at least once a day.

( B) Nursing care for a client in skeletal traction may include assessing pin sites every shift and as needed. The needed weight for a client in skeletal traction is prescribed by the physician, not as desired by the client. The nurse also should ensure that the knots are not tied to the pulley and move freely. The performance of range of motion is indicated for all joints except the ones proximal and distal to the fracture, since this area is immobilized by the skeletal traction to promote healing and prevent further injury and pain.

**A nurse overhears an unlicensed assistive personnel (UAP) talking with a client about the client's marital and family problems. The nurse identifies that the UAP is providing false reassurance when the UAP states: a. "I agree; I think you should get a divorce." b. "Everything will be fine, just wait and see." c. "You should be glad that you have such a loving family." d. "In the scheme of things, you do not have a major problem."

( B) Saying that everything will be fine provides false hope. Agreeing with the client is an example of offering approval. Commenting on how a client should feel is an example of being judgmental. Implying that the problem is minor is an example of minimizing.

**A newly hired nurse, during orientation, is approached by a surveyor from the department of health. The surveyor asks the nurse about the best way to prevent the spread of infection. What is the most appropriate nursing response? a. "Let me get my preceptor." b. "Wash your hands before and after any client care." c. "Clean all instruments and work surfaces with an approved disinfectant." d. "Ensure proper disposal of all items contaminated with blood or body fluids."

( B) The best means to prevent the spread of infection is to break the chain of infection. This is most easily accomplished by the simple act of handwashing before and after all client contact. "Let me get my preceptor" and "Clean all instruments and work surfaces with an approved disinfectant" may be correct, but they are not the best responses for this situation. It is not necessary that all items contaminated with blood or body fluids be disposed.

**The nurse recognizes that what is the reason the faucets on the sinks in a client's room are considered contaminated? a. They are not in sterile areas.ect2 b. They are touched by dirty hands when turning the water on. c. There are large numbers of people who use them each day. d. Water encourages bacterial growth.

( B) Unwashed hands are considered contaminated and are used to turn on sink faucets. Recontamination of washed hands may be prevented by using foot pedals or a paper towel barrier when closing the faucets. They are not considered contaminated because they are not in sterile areas; areas cannot be sterile. It is unrelated to the number of people but rather to being touched by contaminated hands. Although bacterial growth is facilitated in moist environments, this is not why sink faucets are considered contaminated.

**The triage nurse in the emergency department receives four clients simultaneously. Which of the clients should the nurse determine can be treated last? a. Multipara in active labor b. Middle-aged woman with substernal chest painrrect3 c. Older adult male with a partially amputated finger d. Adolescent boy with an oxygen saturation of 91%

( C ) Although a client with a partially amputated finger needs control of bleeding, the injury is not life threatening and the client can wait for care. A woman in active labor should be assessed immediately because birth may be imminent. A woman with chest pain may be experiencing a life-threatening illness and should be assessed immediately. An adolescent with significant hypoxia may be experiencing a life-threatening illness and should be assessed immediately.

**Which statement by an older adult most strongly supports the nurse's conclusion that the client is impacted with stool? a. "I have a lot of gas pains." b. "I don't have much of an appetite." c. "I feel like I have to go and just can't."ncorrect4 d. "I haven't had a bowel movement for several days."

( C) A client with a fecal impaction has the urge to defecate but is unable to do so. Flatulence may occur as a result of immobility, not just obstruction. Anorexia may occur with an impaction but also may be caused by other conditions. The frequency of bowel movements varies for individuals; it may be normal for this individual not to have a bowel movement for several days.

**The nurse is caring for a client admitted with chronic obstructive pulmonary disease (COPD). The nurse should monitor the results of which laboratory test to evaluate the client for hypoxia? a. Red blood cell count b. Sputum cultureCorrect3 c. Arterial blood gas d. Total hemoglobin

( C) All of these laboratory tests assist in the evaluation of a client with respiratory difficulties; however, arterial blood gas analysis is the only test that evaluates gas exchange in the lungs. This provides accurate information about the client's oxygenation status.

**The nurse caring for a client with a systemic infection is aware that the assessment finding that is most indicative of a systemic infection is: Incorrect1 a. White blood cell (WBC) count of 8200/mm3 b. Bilateral 3+ pitting pedal edema c. temperature of 101.3º F d. Pale skin and nail beds

( C) An elevated temperature of 101.3° F is most indicative of a systemic infection. A white blood cell (WBC) count of 8200/mm3 is within the WBC normal range of 5000-10,000/mm3. Pedal edema is generally not related to an infectious process. Pale skin and nail beds may be related to an infectious process but not necessarily.

**A client is admitted voluntarily to a psychiatric unit. Later, the client develops severe pain in the right lower quadrant and is diagnosed as having acute appendicitis. How should the nurse prepare the client for the appendectomy? a. Have two nurses witness the client signing the operative consent form.rrect2 b. Ensure that the health care provider and the psychiatrist sign for the surgery because it is an emergency procedure. c. Ask the client to sign the operative consent form after the client has been informed of the procedure and required care. d. Inform the client's next of kin that it will be necessary for one of them to sign the consent form because the client is on a psychiatric unit.

( C) Because the client is not certified as incompetent, the right of informed consent is retained. The client can sign the consent, but the client's signature requires only one witness. Because there is no evidence of incompetence, the client should sign the consent.

**What should the nurse assess to determine whether a 75-year-old individual is meeting the developmental task associated with aging? Incorrect1 a. Achievement of a personal philosophy b. Adaptation to the children leaving home c. Attainment of a sense of worth as a person d. Adjustment to life in an assisted-living facility

( C) Developing and participating in meaningful activities and satisfaction with past accomplishments increase feelings of self-worth. Achievement of a personal philosophy is a task of early adulthood. Adaptation to the children leaving home is a task of middle adulthood. Adjustment to life in an assisted-living facility is not a developmental task of older adults; not all older adults live in assisted-living facilities.

**A nurse is caring for a client during the transition phase of labor. The nurse determines that the client has entered the second stage of labor when: a. There is restlessness and thrashing about b. There are complaints of sudden and intense back pain c. The client reports that she feels the urge to move her bowels d. The client asks for medication to relieve pain from the strong contractions

( C) During the second stage the presenting part is low in the birth canal and may cause strong sensations of pressure on the rectum; at this time the cervix is fully dilated and the urge to push is great. Restlessness and thrashing about usually begins during the transition phase of the first stage of labor. Complaints of sudden, intense back pain may occur with persistent posterior pressure; usually the pain does not have a sudden onset. Asking for medication to relieve pain from the strong contractions usually occurs during the active phase of the first stage of labor.

**Using the five-digit system, determine the obstetric history in this situation: The client is 38 weeks into her fourth pregnancy. Her third pregnancy, a twin gestation, ended at 32 weeks with a live birth, her second pregnancy ended at 38 weeks with a live birth, and her first pregnancy ended at 18 weeks. a. G4, T2, P1, A1, L2 b. G4, T1, P2, A1, L1 c. G4, T1, P1, A1, L3 d. G4, T2, P1, A1, L1

( C) Four pregnancies = G (gravida ) 4. One pregnancy that ended at 38 weeks = T (term) 1. One pregnancy that ended at 32 weeks = P (preterm) 1. One pregnancy that ended at 18 weeks = A (abortion) 1. One set of twins and a singleton = L (living) 3.

**A senior high school student, whose immunization status is current, asks the school nurse which immunizations will be included in the precollege physical. Which vaccine should the nurse tell the student to expect to receive? a. Hepatitis C (HepC) b. Influenza type B (HIB) c. Measles, mumps, rubella (MMR)rect4 d. Diphtheria, tetanus, pertussis (DTaP)

( C) Individuals born after 1957 should receive one additional dose of MMR vaccine if they are students in postsecondary educational institutions. Currently there is no vaccine for hepatitis C. The HIB immunization is unnecessary. If the student received an additional DTaP at age 12, it is not necessary. A booster dose of tetanus toxoid (Td) should be received every 10 years.

**A client on hospice care is receiving palliative treatment. A palliative approach involves planning measures to: a. Restore the client's health b. Promote the client's recoveryrrect3 c. Relieve the client's discomfort d. Support the client's significant others

( C) Palliative measures are aimed at relieving discomfort without curing the problem. A cure or recovery is not part of palliative care; with a terminal disease the other goals are unrealistic. Although support of significant others is indicated, palliative care is related directly to relieving the client's discomfort.

**A nurse has provided discharge instructions to a client who received a prescription for a walker to use for assistance with ambulation. The nurse determines that the teaching has been effective when the client: a. Picks up the walker and carries it for short distances. b. Uses the walker only when someone else is present. c. Moves the walker no more than 12 inches in front of the client during use. d. States that a walker will be purchased on the way home from the hospital.

( C) Safety is always a consideration when teaching a client how to use an assistive device. Therefore the correct procedure regarding using a walker is to move the walker no more than 12 inches in front to maintain balance and to be effective in forward movement. Carrying the walker when ambulating is incorrect. Once the client is instructed and can demonstrate correct use of a walker, there is no need for someone to be present every time the client uses the walker. If the client is ordered to use a walker as part of the discharge plan, it needs to be provided before leaving the hospital.

**A nurse discusses the philosophy of Alcoholics Anonymous (AA) with the client who has a history of alcoholism. What need must self-help groups such as AA meet to be successful? a. Trustrect2 b. Growth c. Belonging d. Independence

( C) Self-help groups are successful because they support a basic human need for acceptance. A feeling of comfort and safety and a sense of belonging may be achieved in a nonjudgmental, supportive, sharing experience with others. AA meets dependency needs rather than focusing on independence, trust, and growth.

**The health care provider prescribes a low-fat, 2-gram sodium diet for a client with hypertension. The nurse should explain that the purpose of restricting sodium is to: a. Chemically stimulate the loop of Henle b. Diminish the thirst response of the client c. Prevent reabsorption of water in the distal tubules d. Cause fluid to move toward the interstitial compartment

( C) Sodium absorbs water in the kidneys' renal tubules. When dietary intake of sodium is decreased, water is not reabsorbed and edema is reduced. A decrease in sodium will prevent the reabsorption of water. Furosemide stimulates the loop of Henle to inhibit the reabsorption of sodium and chloride at the proximal and distal tubules. Adequate hydration is the major factor that diminishes the thirst response. A low-sodium diet will help move fluid from the interstitial compartment to the intravascular compartment. **A client has 4 ounces of apple juice, 6 ounces of tea, and 240 mL of chicken broth. The nurse calculates that the client ingested how many mL of fluid? Record your answer using a whole number. __________ mL-- 540 mL is a correct calculation. 4 ounces apple juice x 30 mL/ounce = 120 mL, 6 oz tea x 30 mL/ounce = 180 mL, and 240 mL chicken broth provide a total of 540 mL that the client has ingested.

**Which drug requires the nurse to monitor the client for signs of hyperkalemia? a. Furosemide (Lasix) b. Metolazone (Zaroxolyn)rrect3 c. Spironolactone (Aldactone) d. Hydrochlorothiazide (HydroDIURIL)

( C) Spironolactone is a potassium-sparing diuretic; hyperkalemia is an adverse effect. Furosemide, metolazone, and hydrochlorothiazide generally cause hypokalemia.

**A nurse hired to work in a metropolitan hospital provides services for a culturally diverse population. One of the nurses on the unit says it is the nurses' responsibility to discourage "these people" from bringing all that "home medicine stuff" to their family members. Which response by the recently hired nurse is most appropriate? a. "Hospital policies should put a stop to this." b. "Everyone should conform to the prevailing culture." c. "Nontraditional approaches to health care can be beneficial."orrect4 d. "You are right because they may have a negative impact on people's health."

( C) Studies demonstrate that some nontraditional therapies are effective. Culturally competent professionals should be knowledgeable about other cultures and beliefs. Many health care facilities are incorporating both Western and nontraditional therapies. The statement "Everyone should conform to the prevailing culture" does not value diversity. The statement "You are right because they may have a negative impact on people's health" is judgmental and prejudicial. Some cultural practices may bring comfort to the client and may be beneficial, and they may not interfere with traditional therapy.

**A client reaches the point of acceptance during the stages of dying. What response should the nurse expect the client to exhibit? Incorrect1 a. Apathy b. Euphoria c. Detachment d. Emotionalism

( C) When an individual reaches the point of being intellectually and psychologically able to accept death, anxiety is reduced and the individual becomes detached from the environment. Although detached, the client is not apathetic but still may be concerned and use time constructively. Although resigned to death, the individual is not euphoric. In the stage of acceptance, the client is no longer angry or depressed.

**A nurse is caring for a client with acute kidney failure who is receiving a protein restricted diet. The client asks why this diet is necessary. What information should the nurse include in a response to the client's questions? a. A high-protein intake ensures an adequate daily supply of amino acids to compensate for losses. b. Essential and nonessential amino acids are necessary in the diet to supply materials for tissue protein synthesis. c. This supplies only essential amino acids, reducing the amount of metabolic waste products, thus decreasing stress on the kidneys. d. Urea nitrogen cannot be used to synthesize amino acids in the body, so the nitrogen for amino acid synthesis must come from the dietary protein.

( C) The amount of protein permitted in the diet (usually less than 50 g) depends on the extent of kidney function; excess protein causes an increase in urea concentration, which should be avoided Adequate calories are provided to prevent tissue catabolism, which also results in an increase in metabolic waste products. In kidney failure the kidneys are unable to eliminate the waste products of a high protein diet. The body is able to synthesize the nonessential amino acids. Urea is a waste product of protein metabolism; the body is able to synthesize the nonessential amino acids.

**A client is admitted with a diagnosis of premature labor. The nurse discovers that the client has been using heroin throughout her pregnancy. What is the most appropriate action for the nurse to take? a. Notify the nurse manager of the unit. b. Inform no one because all client information is confidential.t3 c. Inform the client's healthcare provider. d. Alert the hospital security department because heroin is an illegal substance.

( C) The fetus of a heroin-addicted mother is at risk for serious complications such as hypoxia and meconium aspiration. It is important to notify the healthcare provider of the client's heroin use, because this information will influence the care of the client and newborn. This information is used only in relation to the client's care. With the client's consent, it may be shared with other social service or health agencies that become involved with the client's long-term care. The nurse manager of the unit may be notified as it relates to the care of the client and her newborn. Client information is confidential and only necessary staff should be privileged to such information. Hospital security would only be notified if actual illicit substances were discovered on hospital premises.

**An 85-year-old client is alert and able to participate in care. The nurse understands that, according to Erikson, a person's adjustment to the period of senescence will depend largely on adjustment to which developmental stage? a. Industry versus inferiority b. Identity versus role confusionorrect3 c. Generativity versus stagnation d. Autonomy versus shame/doubt

( C) The generativity versus stagnation stage precedes integrity versus despair; Erikson theorized that how well people adapt to a present stage depends on how well they adapted to the immediately preceding stage. Industry versus inferiority is the stage of school-age children; it precedes identity versus role confusion, not integrity versus despair. Identity versus role confusion is the stage of adolescence; it precedes intimacy versus isolation, not integrity versus despair. Autonomy versus shame/doubt is the stage of early childhood; it precedes initiative versus guilt, not integrity versus despair.

**A nurse identifies that an older adult has not achieved the desired outcome from a prescribed proprietary medication. When assessing the situation, the client shares that the medication is too expensive and the prescription was never filled. What is an appropriate nursing response? a. Ask the pharmacist to provide a generic form of the medication. b. Encourage the client to acquire the medication over the internet.rrect3 c. Inform the health care provider of the inability to afford the medication. d. Suggest that the client purchase insurance that covers prescription medications.

( C) The health care provider needs to be aware of the reason for the client's lack of response to the medication so that an alternate treatment plan or financial assistance can be arranged (e.g., go to The National Council on the Aging web site [BenefitsCheckUpRx] to establish whether the client is eligible for assistance from any community, state, or federal programs or from the drug company). A health care provider may prefer the proprietary form of the medication. To ask the pharmacist to provide a generic form of the medication is unsafe. To recommend that the client obtain a generic form of the medication is not within the legal role of the nurse, unless the health care provider documents that this is acceptable. Medications purchased over the internet may be illegally imported, counterfeit, expired, or contaminated and therefore should be avoided. Although some prescription insurance plans may help to reduce the cost of some medications, the client may not be able to afford the insurance.

**A hospital has threatened to refuse the discharge of a newborn until the parents pay part of the hospital bill. The nurse is aware that the legal term that best describes this situation is: a. False threats b. Assault and battery c. False imprisonment d. Breach of confidentiality

( C) The hospital is threatening to keep the infant; therefore false imprisonment is threatened. False imprisonment is restraining or confining a person without a clinical reason. False threat may be a term to describe false imprisonment; however it is inaccurate in this situation. Assault and battery legally means to threaten violence and the physical act of violence. Breach of confidentiality is a disclosure to a third party, without client consent or court order of private information.

**Which nursing action is confidential and protected from legal action? a. Providing health teaching regarding family planning. b. Offering first aid at the scene of an automobile collision. c. Reporting incidents of suspected child abuse to the appropriate authorities. d. Administering resuscitative measures to an unconscious child pulled from a swimming pool.

( C) The reporting of possible child abuse is required by law, and the nurse's identity can remain confidential. The nurse is functioning in a professional capacity and therefore can be held accountable. Although the Good Samaritan Act protects health professionals, the nurse is still responsible for acting as any reasonably prudent nurse would in a similar situation.

64 A client has a hiatal hernia. The client is 5 feet 3 inches tall and weighs 140 pounds. When the nurse discusses prevention of esophageal reflux, what should be included? a. "Increase your intake of fat with each meal." b. "Lie down after eating to help your digestion." c. "Reduce your caloric intake to foster weight reduction." d. "Drink several glasses of fluid during each of your meals."

( C) Weight reduction decreases intraabdominal pressure, thereby decreasing the tendency to reflux into the esophagus. Fats decrease emptying of the stomach, extending the period that reflux can occur; fats should be decreased. Lying down after eating increases the pressure against the diaphragmatic hernia, increasing symptoms. Drinking several glasses of fluid during each meal will increase pressure; fluid should be discouraged with meals

**A client is diagnosed with AIDS. When examining the client's oral cavity, the nurse assesses white patchy plaques on the mucosa. The nurse recognizes that this finding most likely represents what opportunistic infection? a. Cytomegalovirus b. Histoplasmosist3 c. Candida albicans d. Human papillomavirus

( C) White patchy plaques on the oral mucosa would most likely be a result of C. albicans, a yeastlike fungal infection. This condition is also known as "thrush." Cytomegalovirus may cause a serious viral infection in persons with HIV, resulting in retinal, gastrointestinal, and pulmonary manifestations. Histoplasmosis is an infection caused by inhalation of spores of the fungus Histoplasma capsulatum and is characterized by fever, malaise, cough, and lymphadenopathy. Human papillomavirus typically manifests as warts on the hands and feet as well as mucous membrane lesions of the oral, anal, and genital cavities. It may be transmitted without the presence of warts through body fluids with some forms associated with cancerous and precancerous conditions.

**A nurse suspects that a client has poison ivy. Assessment findings reveal vesicles on the arms and legs. A vesicle can be described as: a. A lesion filled with purulent drainage. b. An erosion into the dermis. c. A solid mass of fibrous tissue. d. A lesion filled with serous fluid.

( D) A vesicle is a small blisterlike elevation on the skin containing serous fluid. Vesicles are usually transparent. Common causes of vesicles include herpes, herpes zoster, and dermatitis associated with poison oak or ivy. A lesion filled with purulent drainage is known as a pustule; an erosion into the dermis is known as an excoriation or ulcer; and a solid mass of fibrous tissue is known as a papule.

**The nurse instructs a client with a new colostomy to avoid foods and drinks that produce a large amount of gas, and specifically to avoid the intake of: a. Milk b. Cheese c. Coffeet4 d. Cabbage

( D) Cabbage is a gas-producing food that can cause a client with a colostomy problems with odor control and ballooning of the ostomy bag, which may break the device seal and allow leakage. Milk, cheese, and coffee, in moderation, should not cause excessive gas problems. The client with a new colostomy should slowly introduce new foods into the diet to test toleration.

**A client is admitted to the hospital because of multiple chronic health problems. What is the priority nursing intervention at this time? a. Advising the client to join a support group immediately after discharge. b. Assuring the family that staff members will take care of the client's needs.rrect3 c. Reminding the client to keep medical follow-up appointments after discharge. d. Conducting a multidisciplinary staff conference early during the client's hospitalization.

( D) Collaboration of all team members involved in the client's care early during hospitalization will allow for efficient planning of care and help prepare for discharge. The client may or may not be ready to join a support group at that time. Assuring the family that staff members will take care of the client's needs may promote dependence. The client should be encouraged to assume self-care gradually. Although this should be done eventually, it is not the priority at this time.

**What should the nurse include in dietary teaching for a client with a colostomy? a. Liquids should be limited to 1 L per day. b. Non-digestible fiber and fruits should be eliminated. c. A formed stool is an indicator of constipation.ct4 d. The diet should be adjusted to include foods that result in manageable stools.

( D) Each person will need to experiment with diet after a colostomy to determine what foods are best tolerated and also produce stools that are manageable, depending on the type of colostomy. Liquids are typically not limited unless there is a specific reason such as cardiac or renal disease. Food high in fiber such as fruit should be included in the diet as tolerated. Depending on the type of colostomy and the diet, a formed stool is acceptable and does not indicate a constipating diet.

**The nurse is caring for a client that underwent a rhinoplasty surgical procedure 5 hours ago. After administering pain medication, the nurse notes the client is swallowing frequently. The nurse understands that the cause of frequent swallowing is most likely caused from: a. A normal response to the analgesic b. Oral dryness caused by nasal packing c. An adverse reaction to anesthesia d. Bleeding posterior to the nasal packing

( D) Frequent swallowing may indicate bleeding in the posterior pharynx. Oral dryness causes thirst, not an increase in swallowing. Frequent swallowing is not a normal response to rhinoplasty or analgesics/anesthesia.

**A postoperative client says to the nurse, "My neighbor—I mean the person in the next room—sings all night and keeps me awake." The neighboring client has dementia and is awaiting transfer to a nursing home. How can the nurse best handle this situation? a. Tell the neighboring client to stop singing. b. Close the doors to both clients' rooms at night. c. Give the complaining client the prescribed as needed sedative.4 d. Move the post-operative client to a room at the end of the hall.

( D) Moving the post-operative client to a room at the end of the hall would address that client's concerns. A client with dementia will not remember instructions. It is unsafe to close the doors of clients' rooms because they need to be monitored. The use of a sedative should not be the initial intervention.

**The nurse is providing information about blood pressure to Unlicensed Assistive Personnel (UAP) and recalls that the factor that has the greatest influence on diastolic blood pressure is: a. Renal functiont2 b. Cardiac output c. Oxygen saturation d. Peripheral vascular resistance

( D) Peripheral vascular resistance is the impedance of blood flow, or back pressure, by the arterioles, which is the most influential component of diastolic blood pressure. Renal function through the renin-angiotensin-aldosterone system regulates fluid balance and does influence blood pressure. Cardiac output is the determinant of systolic blood pressure. Oxygen saturation does not have a direct effect on diastolic blood pressure.

**A client using fentanyl (Duragesic) transdermal patches for pain management in late-stage cancer dies. What should the hospice nurse who is caring for this client do about the patch? a. Tell the family to remove and dispose of the patch. b. Leave the patch in place for the mortician to remove. c. Have the family return the patch to the pharmacy for disposal. d. Remove and dispose of the patch in an appropriate receptacle.

( D) The nurse should remove and dispose of the patch in a manner that protects self and others from exposure to the fentanyl. Having the family remove and dispose of the patch or having the mortician remove the patch are not the responsibility of nonprofessionals because they do not know how to protect themselves and others from exposure to the fentanyl. It is unnecessary to return a used fentanyl patch.

**Health promotion efforts with the chronically ill client should include interventions related to primary prevention. What should this include? a. Encouraging daily physical exercise b. Performing yearly physical examinations c. Providing hypertension screening programs d. Teaching a person with diabetes how to prevent complications

(A ) Primary prevention activities are directed toward promoting healthful lifestyles and increasing the level of well-being. Performing yearly physical examinations is a secondary prevention. Emphasis is on early detection of disease, prompt intervention, and health maintenance for those experiencing health problems. Providing hypertension screening programs is a secondary prevention. Emphasis is on early detection of disease, prompt intervention, and health maintenance for those experiencing health problems. Teaching a person with diabetes how to prevent complications is a tertiary prevention. Emphasis is on rehabilitating individuals and restoring them to an optimum level of functioning.

**A nurse is evaluating a client's response to fluid replacement therapy. Which clinical finding indicates adequate tissue perfusion to vital organs? a. Urinary output of 30 mL in an hour b. Central venous pressure reading of 2 mm Hg c. Baseline pulse rate of 120 that decreases to 110 beats/min within a 15-minute period d. Baseline blood pressure of 50/30 that increases to 70/40 mm Hg within a 30-minute period

(A) A urinary output rate of 30 mL/hour is considered adequate for perfusion of the kidneys, heart, and brain. A central venous pressure reading of 2 mm Hg indicates hypovolemia. A baseline pulse rate of 120 that decreases to 110 beats/min within a 15-minute period and a baseline blood pressure of 50/30 that increases to 70/40 mm Hg within a 30-minute period indicate improvement but not necessarily adequate tissue perfusion.

**A nurse is providing dietary instruction to a client with cardiovascular disease. Which dietary selection by the client indicates the need for further instruction? a. Whole milk with oatmeal b. Garden salad with olive oil c. Tuna fish with a small apple d. Soluble fiber cereal with skim milk

(A) Although oatmeal is a soluble fiber, whole milk is high in saturated fat and should be avoided. Olive oil contains unsaturated fat. Most fish have a low fat content; fruit does not contain fat. Soluble fiber helps to lower cholesterol; skim milk does not contain fat.

**A back brace is prescribed for a client who had a laminectomy. What instruction should the nurse include in the teaching plan? a. Apply the brace before getting out of bedct2 b. Put the brace on while in the sitting position c. Use the brace when the back begins to feel tired d. Wear the brace when performing twisting exercises

(A) Appling the brace before getting out of bed is done while in the supine position before the body is subjected to the force of gravity in a vertical position. Anatomical landmarks are easier to locate for correct application of the brace, and intraabdominal organs have not shifted toward the pelvic floor by gravity. The brace should be applied while in the supine position, not the sitting position. The brace should be worn as prescribed, not just when the client feels tired. Twisting exercises are contraindicated because they exert excessive pressure on the operative site.

**A client who is recuperating from a spinal cord injury at the T4 level wants to use a wheelchair. What should the nurse teach the client to do in preparation for this activity? a. Push-ups to strengthen arm muscles b. Leg lifts to prevent hip contractures c. Balancing exercises to promote equilibrium d. Quadriceps-setting exercises to maintain muscle tone

(A) Arm strength is necessary for transfers and activities of daily living and for use of crutches or a wheelchair. Equilibrium is not a problem. The client does not have neurological control of the other activities.

**A client has cholelithiasis with possible obstruction of the common bile duct. What should be determined about the client's nutritional status before surgery is scheduled? Correct a. Is the client deficient in vitamins A, D, and K? b. Does the client eat adequate amounts of dietary fiber? c. Does the client consume excessive amounts of protein? d. Are the client's levels of potassium and folic acid increased?

(A) Bile promotes the absorption of fat-soluble vitamins; an obstruction of the common bile duct limits the flow of bile to the duodenum. Knowing if the client eats adequate amounts of dietary fiber is not relevant to the situation. Knowing if the client consumes excessive amounts of protein is unnecessary; however, protein is desirable for wound healing. An increase in potassium and folic acid are not expected.

**Which alternative therapy may be beneficial for the nurse to discuss with a client who has terminal bone cancer? a. biofeedback b. radiotherapy c. bariatric therapy d. radioactive implants

(A) Biofeedback provides information about changes in body function; clients can learn to use this to control a variety of body responses, including pain. Radiotherapy is a part of standard medical regimens. Bariatrics is a type of therapy that focuses on the correction of obesity; it encompasses prevention, control, and treatment of the problem, which involves medications and surgery. Placement of radioactive sources into or in contact with tissues (brachytherapy) is part of standard medical treatment for cancer.

**A nurse in a rehabilitation center teaches clients with quadriplegia to use an adaptive wheelchair. Why is it important that the nurse provide this instruction? a. It is unlikely that the client will regain the ability to walk. b. It prepares them for wearing braces. c. It assists them in overcoming orthostatic hypotension. d. They have the strength in the upper extremities for self-transfer.

(A) Clients with quadriplegia do not have the muscle innervation, strength, or balance needed for ambulation. Bracing and crutch-walking require muscle strength and coordination that an individual with quadriplegia does not have. Orthostatic hypotension can be prevented by a gradual assumption of the upright position and does not necessarily require a wheelchair. Quadriplegia refers to paralysis of all four extremities.

After a laryngectomy is scheduled, the most important factor for the nurse to include in the preoperative teaching plan is: Correct1 a. Establishing a means for communicating postoperatively b. Explaining that there will be a feeding tube postoperatively c. Demonstrating how to care for a permanent laryngeal stoma d. Teaching how to cough to expectorate bronchial secretions effectively

(A) Communication is a priority; it facilitates interaction, limits anxiety, and promotes safety. A nasogastric tube can cause trauma to the suture lines; total parenteral nutrition may be used. Demonstrating how to care for a permanent laryngeal stoma is done postoperatively as the client begins to accept the laryngectomy. After a laryngectomy the client cannot cough; expectoration occurs through the stoma.

**A client is admitted to the hospital for the medical management of burns over 18% of the body's surface. What should the nurse teach the client to help manage pain during dressing changes? a. Deep breathing exercises b. Progressive muscle relaxation c. Active range-of-motion exercises d. Important elements of wound care

(A) Deep breathing exercises are an effective intervention in controlling pain; this is a positive coping skill. Muscle relaxation techniques generally include muscle contraction and then relaxation, which may increase the pain. Active range-of-motion exercises may increase the pain. Understanding important aspects of wound care will not reduce pain; health teaching should be initiated before, not during, a procedure.

**A client is experiencing stomatitis as a result of chemotherapy. Which nursing action is most appropriate when caring for this client? a. Provide frequent saline mouthwashes. b. Use karaya powder to decrease irritation. c. Increase fluid intake to compensate for accompanying diarrhea. d. Provide meticulous skin care of the abdomen with an antiseptic

(A) Saline mouthwashes are soothing to the oral mucosa and help clean the mouth, minimizing infection. Stomatitis refers to the oral cavity; karaya is used to protect the skin around a stoma created on the abdomen. Stomatitis does not cause diarrhea or fluid loss. The abdomen is not involved; stomatitis is an inflammation of the oral mucosa.

**A client who has been in a coma for two months is being maintained on bed rest. The nurse concludes that to prevent the effects of shearing force, the head of the bed should be maintained at an angle of: a. 30 degrees b. 45 degrees c. 60 degrees d. 90 degrees

(A) Shearing force occurs when two surfaces move against each other; when the bed is at an angle greater than 30 degrees, the torso tends to slide and causes this phenomenon. Fortyfive degrees, 60 degrees, and 90 degrees raise the head of the bed too high, which contributes to the client sliding down in bed.

**What intervention is included in the nursing care plan for a 4-month-old infant with tetralogy of Fallot and heart failure? a. Providing small, frequent feedings. b. Positioning the child flat on the back c. Encouraging nutritional fluids often. d. Measuring the head circumference daily.

(A) Small, frequent feedings with adequate rest periods in between may improve the infant's intake at each feeding; infants with tetralogy of Fallot become extremely fatigued while suckling. Positioning the child with the head elevated facilitates respiration; an infant cardiac seat, similar to a car seat, helps maintain the child in the semi-Fowler position. As a means of reducing the cardiac workload, excessive fluids usually are not offered, and fluids may even be restricted. The head circumference is not an important assessment for infants with congenital heart disease; daily head measurements should be taken for infants with hydrocephaly.

**A client newly diagnosed with myasthenia gravis is concerned about fluctuations in physical condition and generalized weakness. When caring for this client it is most important for the nurse to plan to: a. Space activities throughout the day b. Restrict activities and encourage bed rest c. Teach the client about limitations imposed by the disorder d. Have a family member stay at the bedside to give the client support

(A) Spacing activities encourages maximum functioning within the limits of the client's strength and endurance. Bed rest and limited activity may lead to muscle atrophy and calciumdepletion. Teaching the limitations imposed by the disorder is necessary for lifelong psychological adjustment, but does not address the client's concerns at this time. Having a member of the family stay and give the client support should be permitted if requested by the client or family, but does not address the concerns voiced by the client.

**A physician orders a urinalysis for a client with an indwelling catheter. To ensure that an appropriate specimen is obtained, the nurse would obtain the specimen from which site? a. tubing injection port b. distal end of the tubing c. urinary drainage bag d. catheter insertion site

(A) The appropriate site to obtain a urine specimen for a patient with an indwelling catheter is the injection port. The nurse should clean the injection port cap of the catheter drainage tubing with appropriate antiseptic, attach a sterile 5-mL syringe into the port and aspirate the quantity desired. The nurse should apply a clamp to the drainage tubing, distal to the injection port, not obtain the specimen from this site. Urine in the bedside drainage bag is not an appropriate sample, as the urine in the bag may have been there too long; thus a clean sample cannot be obtained from the bag. The client's urine will be contained in the indwelling catheter; there will be no urine at the insertion site.

**A client is cautioned to avoid vitamin D toxicity while increasing protein intake. Which nutrient selected by the client indicates to the nurse that the dietary teaching is understood? a. Tofu b. Eggnog c. Cottage cheese d. Powdered whole milk

(A) Tofu products increase protein without increasing vitamin D because, unlike milk products, tofu does not contain vitamin D. Eggnog contains milk and should be avoided. Cottage cheese, a milk product, contains vitamin D, which should be avoided. Powdered whole milk contains vitamin D and should be avoided.

**A 75-year-old female client tells the nurse that she read about a vitamin that may be related to aging because of its relationship to the structure of cell walls. The nurse determines that the client is probably referring to: a. Vitamin E b. Vitamin A c. Vitamin C d. Vitamin B1

(A) Vitamin E hinders oxidative breakdown of structural lipid membranes in body tissues caused by free radicals in the cells. Vitamin A assists in the formation of visual purple needed for night vision. Vitamin C is used for formation of collagen, which is important for maintaining capillary strength, promoting wound healing, and resisting infection. Vitamin B1 is necessary for protein and fat metabolism and normal function of the nervous system

Heparin 20,000 units in 500 ml D5W at 50 ml/hour has been infusing for 5½ hours. How much heparin has the client received? A. 11,000 units. B. 13,000 units. C. 15,000 units. D. 17,000 units.

(A) is the correct calculation: 20,000 units/500 ml = 40 units (the amount of units in one ml of fluid). 40 units/ml x 50 ml/hr = 2,000 units/hour (1,000 units in 1/2 hour). 5.5 x 2,000 = 11,000 (A). OR, multiply 5 x 2,000 and add the 1/2 hour amount of 1,000 to reach the same conclusion = 11,000 units. Correct Answer: A

**A client with heart failure is on a drug regimen of digoxin (Lanoxin) and furosemide (Lasix). The client dislikes oranges and bananas. Which fruit should the nurse encourage the client to eat? a. Apples b. Grapes c. Apricots d. Cranberries

(C) Lasix is potassium depleting; apricots have more than 440 mg of potassium per 100 g. Apples have about 80 to 110 mg of potassium per 100 g. Grapes have about 80 to 160 mg of potassium per 100 g, depending on the variety. Cranberries have about 65 mg ofpotassium per 100 g.

**Discharge planning for an ambulatory client with Parkinson disease (PD) includes recommending equipment for home use that will help with activities of daily living. To foster independence, the nurse should promote the use of: a. Raised toilet seat b. Side rails for the bed c. Trapeze above the bed d. Crutches for ambulation

(A) raised toilet seat will reduce strain on the back muscles and make it easier for the client to rise from the seat without injury. The client is not bedridden and will not need side rails for the bed or a trapeze above the bed. Clients with Parkinson disease have poor balance and a propulsive gait, which makes it unsafe to use crutches.

**Nurses care for clients in a variety of age groups. In which age group is the occurrence of chronic illness the greatest? a. Older adults b. Adolescents c. Young childrenIncorrect4 d. Middle-aged adults

(A)) Chronic illness increases in older adults because of the multiple stresses of aging. Younger individuals have greater physiological reserves, and chronic illnesses are not common.

**Health promotion efforts within the health care system should include efforts related to secondary prevention. Which activities reflect secondary prevention interventions in relation to health promotion? (Select all that apply.) a. Encouraging regular dental checkups b. Facilitating smoking cessation programs c. Administering influenza vaccines to older adults d. Teaching the procedure for breast self-examination e. Referring clients with a chronic illness to a support group

(A, D) Encouraging regular dental checkups is a secondary prevention activity because it emphasizes early detection of health problems, such as dental caries and gingivitis. Teaching the procedure for breast self-examination is a secondary prevention activity because it emphasizes early detection of problems of the breast, such as cancer. Facilitating smoking cessation programs is a primary prevention activity because it emphasizes health protection against heart and respiratory diseases. Administering influenza vaccines to older adults is a primary prevention activity because it emphasizes health protection against influenza. Referring clients with a chronic illness to a support group is a tertiary prevention activity because it emphasizes care that is provided after illness already exists.

**Which nursing interventions require a nurse to wear gloves? (Select all that apply.) a. Giving a back rub. b. Cleaning a newborn immediately after delivery. c. Emptying a portable wound drainage system. d. Interviewing a client in the emergency department. e. Obtaining the blood pressure of a client who is human immunodeficiency virus (HIV) positive

(A,B) Personal protective equipment (PPE) should be used because the newborn is covered with amniotic fluid and maternal blood. PPE should be used because the nurse may be exposed to blood and fluid that are contained in the portable wound drainage system. PPE is not required for a back rub; there is no indication that the nurse is in contact with body secretions. PPE is not necessary when conducting an interview because it is unlikely that the nurse will come in contact with the client's body fluids. PPE is not necessary when obtaining the blood pressure of a client, even if the client is HIV positive. ** A nurse is caring for a client with hemiplegia who is frustrated. How can the nurse motivate the client toward independence? a. Establish long-range goals for the client. b. Identify errors that the client can correct. c. Reinforce success in tasks accomplished. d. Demonstrate ways to promote self-reliance.

**What nursing actions best promote communication when obtaining a nursing history? (Select all that apply.) Correct1 a. Establishing eye contact b. Paraphrasing the client's message c. Asking "why" and "how" questions d. Using broad, open-ended statements e. Reassuring the client that there is no cause for alarm f. Asking questions that can be answered with a "yes" or "no"

(A,B,D) Eye contact indicates to the client that the nurse is listening and interested. Paraphrasing is an effective interviewing technique; it indicates to the client that the message was heard and invites the client to elaborate further. Open-ended statements provide a milieu in which people can verbalize their problems rather than be placed in a situation of providing a forced response. Asking "why" and "how" questions can be threatening to the client, who may not have the answer to these questions. False reassurance is detrimental to the nurse-client relationship and does not promote communication. Direct questions do not open or promote communication.

**Alternative therapy measures have become increasingly accepted within the past decade, especially in the relief of pain. Which methods qualify as alternative therapies for pain? (Select all that apply.) a. Prayerrect2 b. Hypnosis c. Medication d. Aromatherapy e. Guided imagery

(A,B,D,E) Prayer is an alternative therapy that may relax the client and provide strength, solace, or acceptance. The relief of pain through hypnosis is based on suggestion; also, it focuses attention away from the pain. Some clients learn to hypnotize themselves. Aromatherapy can help relax and distract the individual and thus increase tolerance for pain, as well as relieve pain. Guided imagery can help relax and distract the individual and thus increase tolerance for pain, as well as relieve pain. Analgesics, both opioid and non-opioid, long have been part of the standard medical regimen for pain relief, so they are not considered an alternative therapy.

**A nurse is providing dietary teaching for a client with celiac disease. Which foods should the nurse teach the client to avoid when following a gluten-free diet? (Select all that apply.) a. rye b. oats c. rice d. corn e. wheat

(A,B,E) Rye should be avoided because it is irritating to the gastrointestinal mucosa. Oats should be avoided because they are irritating to the gastrointestinal mucosa. Products containing wheat should be avoided because they are irritating to the gastrointestinal mucosa. Gluten is not found in rice; therefore, it does not have to be avoided. Gluten is not found in corn; therefore, it does not have to be avoided.

**A nurse is caring for an older adult with a hearing loss secondary to aging. What can the nurse expect to identify when assessing this client? (Select all that apply.) a. Dry cerumen b. Tears in the tympanic membrane c. Difficulty hearing high-pitched voices d. Decrease of hair in the auditory canal e. Overgrowth of the epithelial auditory lining

(A,C) Cerumen (ear wax) becomes drier and harder as a person ages. Generally, female voices have a higher pitch than male voices; older adults with presbycusis(hearing loss caused by the aging process) have more difficulty hearing higher-pitched sounds. There is no greater incidence of tympanic tears caused by the aging process. The hair in the auditory canal increases, not decreases. The epithelium of the lining of the ear becomes thinner and drier.

**A nurse teaches a client with calcium-based renal calculi about foods that can be eaten on a low-calcium diet (400 mg/day). The nurse concludes that the teaching was effective when the client selects what food items from the menu? (Select all that apply.) Correct1 a. Baked chickenorrect2 b. Chocolate pudding c. Salmon loaf with cheese saucerrect4 d. Roast beef with mashed potato e. Vanilla ice cream with chocolate syrup

(A,D) Baked chicken is relatively low in calcium. Roast beef and mashed potato have moderate amounts of calcium. Pudding is made with milk and is high in calcium. Cheese is high in calcium. Ice cream is made with milk and is high in calcium.

**Nurses are held responsible for the commission of a tort. The nurse understands that a tort is: a. The application of force to the body of another by a reasonable individual. b. An illegality committed by one person against the property or person of another.ncorrect3 c. Doing something that a reasonable person under ordinary circumstances would not do. d. An illegality committed against the public and punishable by the law through the courts.

(B ) An individual is held legally responsible for actions committed against another individual or an individual's property. The application of force to the body of another is battery, which involves physical harm. Doing something that a reasonable person under ordinary circumstances would not do is the definition of negligence. An illegality committed against the public and punishable by the law through the courts is the definition of a crime.

**When changing the soiled bed linens of a client with a wound that is draining seropurulent material, what personal protective equipment (PPE) is most essential for the nurse to wear? a. Mask b. Clean glovesrrect3 c. Sterile gloves d. Shoe covers

(B ) Clean gloves protect the hands and wrists from microorganisms in the linens. Clean gloves are the first line of defense in preventing the spread of infection. Masks, sterile gloves, and shoe covers are not required for this situation.

**A client has seeds containing radium implanted in the pharyngeal area. What should the nurse include in the client's plan of care? a. Have the client void every two hours. b. Maintain the client in an isolation room. c. Spend time with the client to allow verbalization of feelings. d. Wear two pairs of gloves when touching the client during care.

(B ) During radiation therapy with radium implants the client is placed in isolation so that exposure to radiation by family and staff is decreased. Voiding every two hours is unnecessary; a full bladder will not disrupt the seeds. Excess exposure to radiation is hazardous to personnel. Gloves will not protect the nurse from radiation.

. 52% A client suffered an injury to the leg as a result of a fall. X-ray films indicate an intertrochanteric fracture of the femur. The client will be placed in Buck's traction until surgery is performed. When considering the client's plan of care, the nurse recalls that the primary purpose of Buck's traction is to: a. Reduce the fracturect2 b. Immobilize the fracture c. Maintain abduction of the leg d. Eliminate rotation of the femur

(B) A continuous pull on the lower extremity keeps bone fragments from moving and causing further trauma, pain, and edema. The fracture will be reduced by surgery; Buck's traction is a temporary measure before surgery. Moving the leg away from the midline will not keep the leg in alignment; it is not the purpose of Buck's traction. External rotation of the femur may still occur with Buck's traction.

**A client with pulmonary tuberculosis discusses the dietary plan with the nurse. The nurse expects that the type of diet that will be prescribed for the client is: a. Liquid protein supplements b. Small, frequent, high-calorie meals c. Foods high in calories and low in protein d. Meals low in calories but high in carbohydrates

(B) Clients with tuberculosis tend to have anorexia and lose weight; small, frequent, high calorie meals encourage food intake and provide calories for weight gain. Liquid protein supplements are not necessary; protein and other nutrients can be obtained through natural foods. Low proteins are contraindicated; increased protein intake is necessary for tissue building. Meals low in calories but high in carbohydrates are impossible; carbohydrates contain calories.

**A client with a history of a pulmonary embolus is to receive 3 mg of warfarin (Coumadin) daily. The client has blood drawn twice weekly to ascertain that the international normalized ratio (INR) stays within a therapeutic range. The nurse provides dietary teaching. Which food selected by the client indicates that further teaching is necessary? a. Poached eggst2 b. Spinach salad c. Sweet potatoes d. Cheese sandwich

(B) Dark green, leafy vegetables are high in vitamin K. Influencing the level of vitamin K alters the activity of warfarin because vitamin K acts as a catalyst in the liver for the production of blood-clotting factors and prothrombin. The intake of foods containing vitamin K must be consistent to regulate the warfarin dose so that the INR remains within the therapeutic range. Eggs contain protein and are permitted on the diet. Yellow vegetables contain vitamin A and are permitted on the diet. Dairy products containing protein and bread supplying carbohydrates are permitted on the diet.

**An older client's colonoscopy reveals the presence of extensive diverticulosis. What type of diet should the nurse encourage the client to follow? a. Low-fat b. High-fiber c. High-protein d. Low-carbohydrate

(B) Fiber promotes passage of residue through the intestine, thereby preventing constipation. Constipation causes straining at stool; this increases intraluminal pressure, which can precipitate diverticulitis or perforation of diverticulum. The other diets are not indicated for diverticulosis.

**A client with respiratory difficulties asks why the percussion procedure is being performed. The nurse explains that the primary purpose of percussion is to: a. Relieve bronchial spasm. b. Increase depth of respirations. c. Loosen pulmonary secretions. d. Expel carbon dioxide from the lungs.

(B) Percussion (chest physiotherapy) loosens pulmonary secretions by mechanical means. This is accomplished by vibrations over the lung fields on the client's posterior, anterior, and lateral chest. Percussion does not relieve bronchial spasms. Once pulmonary secretions are loosened by percussion and the client has a clearer airway, the depth of respirations may increase and facilitate removal of carbon dioxide from the lungs.

**The nurse teaches a client who developed degenerative joint disease of the vertebral column positioning techniques, including turning from back to side, keeping the spine straight. The nurse explains that the least effort will be exerted if the client crosses the arm over the chest and: a. Uses the overbed table to pull the upper body up to assist with turning b. Bends the top knee to the side to which the client is turning c. Crosses the ankles while turning and keeps both legs straight d. Flexes the bottom knee to the side the client wishes to turn

(B) Putting the upper arm and leg toward the side to which the client is turning uses body weight to facilitate turning; the spine is kept straight. Using the overbed table to pull up will result in twisting the spinal column. This is unsafe; an overbed table has wheels and is not a stable object. Crossing the ankles while turning with both legs straight can be done if another person were turning the client; when turning alone in this position, the client will have no leverage and turning probably will result in twisting the spinal column. Flexing the bottom knee to the side to which the client wishes to turn will interfere with turning because the bent leg becomes an obstacle and provides a force opposite to the leverage needed to turn.

**A client who is to receive radiation therapy for cancer says to the nurse, "My family said I will get a radiation burn." What is the nurse's best response? a. "Your skin will look like a blistering sunburn." b. "A localized skin reaction usually occurs." c. "A daily application of an emollient will prevent a burn."rect4 d. "Your family must have had experience with radiation therapy."

(B) Radiodermatitis occurs three to six weeks after the start of treatment. The word "burn" should be avoided because it may increase anxiety. Emollients are contraindicated; they may alter the calculated x-ray route and injure healthy tissue. The response about the client's family does not address the client's concern.

**A nurse is teaching a postoperative client about the importance of vitamin C for wound healing. Which food selection demonstrates the client is applying the information correctly? a. bananas b. strawberries c. green beans d. sweet potatoes

(B) Strawberries contain 88 mg of vitamin C (ascorbic acid) per cup. One banana contains 12 mg of ascorbic acid. One cup of green beans contains 21 mg of ascorbic acid. One baked sweet potato contains 25 mg of ascorbic acid.

51% 10. A 1-day-old infant with an imperforate anus undergoes a pull-through procedure with an anoplasty. What should postoperative nursing care include? a. Withholding oral feedings for several daysrrect2 b. Encouraging continuation of breastfeeding c. Placing the infant in the Trendelenburg position d. Positioning the infant supine with the head of the crib elevated

(B) The goal is to prevent constipation to limit trauma to the surgical site. Breast milk produces a softer stool. Oral feedings are started soon after surgery. Placing the infant in the Trendelenburg position will not promote healing in the anal area and may impede respiratory excursion. Positioning the infant supine with the head of the crib elevated will increase pressure in the perianal area, which could compromise healing.

**A client is admitted to the hospital after a motor vehicle accident with multiple abrasions and lacerations to the chest and all four extremities. The nurse helps the client select food items for the upcoming meals and recommends: a. Meatloaf and tea b. Meatloaf and strawberries c. Chicken soup and baked apple d. Chicken soup and buttered bread

(B) The meat provides proteins and the fruit provides vitamin C; both promote wound healing. Although meatloaf provides protein, tea does not provide vitamin C. Chicken soup and a baked apple do not meet the client's need for protein or vitamin C. Chicken soup and buttered bread do not meet the client's need for protein or vitamin C.

**Why is it important for a nurse in the prenatal clinic to provide nutritional counseling to all newly pregnant women? a. Most weight gain is caused by fluid retention. b. Different cultural groups favor different essential nutrients. c. Dietary allowances should not increase throughout pregnancy. d. Pregnant women must adhere to a specific pregnancy dietary regimen.

(B) The nurse should become informed about the cultural eating patterns of clients so that foods containing the essential nutrients that are part of these dietary patterns may be included in the diet. Fluid retention is only one component of weight gain; growth of the fetus, placenta, breasts, and uterus also contributes to weight gain. The need for calories and nutrients increases during pregnancy. Pregnancy diets are not specific; they are composed of the essential nutrients.

**A nurse who promotes freedom of choice for clients in decision-making best supports which principle? a. Justiceect2 b. Autonomy c. Beneficence d. Paternalism

(B) The principle of autonomy relates to the freedom of a person to form his or her own judgments and actions. The nurse promotes autonomy nonjudgmentally so as not to infringe on the decisions or actions of others. Justice means to be righteous, equitable, and to act or treat fairly. Beneficence relates to the state or act of doing good, being kind and charitable. It also includes promotion of well-being and abstaining from the injuring of others. Paternalism encompasses the practice of governing people in a fatherly manner, especially by providing for their needs without infringing on their rights or responsibilities.

**A severely dehydrated infant with gastroenteritis is admitted to the pediatric unit. Nothing-by-mouth (NPO) status is prescribed. The parents ask why their baby cannot be fed. The nurse explains that this is necessary to: a. Correct electrolyte imbalancesct2 b. Allow the intestinal tract to rest c. Determine the cause of the diarrhea d. Prevent perianal irritation from the diarrhea

(B) Withholding food reduces the need for intestinal activity, which rests the intestines and minimizes diarrhea and the loss of fluid. Although intravenous therapy will be started for rehydration and to correct electrolyte imbalances, this is not the reason for the NPO status. Stool cultures are used to determine the cause of the diarrhea. Perianal irritation is prevented with meticulous skin care, not by withholding food and fluids.

**After surgery for a fractured hip, a client states, "I don't remember when I have ever been so uncomfortable." The nurse should: a. notify the healthcare provider b. use distraction techniques c. medicate the client as prescribed d. perform a complete pain assessment

(D) A complete assessment must be performed to determine the location, characteristics, intensity, and duration of the pain. The pain may be incisional, result from a pulmonary embolus , or be caused by neurovascular trauma to the affected leg, and the intervention for each is different. Notifying the health care provider, using distraction techniques, and medicating the client as prescribed may be done after a complete assessment reveals that this is the appropriate intervention; assessment is the priority.

A client's infusion of normal saline infiltrated earlier today, and approximately 500 ml of saline infused into the subcutaneous tissue. The client is now complaining of excruciating arm pain and demanding "stronger pain medications." What initial action is most important for the nurse to take? A. Ask about any past history of drug abuse or addiction. B. Measure the pulse volume and capillary refill distal to the infiltration. C. Compress the infiltrated tissue to measure the degree of edema. D. Evaluate the extent of ecchymosis over the forearm area. Pain and diminished pulse volume

(B) are signs of compartment syndrome, which can progress to complete loss of the peripheral pulse in the extremity. Compartment syndrome occurs when external pressure (usually from a cast), or internal pressure (usually from subcutaneous infused fluid), exceeds capillary perfusion pressure resulting in decreased blood flow to the extremity. (A) should not be pursued until physical causes of the pain are ruled out. (C) is of less priority than determining the effects of the edema on circulation and nerve function. Further assessment of the client's ecchymosis can be delayed until the signs of edema and compression that suggest compartment syndrome have been examined (D). Correct Answer: B

8. The nurse observes an unlicensed assistive personnel (UAP) who is providing a total bed bath for a confused and lethargic client. The UAP is soaking the client's foot in a basin of warm water placed on the bed. What action should the nurse take? a. Remove the basin of water from the client's bed immediately b. Remind the UAP to dry between the client's toes completely c. Advise the UAP that this procedure is damaging to the skin d. Add skin cream to the basin of water while the foot is soaking

(B) is especially important in making sure the patient does not experience skin breakdown due to excessive moisture. Keeping the client's feet clean is necessary, but keeping the client's feet dry is extremely important in skin maintenance.

The healthcare provider prescribes furosemide (Lasix) 15 mg IV stat. On hand is Lasix 20 mg/2 ml. How many milliliters should the nurse administer? A. 1 ml. B. 1.5 ml. C. 1.75 ml. D. 2 ml.

(B) is the correct calculation: Dosage on hand/amount on hand = Dosage desired/x amount. 20 mg : 2 ml = 15 mg : x . 20x = 30. x = 30/20; = 1½ or 1.5 ml. Correct Answer: B

A client is to receive 10 mEq of KCl diluted in 250 ml of normal saline over 4 hours. At what rate should the nurse set the client's intravenous infusion pump? A. 13 ml/hour. B. 63 ml/hour. C. 80 ml/hour. D. 125 ml/hour.

(B) is the correct calculation: To calculate this problem correctly, remember that the dose of KCl is not used in the calculation. 250 ml/4 hours = 63 ml/hour. Correct Answer: B

**An 85-year-old client has just been admitted to a nursing home. When designing a plan of care for this older adult the nurse recalls the expected sensory losses associated with aging. (Select all that apply.) a. Difficulty in swallowing b. Diminished sensation of pain c. Heightened response to stimuli d. Impaired hearing of high-frequency sounds e. Increased ability to tolerate environmental heat

(B,D) Because of aging of the nervous system an older adult has a diminished sensation of pain and may be unaware of a serious illness, thermal extremes, or excessive pressure. As people age they experience atrophy of the organ of Corti and cochlear neurons, loss of the sensory hair cells, and degeneration of the stria vascularis, which affect an older person's ability to perceive high-frequency sounds. An interference with swallowing is a motor, not a sensory, loss, nor is it an expected response to aging. There is a decreased, not heightened, response to stimuli in older adults. There is a decreased, not increased, ability to physiologically adjust to extremes in environmental temperature.

**A nurse fails to act in a reasonable, prudent manner. Which legal principle is most likely to be applied? Incorrect1 a. Malice b. Tort law c. Malpractice d. Case law

(C ) Malpractice is the unskilled or faulty treatment by a professional that causes injury or harm to a client. It can result from a lack of professional knowledge or skill that can be expected in others in the profession, or from a failure to exercise reasonable care or judgment in the applying of professional knowledge, experience, or skill. Malice is the desire or intent to inflict injury, harm, or suffering. A tort is a wrongful act, not including a breach of contract or trust, that results in injury to another person and for which the injured person is entitled to compensation. Case law is law established by judicial decisions in particular cases instead of by legislation action.

**A client is receiving a 2-gram sodium diet. The family asks whether they can bring snacks from home. The nurse suggests that they bring foods low in sodium such as: a. ice cream b. celery sticks c. fresh orange wedges d. peanut butter cookies

(C) An orange contains only trace amounts of sodium. One cup of ice cream contains approximately 115 mg of sodium. One cup of celery contains approximately 106 mg of sodium. Four peanut butter cookies contain 142 mg of sodium.

**A nurse is teaching staff members about the legal terminology used in child abuse.What definition of battery should the nurse include in the teaching? a. Maligning a person's character while threatening to do bodily harm. b. A legal wrong committed by one person against property of another. c. The application of force to another person without lawful justification. d. Behaving in a way that a reasonable person with the same education would not.

(C) Battery means touching in an offensive manner or actually injuring another person. Battery refers to actual bodily harm rather than threats of physical or psychological harm. Battery refers to harm against persons instead of property. Behaving in a way that a reasonable person with the same education would not is the definition of negligence.

**A nurse provides dietary teaching about a low-sodium diet for a client with hypertension. Which nutrient selected by the client indicates an understanding about foods that are low in natural sodium? a. Milk b. Meatrrect3 c. Fruits d. Vegetables

(C) Fruits contain less natural sodium than do other foods. Milk is higher in natural sodium than is fruit. Meat is higher in natural sodium than is fruit. Vegetables are higher in natural sodium than is fruit.

**A nurse is monitoring a client with renal failure for signs of fluid excess. Which finding does the nurse identify as inconsistent with fluid excess? a. Increased weight b. Distended neck veins c. Orthostatic hypotension d. Abnormal breath sounds

(C) Hypertension, not hypotension, is an indicator of fluid volume excess. Fluid excess causes weight gain; one liter weighs 2.2 lb. Fluid excess increases the intravascular volume, leading to jugular vein distention. Fluid excess causes fluid in the alveoli that leads to crackles, a sign of pulmonary edema.

**Before discharging a 9-year-old child who is being treated for acute poststreptococcal glomerulonephritis (APSGN), what information should the nurse plan to give the parents? a. How to obtain the vital signs daily b. Date on which to return to prepare for renal dialysis c. Instructions about which high-sodium foods to avoid d. List of activities that will encourage the child to remain active

(C) Sodium is usually limited to control or prevent edema or hypertension until the child is asymptomatic. The child is usually on a regular diet with sodium restrictions (e.g., salty snacks [potato chips, pretzels, tortilla chips] and hot dogs, bacon, bologna, and other processed meats). It is not necessary to check the vital signs daily, but the health care provider may suggest weighing the child daily. Usually recovery from APSGN is complete. The condition does not cause such severe kidney damage that dialysis is necessary. The child should not be kept active, because rest is needed until the child is asymptomatic.

**A nurse identifies that an older adult has not achieved the desired outcome from a prescribed proprietary medication. When assessing the situation, the client shares that the medication is too expensive and the prescription was never filled. What is an appropriate nursing response? a. Ask the pharmacist to provide a generic form of the medication. b. Encourage the client to acquire the medication over the internet. c. Inform the health care provider of the inability to afford the medication. d. Suggest that the client purchase insurance that covers prescription medications.

(C) The health care provider needs to be aware of the reason for the client's lack of response to the medication so that an alternate treatment plan or financial assistance can be arranged (e.g., go to The National Council on the Aging web site [BenefitsCheckUpRx] to establish whether the client is eligible for assistance from any community, state, or federal programs or from the drug company). A health care provider may prefer the proprietary form of the medication. To ask the pharmacist to provide a generic form of the medication is unsafe. To recommend that the client obtain a generic form of the medication is not within the legal role of the nurse, unless the health care provider documents that this is acceptable. Medications purchased over the internet may be illegally imported, counterfeit, expired, or contaminated and therefore should be avoided. Although some prescription insurance plans may help to reduce the cost of some medications, the client may not be able to afford the insurance.

**A client's serum albumin value is 2.8 g/dL. Which food selected by the client indicates that the nurse's dietary teaching is successful? a. beef broth b. fruit salad c. sliced turkey d. spinach salad

(C) This client's serum albumin value indicates severe depletion of visceral protein stores; the expected range for serum albumin is 3.5 to 5.5 g/dL. White meat turkey (two slices 4 × 2 × 1/4 inch) contains approximately 28 g of protein. A 4 oz serving of beef broth contains approximately 2.4 g of protein. A 6 oz serving of mixed fruit contains approximately 0.5 g of protein. A 3 oz serving of spinach salad contains approximately 9 g of protein.

**To prevent footdrop in a client with a leg cast, the nurse should: a. Encourage complete bed rest to promote healing of the foot. b. Place the foot in traction. c. Support the foot with 90 degrees of flexion. d. Place an elastic stocking on the foot to provide support.

(C) To prevent footdrop (plantar flexion of the foot due to weakness or paralysis of the anterior muscles of the lower leg) in a client with a cast, the foot should be supported with 90 degrees of flexion. Bed rest can cause footdrop, and 45 degrees is not enough flexion to prevent footdrop . Applying an elastic stocking for support also will not prevent footdrop; a firmer support is required.

At the time of the first dressing change, the client refuses to look at her mastectomy incision. The nurse tells the client that the incision is healing well, but the client refuses to talk about it. What would be an appropriate response to this client's silence? A. It is normal to feel angry and depressed, but the sooner you deal with this surgery, the better you will feel. B. Looking at your incision can be frightening, but facing this fear is a necessary part of your recovery. C. It is OK if you don't want to talk about your surgery. I will be available when you are ready. D. I will ask a woman who has had a mastectomy to come by and share her experiences with you.

(C) displays sensitivity and understanding without judging the client. (A) is judgmental in that it is telling the client how she feels and is also insensitive. (B) would give the client a chance to talk, but is also demanding and demeaning. (D) displays a positive action, but, because the nurse's personal support is not offered, this response could be interpreted as dismissing the client and avoiding the problem. Correct Answer: C

The healthcare provider prescribes the diuretic metolazone (Zaroxolyn) 7.5 mg PO. Zaroxolyn is available in 5 mg tablets. How much should the nurse plan to administer? A. ½ tablet. B. 1 tablet. C. 1½ tablets. D. 2 tablets.

(C) is the correct calculation: D/H × Q = 7.5/5 × 1 tablet = 1½ tablets. Correct Answer: C

1. A policy requiring the removal of acrylic nails by all nursing personnel was implemented 6 months ago. Which assessment measure best determines if the intended outcome of the policy is being achieved? a. Number of staff induced injury b. Client satisfaction survey c. Health care-associated infection rate. d. Rate of needle-stick injuries by nurse.

(C)--Acrylic nails are known to carry loads of bacteria and increase the risk of healthcare-associated infections. Therefore, by banning the wearing of acrylic nails, you would expect the prevalence of healthcare-associated infections to decrease. Acrylic nails have nothing to do with staff induced injuries, needle-stick injuries, or patient satisfaction scores.

**A 2-year-old child admitted with a diagnosis of pneumonia was administered antibiotics, fluids, and oxygen. The child's temperature increased until it reached 103° F. When notified, the health care provider determined that there was no need to change treatment, even though the child had a history of febrile seizures. Although concerned, the nurse took no further action. Later, the child had a seizure that resulted in neurological impairment. Legally, who is responsible for the child's injury? a. Health care provider, because this decision took precedence over the nurse's concern b. Health care provider, because of total responsibility for the child's health and treatmentregimen c. Nurse, because failure to further question the health care provider about the child's statusplaced the child at risk d. Neither, because high fevers are common in children and the health care provider had littlecause for concern

(C)It is the nurse's responsibility to foresee potential harm and prevent risks by acting as a client advocate. This is not acceptable as a rationale for inaction. The nurse and health care provider share interdependent roles in the assessment and care of clients. High temperatures are common in children but are nonetheless a valid cause for concern.

**The most effective time to teach clients who have sustained a sudden, traumatic, major loss is most often during the acceptance or adaptation stage of coping. The rationale for this fact is that clients in this stage are: a. Ready for discharge and therefore in need of preparation b. At the peak of mental anguish and therefore open to change c. Less angry and therefore more compliant and more receptive d. Less anxious and more aware of reality and therefore ready to learn

(D) Anxiety or anger associated with other stages of coping interfere with learning. This is too late to start preparation for discharge and teaching. Many factors influence readiness for learning; planning for teaching must begin on the day of admission. The anxiety associated with mental anguish will interfere with the ability to process new information; mental anguish is associated with an earlier stage. Although clients in the acceptance or adaptation phase are less angry, the reason teaching is most effective is not because of their compliance but because new information can be processed more easily.

A client experiences occasional right upper quadrant pain attributed to dyspepsia. The nurse is providing discharge instructions, including a list of foods that cause dyspepsia. The list should include: a. Nuts and popcorn b. Meatloaf and baked potato c. Chocolate and boiled shrimprrect4 d. Fried chicken and buttered corn

(D) Cholecystitis is often accompanied by intolerance to fatty foods, including fried foods and butter. Because nuts and popcorn have a high fiber content, they cause flatulence and pain for clients with lower intestinal problems, such as diverticulosis. Meatloaf and baked potato contain less fat than do fried foods or butter. Neither chocolate nor boiled seafood contains as much fat as fried chicken or butter.

**A mother whose newborn infant son has a cleft lip and palate asks how to feed her baby because he has difficulty suckling. What information should the nurse provide concerning safe feeding technique for this infant? a. "Because he tires easily, it's best to have him lying in bed while he is being fed." b. "Hold him in a horizontal position and feed him slowly to help prevent aspiration." c. "Try using a soft nipple with an enlarged opening so he can get the milk through a chewing motion." d. "Give him brief rest periods and frequent burpings during feedings so he can get rid of swallowed air."

(D) Cleft lip and palate, a congenital defect, prevents the infant from creating a tight seal with the lips to facilitate suckling. As a result, the infant swallows large amounts of air when feeding. The mother should be taught to provide frequent rest periods and to burp the infant often to expel excess air in the stomach. Infants with cleft lip and palate should be held upright during feedings. Newborn infants cannot chew and do not make chewing movements.

**A nurse is caring for a client that has been admitted with right sided heart failure. The nurse notes that the client has dependent edema around the area of the feet and ankles. In order to characterize the severity of the edema, the nurse presses the medial malleolus area and notes an 8 mm depression after release. This nurse understands that the edema should be documented as: a. 1+ b. 2+ c. 3+ d. 4+

(D) Dependent edema around the area of feet and ankles often indicates right sided heart failure or venous insufficiency. The nurse should assess for pitting edema by pressing firmly for several seconds then release to assess for any depression left on the skin. The grading of 1+ to 4+ characterizes the severity of the edema. A grade of grade of 4+ indicates an 8mm depression. A grade of 1+ indicates a 2 mm depression. A grade of 2 + indicates a 4mm depression. A grade of 3+ indicates a 6 mm depression.

**A client is hospitalized for the treatment of thrombophlebitis. What should the nurse include in the client's teaching plan related to how to prevent thrombophlebitis? a. Perform leg exercises b. Sit with the knees flexed c. Apply warm soaks to the legs daily d. Put on elastic stockings before arising

(D) Donning elastic stockings before getting out of bed provides support and promotes venous return; applying stockings while the legs are horizontal ensures that the stockings are in place before dependent edema occurs. Although leg exercises are helpful, this will not provide continuous support for the veins. Sitting with the knees flexed promotes venous stasis and the formation of thrombophlebitis. Warm soaks resolve inflammation; they do not prevent the development of thrombophlebitis.

**What should the nurse include in dietary teaching for a client with a colostomy? a. Liquids should be limited to 1 L per day. b. Non-digestible fiber and fruits should be eliminated. c. A formed stool is an indicator of constipation. d. The diet should be adjusted to include foods that result in manageable stools.

(D) Each person will need to experiment with diet after a colostomy to determine what foods are best tolerated and also produce stools that are manageable, depending on the type of colostomy. Liquids are typically not limited unless there is a specific reason such as cardiac or renal disease. Food high in fiber such as fruit should be included in the diet as tolerated. Depending on the type of colostomy and the diet, a formed stool is acceptable and does not indicate a constipating diet.

**The nurse is teaching a client about adequate hand hygiene. What component of hand washing should the nurse include that is most important for removing microorganisms? a. soap b. time c. water d. friction

(D) Friction is necessary for the removal of microorganisms. Although soap reduces surface tension, which helps remove debris, without friction it has minimal value. Although the length of time the hands are washed is important, without friction it has minimal value. Although water flushes some microorganisms from the skin, without friction it has minimal value.

**A client with a history of chronic obstructive pulmonary disease (COPD) is admitted with acute bronchopneumonia. The client is in moderate respiratory distress. The nurse should place the client in what position to enhance comfort? a. Side-lying position with head elevated 45 degrees b. Sim's position with head elevated 90 degrees c. Semi-Fowler's position with legs elevatedt4 d. High-Fowler's position using the bedside table as an arm rest

(D) High Fowler's position elevates the clavicles and helps the lungs to expand, thus easing respirations. The other options do not promote more comfortable breathing.

52. During the admission assessment of a terminally ill male client that he is an agnostic. What is the best nursing action in response to this statement? a. Provide information about the hours and location of the chapel b. Document the statement of the client's spiritual assessment c. Invite the client to a healing service for people of all religions d. Offer to contact a spiritual advisor of the client's choice

(b)--You should always respect a patient's religious or lack of, values and document it. (A) is inappropriate because the client is Agnostic. (C) is also inappropriate. (D) also inappropriate.

**Which action by a home care nurse would be considered an act of euthanasia? a. Implementing a "do not resuscitate" order in the home health setting. b. Abiding by the decision of a living will signed by the client's family. c. Encouraging a client to consult an attorney to document and assign a power of attorney. d. Knowing that a dying client is overmedicating and not acting on this information.

(D) In this situation being aware that a client is overmedicating and taking no action can be considered an act of euthanasia on the part of the home care nurse. Implementing a "do not resuscitate" order, abiding by the decision of a living will signed by the client's family, and encouraging the client to consult an attorney are all appropriate actions for a home care nurse.

**A nurse is discussing weight loss with an obese individual with Ménière's disease. Which suggestion by the nurse is most important? a. Limit intake to 900 calories a day.ct2 b. Enroll in an exercise class. c. Get involved in diversionary activities when there is an urge to eat. d. Keep a diary of all foods eaten each day.

(D) Keeping a record of what one eats helps to limit unconscious and nervous eating by making the individual aware of intake. Limiting calories to 900 per day is a severe restriction that requires a health care provider's prescription. Exercise causes rapid head movements, which may precipitate a Ménière's attack. Although diversionary activities are a therapeutic intervention, the nurse first should make suggestions that help increase the client's awareness of personal eating habits.

**A nurse reviews the prescribed treatment with the parents of an infant born with bilateral clubfeet. Which parental statement indicates to the nurse that further education is required? a. We'll have to start serial casting right away." b. "The casts will have to be changed every week." c. "The baby may have to have surgery if the problem's not fixed in a few months." d. "We'll have to have the baby fitted with prosthetic devices before he'll be able to walk."

(D) Most children with bilateral clubfeet are eventually able to walk without much difficulty. Prosthetic devices generally are not indicated. Serial casting with cast changes every week is usually successful. If serial casting is not effective, surgical intervention may be necessary.

**A nurse is caring for a client who is experiencing urinary incontinence. The client has an involuntary loss of small amounts (25 to 35 mL) of urine from an overdistended bladder. This should be documented in the medical record as: a. Urge incontinence b. Stress incontinence c. Reflex incontinence d. Overflow incontinence

(D) Overflow incontinence describes what is happening with this client; overflow incontinence occurs with retention of urine with overflow of urine. Urge incontinence describes a strong need to void that leads to involuntary urination. Stress incontinence occurs when a small amount of urine is expelled because of an increase in intraabdominal pressure that occurs with coughing, lifting, or sneezing. Reflex incontinence is an involuntary loss of urine at fairly predictable intervals when certain urinary bladder intervals are reached.

**A client was recently diagnosed with a cancerous lesion of the mouth. What should the nurse ask when analyzing the client's need for health education in relation to this health problem? a. "Are you having difficulty sleeping?" b. "Do feel like your gums are inflamed?" c. "How frequently are you seeing the dentist?" d. "Have you noticed any change in your appetite?"

(D) Problems involving the oral cavity often result in nutritional problems. The question "Have you noticed any change in your appetite?" will elicit more information. An inability to sleep usually is not a characteristic symptom of cancer of the oral cavity; it may occur after the diagnosis because of worry or fear. Gum infections usually are not an early problem after diagnosis of oral cancer. Lesions of the oral cavity do not tend to cause major dental problems.

58% A nurse is caring for an elderly client who has constipation. Which independent nursing intervention helps to reestablish normal bowel pattern? a. Administer a mineral oil enema. b. Offer one cup of fluid every hour. c. Manually remove fecal impactions. d. Offer a cup of prune juice.

(D) Prune juice does not require a health practitioner order and helps to promote bowel movement because it contains sorbitol, which increases water retention in feces. Administration of mineral enema requires an order from a health care provider. Encouraging the client's fluid intake by offering one cup of fluid every hour is helpful in preventing constipation but not as effective in resolving constipation as a prune juice. Removing impactions does not establish regular bowel patterns.

**The nurse is providing care for a client that is on bed rest. The nurse can prevent skin breakdown for this client by: a. Massaging the bony prominences b. Maintaining a sheepskin pad under the client c. Promoting range-of-motion activities d. Encouraging the client to move around as much as possible

(D) The client who is confined to bed should be encouraged to move in bed to prevent prolonged pressure on any one skin surface. Massaging bony prominences increase the risk of skin breakdown. Although sheepskin material allows air to circulate under the client, it does not prevent prolonged pressure. Range-of-motion exercises move joints to prevent contractures; they do not relieve prolonged pressure.

**When assisting a client who had a total hip replacement onto the bedpan on the first postoperative day, the nurse should instruct the client to: a. Turn toward the operative side b. Flex both knees while slowly lifting the pelvis c. Extend both legs and pull on the trapeze to lift the pelvis d. Flex the unaffected knee and pull on the trapeze to raise the pelvis

(D) The pelvis is elevated by actions involving the unaffected upper extremities and unaffected leg. Turning toward the operative side is not permitted because it causes adduction of the leg and can lead to dislocation of the femoral head. Flexing both knees while slowly lifting the pelvis puts pressure on the operative hip, which is contraindicated because it may dislocate the prosthesis. Lifting only with the arms requires strength; the use of both heels puts pressure on the operative hip, which may dislocate the prosthesis.

**A nurse is teaching a class about hepatitis, specifically hepatitis A. Which food should the nurse explain most likely will remain contaminated with the hepatitis A virus after being cooked? a. Canned tuna b. Broiled shrimp c. Baked haddock d. Steamed lobster

(D) The temperature during steaming is never high enough or sustained long enough to kill microorganisms. Processing destroys the virus. Because of the extremely high temperature, broiling sufficiently destroys the virus. Baking will destroy the virus.

**On the third postoperative day following a below-the-knee amputation, a client is refusing to eat, talk, or perform any rehabilitative activities. What is the best initial approach that the nurse should take when interacting with this client? a. Explain why there is a need to increase activity b. Emphasize that with a prosthesis, there will be a return to the previous lifestyle c. Appear cheerful and non-critical regardless of the client's response to attempts at intervention. d. Acknowledge that the client's withdrawal is an expected and necessary part of initial grieving

(D) The withdrawal provides time for the client to assimilate what has occurred and integrate the change in body image. The client is not ready to hear explanations about why there is a need to increase activity until assimilation of the surgery has occurred. Emphasizing a return to the previous lifestyle does not acknowledge that the client must grieve; it also does not allow the client to express any feelings that life will never be the same again. In addition, it may be false reassurance. The client might feel that the nurse has no comprehension of the situation or understanding of feelings if the nurse appears cheerful and noncritical regardless of the client's response to attempts at intervention.

**A client who has intermittently been having painful, swollen knee and wrist joints during the past three months is diagnosed with rheumatoid arthritis. What type of diet should the nurse expect the health care provider to prescribe? a. Salt-free, low-fiber diet b. High-calorie, low-cholesterol diet c. High protein diet with minimal calcium d.Regular diet with vitamins and minerals

(D) There are no dietary restrictions, but iron and vitamins should be encouraged to treat any underlying nutritional deficiencies. A salt-free, low-fiber diet is not indicated. A high-calorie diet will increase the client's weight; this is contraindicated because it will increase the strain on weight-bearing joints. A balanced diet should fulfill nutritional needs; there is no need to increase protein or restrict calcium.

Twenty minutes after beginning a heat application, the client states that the heating pad no longer feels warm enough. What is the best response by the nurse? A. That means you have derived the maximum benefit, and the heat can be removed. B. Your blood vessels are becoming dilated and removing the heat from the site. C. We will increase the temperature 5 degrees when the pad no longer feels warm. D. The body's receptors adapt over time as they are exposed to heat.

(D) describes thermal adaptation, which occurs 20 to 30 minutes after heat application. (A and B) provide false information. (C) is not based on a knowledge of physiology and is an unsafe action that may harm the client. Correct Answer: D

An IV infusion terbutaline sulfate 5 mg in 500 ml of D5W, is infusing at a rate of 30 mcg/min prescribed for a client in premature labor. How many ml/hr should the nurse set the infusion pump? A. 30 B. 60 C. 120 D. 180

(D) is correct calculation: 180 ml/hr = 500 ml/5 mg × 1mg/1000 mcg × 30 mcg/min × 60 min/hr. Correct Answer: D

The nurse mixes 50 mg of Nipride in 250 ml of D5W and plans to administer the solution at a rate of 5 mcg/kg/min to a client weighing 182 pounds. Using a drip factor of 60 gtt/ml, how many drops per minute should the client receive? A. 31 gtt/min. B. 62 gtt/min. C. 93 gtt/min. D. 124 gtt/min.

(D) is the correct calculation: Convert lbs to kg: 182/2.2 = 82.73 kg. Determine the dosage for this client: 5 mcg × 82.73 = 413.65 mcg/min. Determine how many mcg are contained in 1 ml: 250/50,000 mcg = 200 mcg per ml. The client is to receive 413.65 mcg/min, and there are 200 mcg/ml; so the client is to receive 2.07ml per minute. With a drip factor of 60 gtt/ml, then 60 × 2.07 = 124.28 gtt/min (D) OR, using dimensional analysis: gtt/min = 60 gtt/ml X 250 ml/50 mg X 1 mg/1,000 mcg X 5 mcg/kg/min X 1 kg/2.2 lbs X 182 lbs. Correct Answer: D

The UAPs working on a chronic neuro unit ask the nurse to help them determine the safest way to transfer an elderly client with left-sided weakness from the bed to the chair. What method describes the correct transfer procedure for this client? A. Place the chair at a right angle to the bed on the client's left side before moving. B. Assist the client to a standing position, then place the right hand on the armrest. C. Have the client place the left foot next to the chair and pivot to the left before sitting. D. Move the chair parallel to the right side of the bed, and stand the client on the right foot.

(D) uses the client's stronger side, the right side, for weight-bearing during the transfer, and is the safest approach to take. (A, B, and C) are unsafe methods of transfer and include the use of poor body mechanics by the caregiver. Correct Answer: D

**While assessing an immobilized client, the nurse notes that the client has shortened muscles over a joint, preventing full extension. This condition is known as: a. Osteoarthritis b. Osteoporosis c. Muscle atrophy d. Contracture

(D)Immobilized clients are at high risk for the development of contractures. Contractures are characterized by permanent shortening of the muscle covering a joint. Osteoarthritis is a disease process of the weight-bearing joints due to wear and tear. Osteoporosis is a metabolic disease process in which the bones lose calcium. Muscle atrophy is a wasting and/or decrease in the strength and size of muscles due to a lack of physical activity or a neurological or musculoskeletal disorder.

**A client with a left ureteral calculus is scheduled for a transurethral ureterolithotomy. During the preoperative assessment, the nurse expects the client to report pain: a. That is a boring-type pain that is located in the left flank b. That occurs with each urination and is located at the level of the kidneys c. That is dull and constant and located in the costovertebral angle d. That is spasmodic and located in the left side and radiating to the suprapubic area

(D)Pain with ureteral stones is caused by spasm and is excruciating and intermittent; it follows the path of the ureter to the bladder. Pain is spasmodic and excruciating, not boring. Pain intensifies as the calculus lodges in the ureter and spasms occur in an attempt to dislodge it. Spasmodic pain on the left side that radiates to the suprapubis is typical of pain caused by a stone in the renal pelvis.

**Which nursing activities are examples of primary prevention? Select all that apply. a. Preventing disabilities b. Correcting dietary deficiencies c. Establishing goals for rehabilitation d. Assisting with immunization programs e. Facilitating a program about smoking cessation

(D,E) Immunization programs prevent the occurrence of disease and are considered primary interventions. Stopping smoking prevents the occurrence of disease and is considered a primary intervention. Preventing disabilities is a tertiary intervention. Correcting dietary deficiencies is a secondary intervention. Establishing goals for rehabilitation is a tertiary intervention.

**An older adult tells the nurse, "I read about a vitamin that may be related to aging because of its effect on the structure of cell walls. I wonder whether it is wise to take it." The nurse concludes the client probably is referring to: a. Vitamin A b. Vitamin B1 c. Vitamin Correct4 d. Vitamin E

(E ) Vitamin E hinders the oxidative breakdown of structural lipid membranes in body tissues, which is caused by free radicals in the cells. Vitamin A assists in the formation of visual purple needed for night vision. Vitamin C is used for formation of collagen, which is important for maintaining capillary strength, promoting wound healing, and resisting infection. Vitamin B1 is necessary for protein and fat metabolism and for functioning of the nervous system.

40. While suctioning a client's nasopharynx the nurse observes that the client's oxygen saturation remains at 94% which is the same reading obtained prior to starting the procedure. What action should the nurse take in response to this finding? a. Complete the intermittent suction of the nasopharynx. b. Reposition the pulse oximeter clip to obtain a new reading. c. Stop suctioning until the pulse oximeter reading is above 95%. d. Apply an oxygen mask over the client's nose and mouth

(a)--94% is perfect for suctioning! Only if you see the patient's O2 Stat drop below 90% is when you stop the suctioning and hyper-oxygenate the patient for a couple minutes. (B) not helpful. (C) it is ok to keep suctioning until you see a dip below 90% for O2. (D) not necessary at 94%.

10. At 0100 on a male client's second postoperative night, the client states he is unable to sleep and plans to read until feeling sleepy. What action should the nurse implement? a. Leave the room and close the door to the client's room b. Assess the appearance of the client's surgical dressing c. Bring the client a prescribed PRN sedative-hypnotic d. Discuss symptoms of sleep deprivation with the client

(a)--Although the patient has stated he is unable to sleep, the patient has also stated he has a plan, "to read until feeling sleep", which implies the patient plans to sleep. Therefore, (D) is not necessary and (C ) is very unnecessary because it is a stronger sleep aid. Offering melatonin would be more appropriate, but since it is not an option, (A) is correct. (B) does not help the client sleep in any way.

21. To prepare a client for the potential side effects of a newly prescribed medication, what action should the nurse implement? a. Assess the client for health alterations that may be impacted by the effects of the medication b. Teach the client how to administer the medication to promote the best absorption c. Administer a half dose and observe the client for side effects before administering a full dosage d. Encourage the client to drink plenty of fluids to promote effective drug distribution

(a)--Before a new medication is given, an initial assessment should be completed to create a baseline for the patient; then the RN will be able to re-evaluate the patient and see if there have been any health alterations caused by the new medication. (B) this has nothing to do with potential side effects. (C) You should always administer a new medication as prescribed by the MD. (D) The amount of fluids the patient drinks will not affect the drug distribution in the body.

35. When evaluating the effectiveness of a client's nursing care, the nurse first reviews the expected outcomes identified in the plan of care. What action should the nurse take next? a. Determine if the expected outcomes were realistic b. Obtain current client data to compare with expected outcomes c. Modify the nursing interventions to achieve the client's goals d. Review related professional standards of care

(a)--Before you can determine if care is effective you need to be sure the original expected outcomes were realistic. (B) This step comes after (A). (C) Comes after evaluating if the goals are effective and obtaining data. (D) Not necessary in this case.

22. A client is 2 days post-op from a thoracic surgery and is complaining of incisional pain. The client last received pain medication 2 hours ago. He is rating his pain a 5 on a 1-10 scale. After calling the provider, what is the nurse's next action? a. Instruct the client to use guided imagery and slow rhythmic breathing b. Provide at least 20 minutes of back massage and gentle effleurage c. Encourage the client to watch TV. d. Place a hot water circulation device, such as an Aqua K pad, to operative site

(a)--If there are no other PRN pain medications available after an initial dose was given, it is most appropriate to call the provider, then switch to alternative pain management methods; like guided imagery and encouraging slow rhythmic breathing. (B) while massage may be helpful, it is inappropriate for incisional pain as it may open the sutures. (C) While distraction can help reduce pain, watching TV does not rid the patient of the pain. (D) NEVER place a circulation device on an operative site as it may open the sutures!

44. The nurse observes a UAP positioning a newly admitted client who has a seizure disorder. The client is supine and the UAP is placing soft pillows along the side rails. Which action should the nurse implement? a. Instruct the UAP to obtain soft blankets to secure to the side rails instead of pillows b. Ensure that the UAP has placed pillows effectively to protect the client c. Ask the UAP to use some pillows to prop the client in a side-lying position d. Assume responsibility for placing the pillows while the UAP complete another task

(a)--In an ideal world, you would have seizure guard pads, but in the HESI world, you do not have it. SO! Pillows guarding the rails is dangerous and could suffocate your patient if they seizure and it falls and covers their face. Blankets secured to the side rails can be tied down to ensure they do not fall on the patient's face.

9. The nurse in the emergency department observes a colleague viewing the electronic health record (EHR) of a client who holds an elected position in the community. The client is not a part of the colleague's assignment. Which action should the nurse implement? a. Communicate the colleague's actions to the unit charge nurse b. Send an email to facility administration reporting the action c. Write an anonymous complaint to a professional website d. Post a comment about the action on a staff discussion board

(a)--Looking up patients who are not under your direct care is a HIPPA violation and may result in termination of employment, despite the patient's status in society or your curiosity. The first action to implement is to report to your Charge Nurse so he or she may report the incident to the appropriate chain of command.

11. The nursing staff in the cardiovascular intensive care unit are creating a continuous quality improvement project on social media that addresses coronary artery disease (CAD). Which action should the nurse implement to protect client privacy? a. Remove identifying information of the clients who participated b. Recall that authored content may be legally discoverable c. Share material from credible, peer reviewed sources only d. Respect all copyright laws when adding website content

(a)--Since the improvement project is being creating on a social media platform, it is imperative to have all names and patient identifiers removed to protect the client's identity and privacy. Any names posted, regardless of whether or not it is a social media platform or a peer-reviewed source is a HIPPA violation.

29. When performing blood pressure measurement to assess for orthostatic hypotension, which action should the nurse implement first? a. Position the client supine for a few minutes b. Assist the client to stand at the bedside c. Apply the blood pressure cuff securely d. Record the client's pulse rate and rhythm

(a)--The first step to measure orthostatic hypotension is to lie the patient down in a supine position for 3-5 minutes, and measure their blood pressure before having the patient sit up. (B) comes after a measurement is made while sitting. (C) should be done anyways. (D) This step should be done after each position change.

50. The nurse receives a report that a client with an indwelling urinary catheter has an output of 150 mL, for the previous 6 hour shift. Which intervention should the nurse implement first? a. Check the drainage tubing for a kink b. Review the intake and output record. c. Notify the healthcare provider d. Give the client 8 oz of water to drink

(a)--The minimum amount of output a patient should have for one hour is 30mL. In 6 hours, it should at least be 180mL. Since it is under, the first thing to do is check the equipment, in this case the foley tubing! Kinks can prevent adequate output to be in the bag. (B) doesn't help the situation. (C) not necessary unless the tubing is not kinked, the patient has sufficient PO intake, and there are no issues with the foley. (D) giving water would be the last step if the patient is not fluid overloaded.

14. A postoperative client has three different PRN analgesics prescribed for different levels of pain. The nurse inadvertently administers a dose that is not within the prescribed parameters. What actions should the nurse take first? a. Access for side effects of the medication. b. Document the client's responses. c. Complete a medication error report. d. Determine if the pain was relieved.

(a)--This is a medication error. The first step in addressing a medication error is to access for any side effects of the medication on the patient. Certain analgesics may cause respiratory depression, so it is essential to monitor for vital sign changes or respiratory distress. Once noting the patient is stable, you may then contact the provider, document the response, and complete a medication error report.

38. The nurse is discharging an adult woman who was hospitalized for 6 days for treatment of pneumonia. While the nurse is reviewing the prescribed medications, the client appears anxious. What action is most important for the nurse to implement? a. Instruct the client to repeat the medication plan b. Encourage client to take a PRN antianxiety drug c. Provide written instructions that are easy to follow d. Include a family member in the teaching session

(a)--When a patient is anxious, they may not hear or retain what you are trying to teach them. So it is necessary to ask them repeat back your instructions so you can be sure they understood your teaching. This is called "teach back". (B) is not necessary and medication should not be a first line to rid a patient of anxiety. (C) while helpful, some patients may not be willing to read or if anxious, could not be focused enough to read. (D) while helpful, the patient may not always have a family member with them.

5. A 54-year-old male client and his wife were informed this morning that he has terminal cancer. Which nursing intervention is likely to most beneficial? A. Ask her how she would like to participate in the client's care. B. Provide the wife with information about hospice C. Encourage the wife to visit after painful treatments are completed D. Refer her to support group for family members of those dying of cancer

(a)--While the client's wife may be grieving and need support, the priority for the client and client's wife is to make sure the wife feels comfortable participating in the client's care, if at all. Most people have an easier time coming to terms with the death of a loved one when they are involved in their care. (D) is a nice gesture, but will be more appropriate at a later time.

31. The unlicensed assistive personnel (UAP) describes the appearance of the bowel movement of several clients. Which description warrants additional follow up by the nurse? (select all that applies). a. Solid with red streaks. Indicative of hemorrhoids or a lower GI Bleed. Not normal. b. Brown liquid Indicative of an upset stomach or C.Diff. Not normal. c. Multiple hard pellets. Indicative of dehydration or a lack of fiber. Not normal. d. Formed but soft. This is a normal poop! e. Tarry appearance. This is indicative of a lower GI bleed. Not normal.

(a,b,c,e)--Any reported abnormal results from a UAP warrants a follow-up from the Nurse. It's your job! Not theirs'!

7. The nurse assesses a client who has a nasal cannula delivering oxygen at 2 L/min. To assess for skin damage related to the cannula, which areas should the nurse observe? (Select all that apply). A. Tops of the ear B. Bridge of the nose C. Around the nostrils D. Over the cheeks E. Across the forehead

(a,c,d)--This is proper placement of a nasal cannula. Constant pressure from the tubing may create skin damage to the areas of skin and bony prominences the nasal cannula will be resting on.

34. What assessment finding places a client at risk for problems associated with impaired skin integrity? a. Scattered macula of the face b. Capillary refill 5 seconds c. Smooth nail texture d. Absence of skin tenting

(b)--"At risk for problems associated with impaired skin integrity" . "At risk" is the key here; (B) normal capillary refill time is less than 3 seconds. Anything greater means that the patient is not having effective blood flow to the fingers and can cause skin to start necrosis (die). (A)(C)(D) are all normal findings.

36. The nurse attaches a pulse oximeter to a client's fingers and obtains an oxygen saturation reading of 91%. Which assessment finding most likely contributes to this reading? a. BP 142/88 mmHg b. 2+ edema of fingers and hands c. Radial pulse volume is +3 d. Capillary refill time is 2 seconds

(b)--(A) blood pressure has nothing to do with the patient's O2 saturation. (C) This means the patient's pulse is strong and appropriate, but does nothing to the O2 stat. (D) The patient is having appropriate blood flow and this has nothing to do with O2 stats. (B) is correct because when a patient has too much fluid in their body, they will have 2+ edema of fingers and hands, which means this fluid could also be in their lungs or sitting on their chest. Thus, decreasing their O2 stats.

16. A female client's significant other has been at her bedside providing reassurances and support for the past 3 days, as desired by the client. The client's estranged husband arrives and demands that the significant other not be allowed to visit or be given condition updates. Which intervention should the nurse implement? a. Obtain a prescription from the healthcare provider regarding visitation privileges b. Request a consultation with the ethics committee for resolution of the situation c. Encourage the client to speak with her husband regarding his disruptive behavior d. Communicate the client's wishes to all members of the multidisciplinary team

(b)--(A) is not appropriate. (C) would cause excessive stress to the patient and the patient may not want to see her estranged husband at all. (D) while appropriate, it does not help calm the estranged husband or get him off the premises. (B) is most appropriate and professionals who are trained in ethical issues like this can take care of the situation.

45. A CVA (stoke) patient goes into respiratory distress and is placed on a ventilator. The client's daughter arrives with a durable power of attorney, and a living will that indicates there should be no extraordinary life saving measures. What action should the nurse take? a. Refer to the risk manager b. Notify the healthcare provider c. Discontinue the ventilator d. Review the medical record

(b)--A "DNR" or "FULL" code status change can only be completed by the Physician, and therefore requires you to first notify the Physician to put in an order for DNR to extubate the patient. If there is no specific order stating that the patient is a DNR, assume they are a full code! Only a living will or active Durable power of attorney can legally change the status of an unresponsive patient. (C ) can happen after (B) is completed and the MD changes the code status. (D) not this option because active legal documents will override possibly out of date medical records.

13. The nurse is teaching a client how to do active range of motion (ROM) exercises. To exercise the hinge joints, which action should the nurse instruct the client to perform? a. Tilt the pelvis forwards and backwards b. Bend the arm by flexing the ulnar to the humerus c. Turn the head to the right and left d. Extend the arm at the ide and rotate in circles

(b)--Active range of motion is when the patient is completing the physical activity with physical assistance or manipulation from the nurse. The elbow is a hinge joint, as stated in the question, and should be exercised by bending the forearm (ulnar) to the humerus (bicep area).

27. The nurse is providing wound care to a client with a stage 3 pressure ulcer that has a large amount of eschar. The wound care prescription states "clean the wound and then apply collagenase." Collagenase is a debriding agent. The prescription does not specify a cleaning method. Which technique should the nurse use to cleanse the pressure ulcer? a. Lightly coat the wound with povidone-iodine solution b. Irrigate the wound with sterile normal saline c. Flush the wound with sterile hydrogen peroxide d. Remove the eschar with a wet-to-dry dressing

(b)--Eschar is a natural part of a healing wound and can only be removed by the physician or wound care nurse. Unless directly stated in wound care notes, the ONLY solution to be used to clean a wound is sterile normal saline.

49. Which assessment data reflects the need for the nurses to include the problem, "Risk for falls" in a client's plan of care? a. Recent serum hemoglobin level of 16g/dL b. Opioid analgesic received one hour ago c. Stooped posture with a steady gait d. Expressed feelings of depression

(b)--Opioid analgesics can cause impaired balance, which puts the patient at a risk for falls. (A) is normal for males and slightly elevated for females but has nothing to do with increased fall risk. (C) "steady gait" negates the stooped posture and the patient is not prone to falls. (D) depression does not cause falls.

37. The nurse is caring for a hospitalized client who was placed in restraints due to confusion. The family removes the restraints while they are with the client. When the family leaves, what action should the nurse take first? a. Apply the restraints to maintain the client's safety. b. Reassess the client to determine the need for continuing restraints. c. Document the time the family left and continue to monitor the client. d. Call the healthcare provider for a new prescription.

(b)--Restraints, whenever possible should be discontinued when appropriate to decrease the risk of skin breakdown or injury to the patient. Since the patient successfully had them off while the family was around, before you place them back on the patient, you should first see if they are even necessary to begin with. Therefore (A) is not appropriate until after the assessment is done. (C) is not necessary, but you should continue to monitor the patient. (D) is only necessary when you put the restraints back on the patient.

46. Earlier this morning, an elderly Hispanic female was discharged to a LTC facility. The family members are now gathered in the hallway outside her room. What is the best action? a. Ask the family to wait in the cafeteria when the next of kin makes the necessary arrangements b. Provide space and privacy for the family to share their concerns about the client's discharge c. Ask the social worker to encourage the family to clear the hallway d. Explain to the family the client's need for privacy so that she can make independent decisions

(b)--The problem here is that too many people in the hospital hallway can be a safety hazard, so the goal is to create a safe space for the family. Therefore (B) is the most appropriate answer. (A) is incorrect because it is moving the family into an open area which could violate the patient's privacy and the next of kin may not be the decision makers for the patient. (C) the social worker is busy, and it's not their job! (D) LTC infers the patient is elderly and may lack the ability to make independent decisions.

3. The father of an 11-year-old client reports to the nurse that the client has been "wetting the bed" since the passing of his mother and is concerned. Which action is most important for the nurse to enact? A. Reassure the father that it is normal for a pre-teen to wet the bed during puberty B. Inform the father that nocturnal emissions are abnormal and his son is developmentally delayed C. Inform the father that it is most important to let the son know that nocturnal emissions are normal after trauma D. Refer the father and the client to a psychologist

(c) --It is common for adolescents to regress in their biological progression after experiencing a severe trauma, like losing a parent, sibling, or friend. While uncomfortable for the adolescent and parent, it is nothing to be concerned for. Often times, as the patient grieves or comes to terms with the trauma, the nocturnal emissions will cease.

4. The nurse explains to an older adult male the procedure for collecting a 24-hour urine specimen for creatinine clearance. Which action is most important for the nurse to include in their care plan for the shift? A. Assess the client for confusion and reteach the procedure B. Check the urine for color and texture C. Empty the urinal contents into the 24-hour collection container D. Discard the contents of the urinal

(c)--An "older adult male" in the question may imply that the patient may have an altered mental status or be demented. While suggesting, it is not directly stated, therefore (A) is inappropriate. (B) is incorrect because the lab will be assessing the collection specimen after the test is complete. (C) is correct because the nurse should first discard the first specimen, then begin to collect and record the time the first urine specimen was collected. It is important to have strict documentation for output, and to collect every urine specimen within that 24 hour period, otherwise the test must be restarted. (D) defeats the purpose of the 24-hour urine collection test.

6. A client who has a body mass index (BMI) of 30 is requesting information on the initial approach to a weight loss plan. Which action should the nurse recommend first? A. Plan low carbohydrate and high protein meals B. Engage in strenuous activity for an hour daily C. Keep a record of food and drinks consumed daily D. Participated in a group exercise class 3 times a week

(c)--BMI of 30 indicates the patient is obese. (A) While a good step, it is not what should be completed first. (B) While a good step, it is not what should be completed first. (C) The best thing to recommend is to have the patient keep a food journal to be able to go back and track their calorie intake; it may be helpful when meal planning or creating a workout routine plan. (D) Would be appropriate later.

28. A client is admitted with a fever of unknown origin. To assess fever patterns, which intervention should the nurse implement? a. Document the client's circadian rhythms b. Assess for flushed, warm skin regularly c. Measure temperature at regular intervals d. Vary sites for temperature measurement

(c)--In order to best assess when the fevers are coming, it would be best for the RN to measure the patient's temperature once an hour, in regular intervals so comparisons can be made with regards to the patient's activities or medications. (A) Circadian rhythms may have little effect on body temperature. (B) A patient could be flushed and not have a fever, so this is not a precise way to measure temperature. (D) Only one consistent site should or method should be used to measure temperature.

20. While changing a client's post-operative dressing, the nurse observes a red and swollen wound with a moderate amount of yellow and green drainage and a foul odor. Given there is a positive MRSA, which is the most important action for the nurse to take? A. Force oral fluids B. Request a nutrition consult C. Initiate contact precautions D. Limit visitors to immediate family only

(c)--MRSA is a type of antibiotic resistant bacteria and a patient with this should be placed on contact precautions. (A) oral fluids will not help rid the patient of the infection. (B) nor nutrition. (D) limiting visitors to immediate family is not necessary as anyone is at risk for contracting MRSA from an infected wound.

48. A male Native American presents to the clinic with complaints of frequent abdominal cramping and nausea. He states that he has chronic constipation and has not had a bowel movement in five days, despite trying several home remedies. Which intervention is most important for the nurse to implement? a. Evaluate the stool samples for presence of blood b. Assess for the presence of an impaction c. Determine what home remedies were used d. Obtain list of prescribed home medication

(c)--The key aspect here is that the patient is "Native American" and that he has had constipation despite trying "several home remedies" (assume herbal). The most important thing to do is to have an exact list of the home remedies so that you can be sure that any prescribed medications don't interact with the remedies that are already in the patient's system.

47. A client with rheumatoid arthritis is experiencing chronic pain in both hands and wrists. Which information about the client is most important for the nurse to obtain when planning care? a. Amount of support provided by family members b. Measurement of pain using a scale of 0 to 10 c. The ability to perform ADLs d. Nonverbal behaviors exhibited when pain occurs

(c)--The key words here are "Chronic pain" meaning the patient has dealt with this pain and lived with this pain for awhile. Therefore (B) is incorrect and more appropriate for acute pain or before you give pain meds. (A) is nice but has nothing to do with the patient's hands or wrists and the patient's plan of care. (D) while important, it is not the most important. With patients who have rheumatoid arthritis, ADLs are most important because if the patient has too much pain, they may not be able to complete their own basic care.

2. Which assessment data would provide the most accurate determination of proper placement of a nasogastric tube? A) Aspirating gastric contents to assure a pH value of 4 or less. B) Hearing air pass in the stomach after injecting air into the tubing. C) Examining a chest x-ray obtained after the tubing was inserted. D) Checking the remaining length of tubing to ensure that the correct length was inserted. A) Aspirating gastric contents to assure a pH value of 4 or less.

(c)--This is a method used to determine proper placement of NG tubing, but not the most accurate. B) Hearing air pass in the stomach after injecting air into the tubing. This is a method used to determine proper placement of NG tubing, but not the most accurate. C) Examining a chest x-ray obtained after the tubing was inserted. After placing an NG-tube, the placement of the tube is confirmed via x-ray since it is the most accurate way to ensure the tube has not been placed in the lungs, which would pose an aspiration risk. D) Checking the remaining length of tubing to ensure that the correct length was inserted. This is not an indicator of proper placement. You could very well be in a lung.

32. A female UAP is assigned to take the vital signs of a client with pertussis for whom droplet precautions have been implemented. The UAP requests a change in assignment because she has not yet been fitted for a particulate filter mask. Which action should the nurse take? a. Advise the UAP to wear a standard face mask to take vital signs, and then get fitted for a filter mask before providing personal care b. Send the UAP to be fitted for a particulate filter mask immediately so she can provide care to this client c. Instruct the UAP that a standard mask is sufficient for the provision of care for the assigned client d. Before changing assignments, determine which staff members have fitted particulate filter masks

(c)--This is one of Elsevier's famous trick questions! You need to pay attention to the wording of the question and know your precaution measures and differences! The question asks about DROPLET precautions, not AIRBORNE precautions, so only a standard mask is required to enter this room. AIRBORNE precautions are for patients' with TB, meningitis, etc. Droplet are for patients with the flu, rhinovirus, etc. Be careful!!

17. When measuring vital signs, the nurse observes that a client is using accessory neck muscles during respirations. What follow-up action should the nurse take first? a. Determine pulse pressure b. Auscultate heart sounds c. Measure oxygen saturation d. Check for neck vein distention

(c)--Using accessory neck muscles during respirations is a serious sign of respiratory distress. The patient is a having a hard time breathing and as such, the first thing to do would be to measure oxygen saturation. (A) Pulse pressure is the difference between systolic and diastolic blood pressure. It is measured in millimeters of mercury (mmHg). It represents the force that the heart generates each time it contracts. (B) This has nothing to do with the heart. (D) Neck vein distention that is present is a sign of increased CVP (force on the aorta) and is not appropriate here.

25. While interviewing a client, the nurse records the assessment in the electronic health record. Which statement is most accurate regarding electronic documentation during an interview? a. The client's comfort level is increased when the nurse breaks eye contact to type notes into the record b. The interview process is enhanced with electronic documentation and allows the client to speak at a normal pace c. The nurse has limited ability to observe nonverbal communication while entering the assessment electronically d. Completing the electronic record during an interview is a legal obligation of the examining nurse

(c)--While looking at the computer, the RN may have limited ability to visualize the nonverbal communication from the patient. (A) you should be trying to initiate some, not constant eye contact with your patient while completing electronic documentation. (B) The patient can speak at a normal pace with or without HER. (D) while the electronic record during an interview becomes a legal document, it is an obligation for your job, not for legal matters.

54. The nurse plans to assist a male client out of bed for the first time since his surgery yesterday. His wife objects and tells the nurse to get out of the room because her husband is too ill to get out of bed. A. Administer nasal oxygen at a rate of 5 L/min B. Help the client to lie back down in the bed C. Quickly pivot the client to the chair and elevate the legs D. Check the client's blood pressure and pulse deficit

(d)--(A) is incorrect because it does not say that the patient has a decrease in O2 stats. (B) is incorrect because you assume the patient is already laying down and you have a task at hand. (C) MOVE SLOW after surgery! Never quickly move a patient. You could injury yourself and the client. (D) is correct; make sure they are physiologically stable first before moving them.

12. A male client with unstable angina needs a cardiac catheterization, so the healthcare provider explains the risks and benefits of the procedure, and then leaves to set up for the procedure. When the nurse presents the consent form for signature, the client hesitates and asks how the wires will keep his heart going. Which action should the nurse take? a. Answer the client's specific questions with a short understandable explanation b. Postpone the procedure until the client understands the risks and benefits c. Call the client's next of kin and ask them to provide verbal consent d. Page the healthcare provider to return and provide additional explanation

(d)--A patient should not sign a consent if they do not completely understand the procedure, benefits and risks. Although you may have an understanding of the procedure, it is the Physician and physician ONLY who can review the process of the procedure and benefits/risks with the client. That task is out of your scope as an RN.

26. A female client with chronic back pain has been taking muscle relaxants and analgesics to manage the discomfort, but is now experiencing an acute episode of pain that is not relieved by this medication regime. The client tells the nurse that she does not want to have back surgery for a herniated intervertebral disk, and reports that she has found acupuncture effective in resolving past acute episodes. Which response is best for the nurse to provide? a. Surgery removes the disk and is the only treatment that can totally resolve the pain b. The medication regimen you previously used should be re-evaluated for dose adjustment c. Massage and hot pack treatments are less invasive and can provide temporary relief d. Acupuncture is a complementary therapy that is often effective for management of pain

(d)--Acknowledgment of pain-relieving methods that work for the patient is extremely important. Since the patient has stated that acupuncture, a complementary therapy and alternative therapy of pain relief, has worked for the patient, you can acknowledge the effectiveness. (A) while effective, surgery does not always relieve pain and is not necessary for acute pain. (B) medications can be re-evaluated but has not helped the patient's current pain and is not appropriate. (C) while massage and heat can help pain, the patient has already stated that acupuncture has been effective at managing her pain and is another safe method of pain relief.

42. A male client presents to the clinic stating that he has a high stress job and is having difficulty falling asleep at night. The client reports having a constant headache and is seeking medication to help him sleep. Which intervention should the nurse implement? a. Determine the client's sleep and activity pattern b. Obtain prescription for client to take when stressed c. Refer client for a sleep study and neurological follow-up d. Teach coping strategies to use when feeling stressed

(d)--Before medicating a patient, always consider other non-pharmaceutical therapeutic methods. In this case, the patient stated he has a high stress job that is CAUSING him to have difficulty sleeping. Therefore, teaching coping strategies to use when feeling stressed could help him sleep without the need for a sleeping pill. (A) is incorrect because the patient already said he is having difficulty sleeping, this would extra nice to know information. (B) save prescriptions for the last step if all other methods fail. (C) again, this is if the other methods fail first.

24. An older adult male client is admitted to the medical unit following a fall at home. When undressing him, the nurse notes that he is wearing an adult diaper and skin breakdown is obvious over his sacral area. What action should the nurse implement first? a. Establish a toileting schedule to decrease episodes of incontinence b. Complete a functional assessment of the client's self-care abilities c. Apply a barrier ointment to intact areas that may be exposed to moisture d. Determine the size and depth of skin breakdown over the sacral area

(d)--Before you can treat the skin breakdown, you must first measure and record the affected area so you can have something to compare after you begin your interventions/treatments. Then you can continue with (A)(B) and (C).

19. Which instruction should the nurse include in the discharge teaching plan for an adult client with hypernatremia? a. Monitor daily urine output volume b. Drink plenty of water whenever thirsty c. Use salt tablets for sodium content d. Review food labels for sodium content

(d)--Hypernatremia is when a patient has a Sodium level that is too high, therefore it is most appropriate to teach the patient to check sodium levels on food labels before discharging. (A) While a high sodium level can decrease urine output, it is most important for the patient to be able to identify high sodium foods to decrease the risk of developing hypernatremia again. (B) Drinking too much water when thirsty can cause hyponatremia. (C) is incorrect because the patient is already at risk for developing high sodium levels again.

53. The nurse is reviewing the signed operative consent with a client who is admitted for the removal of a lipoma on the left leg. The client states that the consent form should say the removal of a lipoma on the right leg. Which intervention should the nurse implement? A. Notify the OR staff of the client's confusion B. Have the client sign a new surgical consent C. Add the additional information to the consent D. Inform the surgeon about the client's concern

(d)--If there are any discrepancies or concerns from the patient prior to signing the surgery consent, then the RN needs to call the surgeon to come speak to the patient to clear any confusion. (A) this client is not confused, never assume they are. (B) You will do this after the proper leg is discussed with the surgeon. (C) never add information to a consent! That is not your job and is only done with the surgeon's knowledge.

33. In-home hospice care is arranged for a client with stage 4 lung cancer. While the palliative nurse is arranging for discharge, the client verbalizes concerns about pain. What action should the nurse implement? a. Explain the respiratory problems that can occur with morphine use. b. Teach family how to evaluate the effectiveness of analgesics. c. Recommend asking the healthcare professional for a patient-controlled analgesic (PCA) pump. d. Provide client with a schedule of around-the-clock prescribed analgesic use.

(d)--It is common for cancer patients to have recurrent pain and it is the patient's right to ask about aspects of their care, such as pain management. (A) Morphine is actually used for both pain and to help patient's breathe better, it is not an opioid and does not cause respiratory depression. (B) The family is not trained to do this and is not appropriate. Pain is subjective and only the patient can say if the analgesics are effective. (C) PCA pumps are given in extreme cases when pain cannot be controlled by a previously tried pain management schedule like (D).

39. What instruction should the nurse provide for an UAP caring for a client with MRSA who has an order for contact precautions? a. Do not allow visitors until precautions are discontinued b. Wear sterile gloves when handling the client's body fluid c. Have the client wear a mask whenever someone enters the room d. Don a gown and gloves when entering the return

(d)--MRSA requires contact precautions; an isolation gown, gloves. (A) visitors can still come if they wear the appropriate PPE. (B) not necessary as sterile gloves are not needed for body fluid. (C) a standard mask is only needed for droplet precautions...minus this time of COVID.

41. UAP has lowered the head of the bed to change the lines for a client who is bedbound with a foley catheter and enteral tube feeds. Which change from the client warrants the most immediate intervention by the nurse? a. A feeding is infusing at 40 mL/hr through an enteral feeding tube b. The urine meter attached to the urinary drainage bag is completely full c. There is a large dependent loop in the client's urinary drainage tubing d. Purulent drainage is present around the insertion site of the feeding tube

(d)--The wording here is key. If it were an NG tube feeding, then the patient must ALWAYS be at, at least a 30 degree angle. BUT because it says an enteral tube feeds (through the stomach), it is ok to lower the bed down all the way. That being said, purulent drainage (D) shows that the tube feeds are leaking and thus the nurse needs to pause the feeds and sit the patient up again. This is the most immediate need over the urine drainage bag being full or kinked.

55. When entering the room of an adult male, the nurse finds that the client is very anxious. Before providing care, what action should the nurse take first? a. Divert the client's attention b. Call for additional help from staff c. Document the planned action d. Re-assess the client situation

(d)--When a patient is anxious, the first thing to do is re-assess them and find out why the are anxious. (A) can be helpful after finding out why they are anxious. (B) the patient is not a threat, no help is needed. (C) appropriate after you find out why they are anxious.

51. The nurse is conducting an initial admission assessment for a woman who is Mexican American and who is scheduled to deliver a baby by C-section in the next 24 hours. What should the nurse include in the assessment? a. Provider an interpreter to convey the meaning of words and messages in translation b. Commend the client for her patience after a long wait in the admission process c. Arrange for the hospital chaplain to visit the client during her hospital stay d. Rely on cultural norms as the basis for providing nursing care for this client

(d)--Whenever Elsevier points out the ethnicity or race of a patient, there are key cultural aspects you should look for in your answer (D). Don't assume (A) is correct because she could very well speak English!

15. When assessing a male client, the nurse finds that he is fatigued, and is experiencing muscle weakness, leg cramps, and cardiac dysrhythmias. Based on these findings, the nurse plans to check the client's laboratory values to validate the existence of which? a. Hyperphosphatemia b. Hypocalcemia c. Hypermagnesemia d. Hypokalemia

(d)--a. Hyperphosphatemia- muscle cramps, tetany, and perioral numbness or tingling b. Hypocalcemia - paresthesia, muscle spasms, cramps, tetany, numbness, and seizures c. Hypermagnesemia - (levels greater than 12 mmol/dL) can lead to cardiovascular complications (hypotension, and arrhythmias) and neurological disorder (confusion and lethargy) d. Hypokalemia- muscle weakness, leg cramps, and cardiac dysrhythmias. Normal range is 3.5- 5.0.

43. The nurse is teaching a client about the use of the syringes and needles for home administration of medications. Which action by the client indicates an understanding of standard precautions? a. Remove needle before discarding used syringes b. Wear gloves to dispose of the needle and syringe c. Don a face mask before administering the medication d. Washes hands before handling the needle and syringe

(d)--a. Remove needle before discarding used syringes (not safe, you could poke yourself) b. Wear gloves to dispose of the needle and syringe (yeah but not the best answer) c. Don a face mask before administering the medication (this is droplet precautions) d. Washes hands before handling the needle and syringe (this is standard precautions) Standard precautions; If you don't know these, review them because it is the foundation for nursing!

Triaging colors: Green, Yellow, Red, Black

-(not urgent, can get up and walk) -(not life threatening, can be treated within 30min-2hours) -(highest priority, respiratory issues, Loss of consciousness) -(dead)

An older client who is a resident in a LTC facility has been bedridden for a week. Which finding should the nurse identify as a client risk factor for pressure ulcers?

Answer: Rashes in the axillary, groin, and skin fold regions.

Seconal 0.1 gram PRN at bedtime is prescribed to a client for rest. The scored tablets are labeled grain 1.5 per tablet. How many tablets should the nurse plan to administer? A. 0.5 tablet. B. 1 tablet. C. 1.5 tablets. D. 2 tablets.

15 gr=1 Gm. Converting the prescribed dose of 0.1 grams to grains requires multiplying 0.1 × 15 = 1.5 grains. The tablets come in 1.5 grains, so the nurse should plan to administer 1 tablet (B). Correct Answer: B

Which patient would you assess first:

1st-Which patient can I look at and they will die if I don't do something 2nd-Most critical state 3rd-ABC 4th-SOB you should be very concerned

A female client asks the nurse to find someone who can translate into her native language her concerns about a treatment. Which action should the nurse take?

Answer: Request and document the name of the certified translator

66. The nurse is preparing a liquid medication for a 2-year-old. The dose is 2.2 mL. What delivery devise will the nurse select to prepare the medication?

3 mL needleless syringe

23. A client with cirrhosis and ascites is receiving furosemide 40 mg BID. The pharmacy provides 20 mg tablets. How many tablets should the client receive each day? [Enter numeric value only]

4 tablets 40 mg BID (BID is 2 times a day). So 40mg x 2 = 80mg/day. 80 mg day/20mg tablets available = 4 tablets a day.

A female client asks the nurse to find someone who can translate into her native language her concerns about a treatment. Which action should the nurse take? A. Explain that anyone who speaks her language can answer her questions. B. Provide a translator only in an emergency situation. C. Ask a family member or friend of the client to translate. D. Request and document the name of the certified translator.

A certified translator should be requested to ensure the exchanged information is reliable and unaltered. To adhere to legal requirements in some states, the name of the translator should be documented (D). Client information that is translated is private and protected under HIPAA rules, so (A) is not the best action. Although an emergency situation may require extenuating circumstances (B), a translator should be provided in most situations. Family members may skew information and not translate the exact information, so (C) is not preferred. Correct Answer: D

During a physical assessment, a female client begins to cry. Which action is best for the nurse to take? A. Request another nurse to complete the physical assessment. B. Ask the client to stop crying and tell the nurse what is wrong. C. Acknowledge the client's distress and tell her it is all right to cry. D. Leave the room so that the client can be alone to cry in private.

Acknowledging the client's distress and giving the client the opportunity to verbalize her distress (C) is a supportive response. (A, B, and D) are not supportive and do not facilitate the client's expression of feelings. Correct Answer: C

A client with acute hemorrhagic anemia is to receive four units of packed RBCs (red blood cells) as rapidly as possible. Which intervention is most important for the nurse to implement? A. Obtain the pre-transfusion hemoglobin level. B. Prime the tubing and prepare a blood pump set-up. C. Monitor vital signs q15 minutes for the first hour. D. Ensure the accuracy of the blood type match.

All interventions should be implemented prior to administering blood, but (D) has the highest priority. Any time blood is administered, the nurse should ensure the accuracy of the blood type match in order to prevent a possible hemolytic reaction. Correct Answer: D

A client with pneumonia has a decrease in oxygen saturation from 94% to 88% while ambulating. Based on these findings, which intervention should the nurse implement first? A. Assist the ambulating client back to the bed. B. Encourage the client to ambulate to resolve pneumonia. C. Obtain a prescription for portable oxygen while ambulating. D. Move the oximetry probe from the finger to the earlobe.

An oxygen saturation below 90% indicates inadequate oxygenation. First, the client should be assisted to return to bed (A) to minimize oxygen demands. Ambulation increases aeration of the lungs to prevent pooling of respiratory secretions, but the client's activity at this time is depleting oxygen saturation of the blood, so (B) is contraindicated. Increased activity increases respiratory effort, and oxygen may be necessary to continue ambulation (C), but first the client should return to bed to rest. Oxygen saturation levels at different sites should be evaluated after the client returns to bed (D). Correct Answer: A

While instructing a male client's wife in the performance of passive range-of-motion exercises to his contracted shoulder, the nurse observes that she is holding his arm above and below the elbow. What nursing action should the nurse implement? A. Acknowledge that she is supporting the arm correctly. B. Encourage her to keep the joint covered to maintain warmth. C. Reinforce the need to grip directly under the joint for better support. D. Instruct her to grip directly over the joint for better motion.

Answer: (A)Acknowledge that she is supporting the arm correctly

Which snack food is best for the nurse to provide a client with myasthenia gravis who is at risk for altered nutritional status? A. Chocolate pudding. B. Graham crackers. C. Sugar free gelatin. D. Apple slices.

Answer: (A)Chocolate pudding

In developing a plan of care for a client with dementia, the nurse should remember that confusion in the elderly

Answer: Often follows relocation to new surroundings

After completing an assessment and determining that a client has a problem, which action should the nurse perform next? A. Determine the etiology of the problem. B. Prioritize nursing care interventions. C. Plan appropriate interventions. D. Collaborate with the client to set goals.

Answer: (A)Determine the etiology of the problem

An African-American grandmother tells the nurse that her 4-year-old grandson is suffering with "miseries." Based on this statement, which focused assessment should the nurse conduct? A. Inquire about the source and type of pain. B. Examine the nose for congestion and discharge. C. Take vital signs for temperature elevation. D. Explore the abdominal area for distension.

Answer: (A)Inquire about the source and type of pain.

When assisting an 82-year-old client to ambulate, it is important for the nurse to realize that the center of gravity for an elderly person is the A. Arms. B. Upper torso. C. Head. D. Feet.

Answer: (B) Upper Torso

An elderly male client who is unresponsive following a cerebral vascular accident (CVA) is receiving bolus enteral feedings though a gastrostomy tube. What is the best client position for administration of the bolus tube feedings? A. Prone. B. Fowler's. C. Sims'. D. Supine.

Answer: (B)Fowler's

A female client with a nasogastric tube attached to low suction states that she is nauseated. The nurse assesses that there has been no drainage through the nasogastric tube in the last two hours. What action should the nurse take first? A. Irrigate the nasogastric tube with sterile normal saline. B. Reposition the client on her side. C. Advance the nasogastric tube an additional five centimeters. D. Administer an intravenous antiemetic prescribed for PRN use.

Answer: (B)Reposition the client on her side

An elderly client who requires frequent monitoring fell and fractured a hip. Which nurse is at greatest risk for a malpractice judgment? A. A nurse who worked the 7 to 3 shift at the hospital and wrote poor nursing notes. B. The nurse assigned to care for the client who was at lunch at the time of the fall. C. The nurse who transferred the client to the chair when the fall occurred. D. The charge nurse who completed rounds 30 minutes before the fall occurred.

Answer: (B)The nurse who transferred the client to the chair when the fall occurred.

During a visit to the outpatient clinic, the nurse assesses a client with severe osteoarthritis using a goniometer. Which finding should the nurse expect to measure? A. Adequate venous blood flow to the lower extremities. B. Estimated amount of body fat by an underarm skinfold. C. Degree of flexion and extension of the client's knee joint. D. Change in the circumference of the joint in centimeters.

Answer: (C)Degree of flexion and extension of the client's knee joint.

The nurse is completing a mental assessment for a client who is demonstrating slow thought processes, personality changes, and emotional lability. Which area of the brain controls these neuro-cognitive functions? A. Thalamus. B. Hypothalamus. C. Frontal lobe. D. Parietal lobe.

Answer: (C)Frontal Lobe

An unlicensed assistive personnel (UAP) places a client in a left lateral position prior to administering a soap suds enema. Which instruction should the nurse provide the UAP? A. Position the client on the right side of the bed in reverse Trendelenburg. B. Fill the enema container with 1000 ml of warm water and 5 ml of castile soap. C. Reposition in a Sim's position with the client's weight on the anterior ilium. D. Raise the side rails on both sides of the bed and elevate the bed to waist level.

Answer: (C)Reposition in a Sim's position with the client's weight on the anterior ilium.

Which intervention is most important for the nurse to implement for a male client who is experiencing urinary retention? A. Apply a condom catheter. B. Apply a skin protectant. C. Encourage increased fluid intake. D. Assess for bladder distention.

Answer: (D)Assess for bladder distention.

During the initial morning assessment, a male client denies dysuria but reports that his urine appears dark amber. Which intervention should the nurse implement? A. Provide additional coffee on the client's breakfast tray. B. Exchange the client's grape juice for cranberry juice. C. Bring the client additional fruit at mid-morning. D. Encourage additional oral intake of juices and water.

Answer: (D)Encourage additional oral intake of juices and water.

An elderly resident of a long-term care facility is no longer able to perform self-care and is becoming progressively weaker. The resident previously requested that no resuscitative efforts be performed, and the family requests hospice care. What action should the nurse implement first? A. Reaffirm the client's desire for no resuscitative efforts. B. Transfer the client to a hospice inpatient facility. C. Prepare the family for the client's impending death. D. Notify the healthcare provider of the family's request.

Answer: (D)Notify the healthcare provider of the family's request.

Twenty minutes after beginning a heat application, the client states that the heating pad no longer feels warm enough. What is the best response by the nurse? A. That means you have derived the maximum benefit, and the heat can be removed. B. Your blood vessels are becoming dilated and removing the heat from the site. C. We will increase the temperature 5 degrees when the pad no longer feels warm. D. The body's receptors adapt over time as they are exposed to heat.

Answer: (D)The body's receptors adapt over time as they are exposed to heat.

The healthcare provider prescribes morphine sulfate 4mg IM STAT. Morphine comes in 8 mg per ml. How many ml should the nurse administer?

Answer: 0.5 ml.

The healthcare provider prescribes the diuretic metolazone 7.5mg PO. Metolazone is available in 5mg tablets. How much should the nurse plan to administer?

Answer: 1 1/2 tablets

Seconal 0.1 gram PRN at bedtime is prescribed to a client for rest. The scored tablets are labeled grain 1.5 per tablet. How many tablets should the nurse plan to administer?

Answer: 1 tab

Secobarbital (Seconal) 150mg is prescribed at bedtime for a male client who is scheduled for surgery in the morning. The scored tablets are labeled 0.1 g/tablet. How many tablets should the nurse administer?

Answer: 1.5

A 73 year old female client had a hemiarthroplasty of the left hip yesterday due to a fracture resulting from a fall. In reviewing hip precautions with the client, which instruction should the nurse include in the client's teaching plan?

Answer: Place a pillow between your knees while lying in bed to prevent hip dislocation.

The nurse mixes 50mg of Nipride in 250ml of D5W and plans to administer the solution at a rate of 5mcg/kg/min to a client weighing 182lbs. Using a drip factor of 60gtt/ml, how many drops per min should the client receive?

Answer: 124gtt/min lbs to kg 182/2.2 = 82.73kg Dosage for the client 5mcg X 82.73 =413mcg/min mcg / ml 250/50000mcg =200 mcg/ml 2.07ml/min with a drip factor of 60gtt/ml 60 x 2.07 =124.28gtt/min

The IV infusion terbutaline sulfate 5mg in 500ml of D5W, infusing at a rate of 30 mcg/min, is prescribed for a client in premature labor. How many ml/hr should the nurse set the infusion pump?

Answer: 180mL/hr

A client is to receive 10mEq of KCL diluted in 250ml of normal saline over 4hrs. At what rate should the nurse set the clients IV infusion pump?

Answer: 63ml/hr

A healthcare provider prescribes an IV infusion of 1000ml of Ringer's Lactate with 30 units of Pitocin to run in over 4hrs for a client who has just delivered a 10lbs infant via cesarean section. The tubing has been changed to a 20gtt/ml administration set. The nurse plans to set the flow rate to how many gtt/min?

Answer: 83 gtt/min 20gtt/ml x 1000 ml/4hrs x 1hr/60mins = 83gtt/min

A male client is being discharged with a prescription for the bronchodilator theophylline tells the nurse that he understands he is to take three doses of the medication each day. Since, at the time of discharge, timed-release capsules are not available, which dosing schedule should the nurse advise the client to follow?

Answer: 8am, 4pm, midnight

What is the most important reason for starting IV infusions in the upper extremities rather than the lower extremities of adults?

Answer: A decreased flow rate could result in the formation of a thrombosis.

The nurse is assessing the nutritional status of several clients. Which client has the greatest nutritional need for additional intake of protein?

Answer: A lactating woman nursing her 3 day old infant.

Which response by a client with a nursing diagnosis of "Spiritual Distress" indicates to the nurse that a desired outcome measure has been met?

Answer: Accepts that punishment from God is not related to illness. Acceptance that she is not being punished by God indicates a desired outcome

A hospitalized male client is receiving nasogastric tube feedings via a small-bore tube and a continuous pump infusion. He reports that he had a bad bout of severe coughing a few minutes ago, but feels fine now. What action is best for the nurse to take?

Answer: After clearing the tube with 30 ml of air, check the pH of fluid withdrawn from the tube.

The nurse is teaching a client with numerous allergies how to avoid allergens. Which instruction should be included in this teaching plan?

Answer: Avoid any types of spays, powders, and perfumes.

On admission, a client presents a signed living will that includes a DNR prescription. When the client stops breathing, the nurse performs CPR and successfully revives the client. What legal issues could be brought against the nurse?

Answer: Battery

A client who is a Jehovah's witness is admitted to the nursing unit. Which concern should the nurse have for planning care in terms of the client's beliefs?

Answer: Blood transfusions are forbidden

An elderly client with a fractured left hip is on strict bedrest. Which nursing measure is essential to the client's nursing care?

Answer: Gently lift the client when moving into a desired position

A resident in a skilled nursing facility for short term rehab after a hip replacement tells the nurse, "I don't want any more blood taken for those useless tests" Which narrative documentation should the nurse enter into the client's medical record?

Answer: Healthcare provider notified of client's refusal to have blood specimens collected for testing.

A client is receiving a cephalosporin antibiotic IV and complains of pain and irritation at the infusion site. the nurse observes erythema, swelling, and a red streak along the vessel above the IV access site. Which action should the nurse take at this time?

Answer: Initiate an alternate site for the IV infusion of the medication

At the time of the first dressing change, the client refuses to look at her mastectomy incision. The nurse tells the client that the incision is healing well, but the client refuses to talk about it. What would be an appropriate response to the client's silence?

Answer: It is OK if you don't want to talk about your surgery. I will be available when you are ready

An adult male client with a history of hypertension tells the nurse that he is tired of taking antihypertensive medications and is going to try spiritual meditation instead. What should be the nurse's first response?

Answer: It is important that you continue your medication while learning to meditate.

During the daily nursing assessment, a client begins to cry and states that the majority of family and friends have stopped calling and visiting. What action should the nurse take?

Answer: Listen and show interest as the client expresses these feelings

Examination of a client complaining of itching on his right arm reveals a rash made up of multiple flat areas of redness ranging from pinpoint to 0.5cm in diameter. How should the nurse record this finding?

Answer: Localized red rash comprises flat areas, pinpoint to 0.5 cm in diameter.

When assessing a client with wrist restraints, the nurse observes that the fingers on the right hand are blue. What action should the nurse implement first?

Answer: Loosen the right wrist restraint.

A client's infusion of normal saline infiltrated earlier today and approximately 500ml of saline infused into the subcutaneous tissue. The client is now complaining of excruciating arm pain and demanding stronger pain medications. What initial action is most important for the nurse to take?

Answer: Measure the pulse volume and capillary refill distal to the infiltration.

The nurse observes that a male client has removed the covering form an ice pack applied to his knee. What action should the nurse take first?

Answer: Observe the appearance of the skin under the ice pack.

A Sub-Saharan African widowed immigrant woman lives with her deceased husband's brother and his family, which includes the brother-in-law's children and the widow's adult children. Each family member speaks fluent English. Surgery was recommended for the client. What is the best plan to obtain consent for surgery for this client.?

Answer: Tell the surgeon that the brother-in-law will decide after explanation of the proposed surgery is provided to him and the widow.

A postoperative client will need to perform daily dressing changes after discharge. Which outcome statement best demonstrates the client's readiness to manage his wound care after discharge?

Answer: The client demonstrates the wound care procedure correctly.

Which nutritional assessment data should the nurse collect to best reflect total muscle mass in an adolescent? A. Height in inches or centimeters. B. Weight in kilograms or pounds. C. Triceps skin fold thickness. D. Upper arm circumference.

Answer: Upper arm Circumference

During shift change report, the nurse receives report that a client has abnormal heart sounds. Which placement of the stethoscope should the nurse use to hear the client's heart sounds?

Answer: Use the stethoscope bell over the valvular areas of the anterior chest.

A client who is 5'5" tall and weighs 200 pounds is scheduled for surgery the next day. What question is most important for the nurse to include during the preoperative assessment?

Answer: What vitamin and mineral supplements do you take?

The nurse prepares a 1,000 ml IV of 5% dextrose and water to be infused over 8 hours. The infusion set delivers 10 drops per milliliter. The nurse should regulate the IV to administer approximately how many drops per minute? A. 80 B. 8 C. 21 D. 25

Answer:(C)21

88. The nurse is providing care to an 86-year-old admitted for a heart catheterization. The nurse determines the client does not have an advance directive (AD) on file. What are the nurse's next steps? (Select all that apply.)

Ask the client, "Have you considered completing the paperwork for an AD?" Tell the client, "An AD helps the staff provide care according to your wishes."

15. The nurse is preparing to administer a bolus tube feeding. What steps must the nurse include prior to administering the feeding? (Select all that apply.)

Aspirate the stomach contents. Assess bowel sounds. Warm the feeding to room temperature. Assess the pH of the stomach contents.

62. A client in a long-term care facility reports to the nurse, "I have not had a bowel movement in 2 days." What is the nurse's first action?

Assess the client's medical record to determine the client's normal bowel pattern.

92. An older client who had abdominal surgery 3 days earlier was given a barbiturate for sleep and is now requesting to go to the bathroom. What is the priority nursing action for this client?

Assist the client to walk to the bathroom and do not leave the client alone.

107. At hand-off report the off going nurse reports a new 1000 mL IV bag of D5LR was hung at 1845. The prescribed infusion rate is 75 mL/hr. The oncoming nurse assesses the client at 1915 and notes there is less than 50 mL left in the IV bag. What is the nurse's next action?

Auscultate the client's lungs.

16. A client's blood pressure reading is 156/94 mm Hg. Which action should the nurse take first?

Compare the current reading with the client's previously documented blood pressure readings.

**A client comes to the clinic complaining of a productive cough with copious yellow sputum, fever, and chills for the past two days. The first thing the nurse should do when caring for this client is to: a. Encourage fluids b. Administer oxygen3 c. Take the temperature d. Collect a sputum specimen

Baseline vital signs are extremely important; physical assessment precedes diagnostic measures and intervention. This is done after the health care provider makes a medical diagnosis; this is not an independent function of the nurse. Encouraging fluids might be done after it is determined whether a specimen for blood gases is needed; this is not usually an independent function of the nurse. Oxygen is administered independently by the nurse only in an emergency situation. A sputum specimen should be obtained after vital signs and before administration of antibiotics.

After completing an assessment and determining that a client has a problem, which action should the nurse perform next? A. Determine the etiology of the problem. B. Prioritize nursing care interventions. C. Plan appropriate interventions. D. Collaborate with the client to set goals.

Before planning care, the nurse should determine the etiology, or cause, of the problem (A), because this will help determine (B, C, and D). Correct Answer: A

A client who is a Jehovah's Witness is admitted to the nursing unit. Which concern should the nurse have for planning care in terms of the client's beliefs? A. Autopsy of the body is prohibited. B. Blood transfusions are forbidden. C. Alcohol use in any form is not allowed. D. A vegetarian diet must be followed.

Blood transfusions are forbidden (B) in the Jehovah's Witness religion. Judaism prohibits (A). Buddhism forbids the use of (C) and drugs. Many of these sects are vegetarian (D), but the direct impact on nursing care is (B). Correct Answer: B

Which assessment data would provide the most accurate determination of proper placement of a nasogastric tube? A. Aspirating gastric contents to assure a pH value of 4 or less. B. Hearing air pass in the stomach after injecting air into the tubing. C. Examining a chest x-ray obtained after the tubing was inserted. D. Checking the remaining length of tubing to ensure that the correct length was inserted.

Both (A and B) are methods used to determine proper placement of the NG tubing. However, the best indicator that the tubing is properly placed is (C). (D) is not an indicator of proper placement. Correct Answer: C

114. The nurse is instructing a client with cholecystitis regarding diet choices. Which meal best meets the dietary needs of this client?

Broiled fish, green beans, and an apple

A young mother of three children complains of increased anxiety during her annual physical exam. What information should the nurse obtain first? A. Sexual activity patterns. B. Nutritional history. C. Leisure activities. D. Financial stressors.

Caffeine, sugars, and alcohol can lead to increased levels of anxiety, so a nutritional history (C) should be obtained first so that health teaching can be initiated if indicated. (A and C) can be used for stress management. Though (D) can be a source of anxiety, a nutritional history should be obtained first. Correct Answer: B

80. The nurse is providing care to a client who had major abdominal surgery. Upon return from the recovery room, the client's vital signs were at the pre-operative baseline. The client was sleepy, but arousable, and the skin was warm and dry to the touch. At the 1 hour post admission assessment the nurse notes: heart rate 120 and thready, B/P 70/40 mm Hg, and the skin is cool and clammy to the touch. What are the priority nursing actions? (Select all that apply.)

Call the health care provider. Observe for restlessness/confusion. Administer oxygen by re-breather mask. Observe the abdominal bandage.

25. After the nurse tells an older client that an IV line needs to be inserted, the client becomes very apprehensive, loudly verbalizing a dislike for all health care providers and nurses. How should the nurse respond?

Calmly reassure the client that the discomfort will be temporary.

**A complete blood count (CBC), urinalysis, and x-ray examination of the chest are prescribed for a client before surgery. The client asks why these tests are done. Which is the best reply by the nurse? a. "Don't worry; these tests are routine." b. "They are done to identify other health risks."correct3 c. "They determine whether surgery will be safe." d. "I don't know; your health care provider prescribed them."

Certain diagnostic tests (e.g., CBC, urinalysis, chest x-ray examination) are done preoperatively to rule out the existence of health problems that may increase the risks involved with surgery. Feelings will not be dispelled by telling the client not to worry; it also blocks further communication. Surgery poses a risk despite test results. Lack of knowledge without a statement of plans to obtain the information suggests incompetence on the part of the nurse.

43. The nurse assesses a 2-year-old who is admitted for dehydration and finds that the peripheral IV rate by gravity has slowed, even though the venous access site is healthy. What should the nurse do next?

Check for kinks in the tubing and raise the IV pole.

54. In assisting an older adult client prepare to take a tub bath, which nursing action is most important?

Check the bath water temperature.

6. The nurse transcribes the postoperative prescriptions for a client who returns to the unit following surgery and notes that an antihypertensive medication that was prescribed preoperatively is not listed. Which action should the nurse take?

Contact the health care provider to renew the prescription for the medication.

When conducting an admission assessment, the nurse should ask the client about the use of complimentary healing practices. Which statement is accurate regarding the use of these practices? A. Complimentary healing practices interfere with the efficacy of the medical model of treatment. B. Conventional medications are likely to interact with folk remedies and cause adverse effects. C. Many complimentary healing practices can be used in conjunction with conventional practices. D. Conventional medical practices will ultimately replace the use of complimentary healing practices.

Conventional approaches to health care can be depersonalizing and often fail to take into consideration all aspects of an individual, including body, mind, and spirit. Often complimentary healing practices can be used in conjunction with conventional medical practices (C), rather than interfering (A) with conventional practices, causing adverse effects (B), or replacing conventional medical care (D). Correct Answer: C

An African-American grandmother tells the nurse that her 4-year-old grandson is suffering with "miseries." Based on this statement, which focused assessment should the nurse conduct? A. Inquire about the source and type of pain. B. Examine the nose for congestion and discharge. C. Take vital signs for temperature elevation. D. Explore the abdominal area for distension.

Correct Answer: A Different cultural groups often have their own terms for health conditions. African-American clients may refer to pain as "the miseries. " Based on understanding this term, the nurse should conduct a focused assessment on the source and type of pain (A). (B, C, and D) are important, but do not focus on "miseries" (pain).

A client is receiving a cephalosporin antibiotic IV and complains of pain and irritation at the infusion site. The nurse observes erythema, swelling, and a red streak along the vessel above the IV access site. Which action should the nurse take at this time? A. Administer the medication more rapidly using the same IV site. B. Initiate an alternate site for the IV infusion of the medication. C. Notify the healthcare provider before administering the next dose. D. Give the client a PRN dose of aspirin while the medication infuses.

Correct Answer: B A cephalosporin antibiotic that is administered IV may cause vessel irritation. Rotating the infusion site minimizes the risk of thrombophlebitis, so an alternate infusion site should be initiated (B) before administering the next dose. Rapid administration (A) of intravenous cephalosporins can potentiate vessel irritation and increase the risk of thrombophlebitis. (C) is not necessary to initiate an alternative IV site. Although aspirin has antiinflammatory actions, (D) is not indicated.

A female client with a nasogastric tube attached to low suction states that she is nauseated. The nurse assesses that there has been no drainage through the nasogastric tube in the last two hours. What action should the nurse take first? A. Irrigate the nasogastric tube with sterile normal saline. B. Reposition the client on her side. C. Advance the nasogastric tube an additional five centimeters. D. Administer an intravenous antiemetic prescribed for PRN use. .

Correct Answer: B The immediate priority is to determine if the tube is functioning correctly, which would then relieve the client's nausea. The least invasive intervention, (B), should be attempted first, followed by (A and C), unless either of these interventions is contraindicated. If these measures are unsuccessful, the client may require an antiemetic (D)

During a visit to the outpatient clinic, the nurse assesses a client with severe osteoarthritis using a goniometer. Which finding should the nurse expect to measure? A. Adequate venous blood flow to the lower extremities. B. Estimated amount of body fat by an underarm skinfold. C. Degree of flexion and extension of the client's knee joint. D. Change in the circumference of the joint in centimeters.

Correct Answer: C The goniometer is a two-piece ruler that is jointed in the middle with a protractor-type measuring device that is placed over a joint as the individual extends or flexes the joint to measure the degrees of flexion and extension on the protractor (C). A doppler is used to measure blood flow (A). Calipers are used to measure body fat (B). A tape measure is used to measure circumference of body parts (D).

The nurse notices that the Hispanic parents of a toddler who returns from surgery offer the child only the broth that comes on the clear liquid tray. Other liquids, including gelatin, popsicles, and juices, remain untouched. What explanation is most appropriate for this behavior? A. The belief is held that the "evil eye" enters the child if anything cold is ingested. B. After surgery the child probably has refused all foods except broth. C. Eating broth strengthens the child's innate energy called "chi." D. Hot remedies restore balance after surgery, which is considered a "cold" condition.

Correct Answer: D Common parental practices and health beliefs among Hispanic, Chinese, Filipino, and Arab cultures classify diseases, areas of the body, and illnesses as "hot" or "cold" and must be balanced to maintain health and prevent illness. The perception that surgery is a "cold" condition implies that only "hot" remedies, such as soup, should be used to restore the healthy balance within the body, so (D) is the correct interpretation. (A, B, and C) are not correct interpretations of the noted behavior. "Chi" is a Chinese belief that an innate energy enters and leaves the body via certain locations and pathways and maintains health. The "evil eye," or "mal ojo," is believed by many cultures to be related to the balance of health and illness but is unrelated to dietary practice.

The nurse is performing nasotracheal suctioning. After suctioning the client's trachea for fifteen seconds, large amounts of thick yellow secretions return. What action should the nurse implement next? A. Encourage the client to cough to help loosen secretions. B. Advise the client to increase the intake of oral fluids. C. Rotate the suction catheter to obtain any remaining secretions. D. Re-oxygenate the client before attempting to suction again.

Correct Answer: D Suctioning should not be continued for longer than ten to fifteen seconds, since the client's oxygenation is compromised during this time (D). (A, B, and C) may be performed after the client is re-oxygenated and additional suctioning is performed.

A hospitalized male client is receiving nasogastric tube feedings via a small-bore tube and a continuous pump infusion. He reports that he had a bad bout of severe coughing a few minutes ago, but feels fine now. What action is best for the nurse to take? A. Record the coughing incident. No further action is required at this time. B. Stop the feeding, explain to the family why it is being stopped, and notify the healthcare provider. C. After clearing the tube with 30 ml of air, check the pH of fluid withdrawn from the tube. D. Inject 30 ml of air into the tube while auscultating the epigastrium for gurgling.

Coughing, vomiting, and suctioning can precipitate displacement of the tip of the small bore feeding tube upward into the esophagus, placing the client at increased risk for aspiration. Checking the sample of fluid withdrawn from the tube (after clearing the tube with 30 ml of air) for acidic (stomach) or alkaline (intestine) values is a more sensitive method for these tubes, and the nurse should assess tube placement in this way prior to taking any other action (C). (A and B) are not indicated. The auscultating method (D) has been found to be unreliable for small-bore feeding tubes. Correct Answer: C

100. During a clinic visit, the mother of a 7-year-old reports to the nurse that her child is often awake until midnight playing and is then very difficult to awaken in the morning for school. Which assessment data should the nurse obtain in response to the mother's concern?

Description of the family's home environment

70. In taking a client's history, the nurse asks about the stool characteristics. Which description should the nurse report to the health care provider as soon as possible?

Daily black, sticky stool

During the initial morning assessment, a male client denies dysuria but reports that his urine appears dark amber. Which intervention should the nurse implement? A. Provide additional coffee on the client's breakfast tray. B. Exchange the client's grape juice for cranberry juice. C. Bring the client additional fruit at mid-morning. D. Encourage additional oral intake of juices and water.

Dark amber urine is characteristic of fluid volume deficit, and the client should be encouraged to increase fluid intake (D). Caffeine, however, is a diuretic (A), and may worsen the fluid volume deficit. Any type of juice will be beneficial (B), since the client is not dysuric, a sign of an urinary tract infection. The client needs to restore fluid volume more than solid foods (C). Correct Answer: D

102. The nurse notes in the client's plan of care altered sleep patterns related to nocturia. Which nursing actions are important for the nurse to provide? (Select all that apply.)

Decrease intake of fluids after the evening meal. Assess the client's usual sleep pattern.

4. When taking a client's blood pressure, the nurse is unable to distinguish the point at which the first sound was heard. Which is the best action for the nurse to take?

Deflate the cuff to zero and wait 30 to 60 seconds before reattempting the reading.

84. A hospitalized client has had difficulty falling asleep for two nights, and is becoming irritable and restless. Which action by the nurse is best?

Determine the client's usual bedtime routine and include these rituals in the plan of care as safety allows.

12. The nurse who is preparing to give a 14-year-old client a prescribed antipsychotic medication notes that parental consent has not been obtained. Which action should the nurse take?

Do not give the medication and document the reason.

34. The nurse is using the Glasgow Coma Scale to perform a neurologic assessment. A comatose client winces and pulls away from a painful stimulus. Which action should the nurse take next?

Document that the client responds to painful stimulus.

42. Which instruction is most important for the nurse to include when teaching a client with limited mobility strategies to prevent venous thrombosis?

Dorsiflex and plantarflex the feet 10 times each hour.

**A graduate nurse is preparing to apply to the State Board of Nursing for licensure to practice as a licensed practical nurse. What group primarily is protected under the regulations of the practice of nursing? a. The public b. Practicing nurses c. The employing agency d. People with health problems

Each state or province protects the health and welfare of its populace by regulating nursing practice. Although the members of the nursing profession can benefit also from a clear description of their role, this is not the primary purpose of the law. The employing agency does assume responsibility for its employees and therefore benefits from maintenance of standards, but this is not the purpose of the law. People with health problems are just one portion of the population that is protected; this answer is too limited.

112. Which action is most important for the nurse to include in the plan of care for a client at high risk for the development of postoperative thrombus formation?

Encourage frequent ambulation in the hallway.

The nurse notices that the mother a 9-year-old Vietnamese child always looks at the floor when she talks to the nurse. What action should the nurse take? A. Talk directly to the child instead of the mother. B. Continue asking the mother questions about the child. C. Ask another nurse to interview the mother now. D. Tell the mother politely to look at you when answering.

Eye contact is a culturally-influenced form of non-verbal communication. In some non-Western cultures, such as the Vietnamese culture, a client or family member may avoid eye contact as a form of respect, so the nurse should continue to ask the mother questions about the child (B). (A, C, and D) are not indicated. Correct Answer: B

3. The nurse worked with a client to alleviate pain with aroma and relaxation therapy. Twenty minutes after working with the client, the nurse returns to the room and finds the client's eyes are closed and breathing deeply. What is the best entry for the nurse to document this finding?

Eyes closed, deeply breathing

65. The nurse is working with one LPN and two aides on a 20 bed unit. Which are the appropriate tasks to delegate to the appropriate person? (Select all that apply.)

Feeding an elderly and confused client to the aide Reinforcing the discharge teaching instructions to the LPN Administering a po pain medication to the LPN Performing the routine dressing change 5 days after surgery to the LPN

When evaluating a client's plan of care, the nurse determines that a desired outcome was not achieved. Which action will the nurse implement first? A. Establish a new nursing diagnosis. B. Note which actions were not implemented. C. Add additional nursing orders to the plan. D. Collaborate with the healthcare provider to make changes.

First, the nurse reviews which actions in the original plan were not implemented (B) in order to determine why the original plan did not produce the desired outcome. Appropriate revisions can then be made, which may include revising the expected outcome, or identifying a new nursing diagnosis (A). (C) may be needed if the nursing actions were unsuccessful, or were unable to be implemented. (D) other members of the healthcare team may be necessary to collaborate changes once the nurse determines why the original plan did not produce the desired outcome. Correct Answer: B

**A client who recently experienced a brain attack (CVA) and who has limited mobility complains of constipation. What is most important for the nurse to determine when collecting information about the constipation? Incorrect1 a. Presence of distention b. Extent of weight gained c. Amount of high-fiber food consumed d. Length of time this problem has existed

First, the nurse should establish when the client last defecated because the client may have perceived constipation. Abdominal distention may or may not be observed with constipation. Weight gain has no relationship to constipation. Although lack of bulk in the diet can lead to constipation, particularly in clients with limited activity or an inadequate fluid intake, the lack of bulk in the diet is not the most significant information to obtain at this time.

10. The nurse is preparing to administer a new medication through an existing IV line containing a vasopressor. What action must the nurse take first?

Flush the line with normal saline at the same rate as the vasopressor.

**An 80-year-old female is admitted to the hospital because of complications associated with severe dehydration. The client's daughter asks the nurse how her mother could have become dehydrated because she is alert and able to care for herself. The nurse's best response is: a. "The body's fluid needs decrease with age because of tissue changes." b. "Access to fluid may be insufficient to meet the daily needs of the older adult." c. "Memory declines with age, and the older adult may forget to ingest adequate amounts of fluid." d. "The thirst reflex diminishes with age, and therefore the recognition of the need for fluid is decreased."

For reasons that are still unclear, the thirst reflex diminishes with age, and this may lead to a concomitant decline in fluid intake. There are no data to support the statement "The body's fluid needs decrease with age because of tissue changes." The statement "Access to fluid may be insufficient to meet the daily needs of the older adult" is not true for an alert person who is able to perform the activities of daily living. Research does not support progressive memory loss in normal aging as a contributor to decreased fluid intake.

110. The nurse is providing care to a client receiving sq heparin every 12 hours at 8:00 am and 8:00 pm. The healthcare provider prescribes an aPTT test. At what time will the nurse plan on drawing the test? 9:00am 111. The nurse administered 10 mg of diazepam to the preoperative client. What steps will the nurse take next? (Select all that apply.)

Instruct the client not to get out of bed. Place the call bell within the client's reach. Place the side rails up, according to institutional policy.

The nurse assigns a UAP to obtain vital signs from a very anxious client. What instructions should the nurse give the UAP? A. Remain calm with the client and record abnormal results in the chart. B. Notify the medication nurse immediately if the pulse or blood pressure is low. C. Report the results of the vital signs to the nurse. D. Reassure the client that the vital signs are normal.

Interpretation of vital signs is the responsibility of the nurse, so the UAP should report vital sign measurements to the nurse (C). (A, B, and D) require the UAP to interpret the vital signs, which is beyond the scope of the UAP's authority. Correct Answer: C

The nurse is instructing a client with high cholesterol about diet and life style modification. What comment from the client indicates that the teaching has been effective? A. If I exercise at least two times weekly for one hour, I will lower my cholesterol. B. I need to avoid eating proteins, including red meat. C. I will limit my intake of beef to 4 ounces per week. D. My blood level of low density lipoproteins needs to increase.

Limiting saturated fat from animal food sources to no more than 4 ounces per week (C) is an important diet modification for lowering cholesterol. To be effective in reducing cholesterol, the client should exercise 30 minutes per day, or at least 4 to 6 times per week (A). Red meat and all proteins do not need to be eliminated (B) to lower cholesterol, but should be restricted to lean cuts of red meat and smaller portions (2-ounce servings). The low density lipoproteins (D) need to decrease rather than increase. Correct Answer: C

Examination of a client complaining of itching on his right arm reveals a rash made up of multiple flat areas of redness ranging from pinpoint to 0.5 cm in diameter. How should the nurse record this finding? A. Multiple vesicular areas surrounded by redness, ranging in size from 1 mm to 0.5 cm. B. Localized red rash comprised of flat areas, pinpoint to 0.5 cm in diameter. C. Several areas of red, papular lesions from pinpoint to 0.5 cm in size. D. Localized petechial areas, ranging in size from pinpoint to 0.5 cm in diameter.

Macules are localized flat skin discolorations less than 1 cm in diameter. However, when recording such a finding the nurse should describe the appearance (B) rather than simply naming the condition. (A) identifies vesicles - fluid filled blisters - an incorrect description given the symptoms listed. (C) identifies papules - solid elevated lesions, again not correctly identifying the symptoms. (D) identifies petechiae- pinpoint red to purple skin discolorations that do not itch, again an incorrect identification. Correct Answer: B

96. The nurse is concerned the client will develop a nosocomial infection. Which nursing action is best for the nurse to take when providing care for an incontinent client?

Maintain standard precautions.

122. The nurse is preparing to insert an IV, and cap off the IV with an intermittent infusion devise for an 80-year-old who is prescribed IV antibiotics every 8 hours. The client is taking po fluids well. What supplies will the nurse take into the room for this procedure? (Select all that apply.)

Normal saline in a 10 mL syringe Clear plastic sterile bandage kin preparation antiseptic swab

- 77. The nurse is called to the waiting room of a pediatric clinic. The frantic mother states, "I think my 4-month-old baby is choking!" What steps will the nurse take? (Select all that apply.)

Note any obstruction or absence of breathing. Deliver five backslaps between the shoulder blades. Place the infant over the nurse's arm.

48. A 75-year-old client states to the nurse, "I am just not hungry anymore." The client has lost 10 pounds/4.53 kg in the past 4 months. Which snacks will the nurse recommend to the client? (Select all that apply.)

Nuts Milkshakes Peanut butter and crackers Glass of whole fat milk

19. The nurse is preparing the room for a client after a laparotomy with a 5 inch midline abdominal incision. The nurse plans on teaching the client how to splint the wound when coughing or deep breathing. What extra item will the nurse place in the client's room?

Pillow

11. Which steps should the nurse take when administering ear drops to an adult client? (Select all that apply.)

Place the client in a side-lying position. Pull the auricle upward and outward.

79. The nurse is teaching a group of young adults with families about preparing their underground shelter in the event of a tornado. What instructions will the nurse include in teaching plan for these families? (Select all that apply.)

Plan for 1 gallon of water per family member for at least 3 days. Don't forget a can opener with the supply of canned food. Make sure you include a first aid kit in the shelter. Pack shoes with sturdy soles and they must completely cover the feet.

Three days following surgery, a male client observes his colostomy for the first time. He becomes quite upset and tells the nurse that it is much bigger than he expected. What is the best response by the nurse? A. Reassure the client that he will become accustomed to the stoma appearance in time. B. Instruct the client that the stoma will become smaller when the initial swelling diminishes. C. Offer to contact a member of the local ostomy support group to help him with his concerns. D. Encourage the client to handle the stoma equipment to gain confidence with the procedure.

Postoperative swelling causes enlargement of the stoma. The nurse can teach the client that the stoma will become smaller when the swelling is diminished (B). This will help reduce the client's anxiety and promote acceptance of the colostomy. (A) does not provide helpful teaching or support. (C) is a useful action, and may be taken after the nurse provides pertinent teaching. The client is not yet demonstrating readiness to learn colostomy care (D). Correct Answer: B

57. A nurse is working in an occupational health clinic when an employee walks in and states, "I was walking outside and I believe I was just struck by lightning." The client is alert but reports feeling faint. Which assessment will the nurse perform first?

Pulse characteristics

41. The clinic nurse is taking the vital signs of a 1-year-old. Which finding should the nurse bring to the attention of the healthcare provider?

Pulse: 80 beats/min

123. When turning an immobile bedridden client without assistance, which action by the nurse best ensures client safety?

Put bed rails up on the side of bed opposite from the nurse.

71. The nurse is providing care to a client immediately after a total right mastectomy. What steps will the nurse include when positioning the client? (Select all that apply.)

Raise the head of the bed 30 to 45 degrees. Elevate her right arm under two pillows.

91. A 76-year-old client has returned from surgery. The nurse plans on decreasing the chance of respiratory compromise for this client. What will the nurse include in this client's plan of care? (Select all that apply.)

Raise the head of the bed to no less than a 45 degrees angle. Have the client use an incentive spirometer 10 times every hour while awake. Ask the client to take deep breaths and cough five times every hour while awake.

In developing a plan of care for a client with dementia, the nurse should remember that confusion in the elderly A. is to be expected, and progresses with age. B. often follows relocation to new surroundings. C. is a result of irreversible brain pathology. D. can be prevented with adequate sleep.

Relocation (B) often results in confusion among elderly clients--moving is stressful for anyone. (A) is a stereotypical judgment. Stress in the elderly often manifests itself as confusion, so (C) is wrong. Adequate sleep is not a prevention (D) for confusion. Correct Answer: B

98. During evacuation of a group of clients from a medical unit because of a fire, the nurse observes an ambulatory client walking alone toward the stairway at the end of the hall. Which action should the nurse take?

Remind the client to walk carefully down the stairs until reaching a lower floor.

40. The nurse is preparing to initiate parenteral nutrition (PN) for a client. What actions will the nurse consider when administering PN? (Select all that apply.)

Remove the PN from the refrigerator 30 minutes before infusing. Assure the infusion time for the PN does not exceed 24 hours. Return amber and cloudy solutions of PN to the pharmacy.

63. The spouse is at the bedside of the client who just died. The hospice nurse states to the spouse, "I know your children want to come over and say goodbye before we call the funeral home. Just let me know when you are ready for me to prepare the body." What steps will the nurse include in the postmortem care? (Select all that apply.)

Remove the existing Foley catheter. Close the client's eyes. Remove soiled padding under the client.

51. The nurse is talking with the spouse of a client admitted to the long-term care center. The client has end-stage renal cancer and is admitted for palliative care while awaiting hospice placement. The client often moans and groans, but is otherwise non-communicative and somnolent. What will the nurse include in the spouse's teaching regarding the care of the client? (Select all that apply.)

Repositioning every 2 hours Round-the-clock pain medication administration Assessment for skin breakdown Back rubs three times a day

**What is a nurse's most appropriate response, based on current research, when asked about spanking as a disciplinary technique? a. "Effectiveness depends on the child's age." b. "Spanking is strongly suggestive of negative role modeling." c. "Spanking may be the only option when no other technique works." d. "Research studies have shown it to be an effective disciplinary technique."

Research suggests that children who are spanked tend to use aggressive behavior; as they grow older they learn their own behavior through their parents' behavior. Age is not significant in terms of the effectiveness of spanking. Research studies contradict the assertion that spanking is an effective disciplinary technique.

118. When bathing an uncircumcised boy older than 3 years, which action should the nurse take?

Retract the foreskin gently to cleanse the penis.

33. A client becomes angry while waiting for a supervised break to smoke a cigarette outside and states, "I want to go outside now and smoke. It takes forever to get anything done here!" Which nursing action is best for this client?

Review the schedule of outdoor breaks with the client.

108. The nurse selects the best site for insertion of an IV catheter in the client's right arm. Which documentation should the nurse use to identify placement of the IV access?

Right cephalic vein

2. Which nonverbal action should the nurse implement to demonstrate active listening?

Sit facing the client.

93. The postoperative nurse is reviewing the use of an incentive spirometer. Which instructions will the nurse include in the client's teaching plan? (Select all that apply.)

Sit in an upright position. Hold breath for 5 seconds after inhaling on the spirometer. Place mouth securely around the mouthpiece of the spirometer.

The nurse is evaluating client learning about a low-sodium diet. Selection of which meal would indicate to the nurse that this client understands the dietary restrictions? A. Tossed salad, low-sodium dressing, bacon and tomato sandwich. B. New England clam chowder, no-salt crackers, fresh fruit salad. C. Skim milk, turkey salad, roll, and vanilla ice cream. D. Macaroni and cheese, diet Coke, a slice of cherry pie.

Skim milk, turkey, bread, and ice cream (C), while containing some sodium, are considered low-sodium foods. Bacon (A), canned soups (B), especially those with seafood, hard cheeses, macaroni, and most diet drinks (D) are very high in sodium. Correct Answer: C

53. Which serum laboratory value should the nurse monitor carefully for a client who has a nasogastric (NG) tube to suction for the past week?

Sodium

97. Which action should the nurse implement when providing wound care instructions to a client who does not speak English?

Speak directly to the client, with an interpreter translating.

56. The nurse teaches the use of a gait belt to a caregiver whose spouse has right-sided weakness and needs assistance with ambulation. The caregiver performs a return demonstration of the skill. Which observation indicates that the caregiver has learned how to use the belt

Standing on the spouse's weak side, the caregiver provides security by holding the gait belt from the back

35. The nurse is obtaining a lie-sit-stand blood pressure reading on a client. Which action is most important for the nurse to take for this client?

Stay with the client while the client is standing.

119. The nurse is preparing to change the bed of a client who is non-responsive, and receiving continuous enteral tube feedings. What step must the nurse take prior to changing the bed?

Stop the feeding for 15 minutes prior to changing the bed.

26. The nurse is planning care for a client with an indwelling urinary catheter. Which nursing action has the highest priority?

Take the client's temperature every 4 hours.

101. The nurse is assessing several clients prior to surgery. Which factor in a client's history poses the greatest threat for complications to occur during surgery?

Taking anticoagulants for the past year

22. The nurse finds a client crying behind a locked bathroom door. The client will not open the door. Which action should the nurse take first?

Talk to the client and attempt to find out why the client is crying.

47. A terminally ill client tells the nurse, "I am so tired and in so much pain! Please help me to die." Which is the best response for the nurse to provide?

Talk with the client about thoughts and feelings about death.

46. A 20-year-old female client with a noticeable body odor has refused to shower for the last 3 days. She states, "I have been told that it is harmful to bathe during my period." Which action should the nurse take first?

Teach the importance of personal hygiene during menstruation with the client.

When assisting an 82-year-old client to ambulate, it is important for the nurse to realize that the center of gravity for an elderly person is the A. Arms. B. Upper torso. C. Head. D. Feet.

The center of gravity for adults is the hips. However, as the person grows older, a stooped posture is common because of the changes from osteoporosis and normal bone degeneration, and the knees, hips, and elbows flex. This stooped posture results in the upper torso (B) becoming the center of gravity for older persons. Although (A) is a part, or an extension of the upper torso, this is not the best and most complete answer. Correct Answer: B

The nurse is teaching a client proper use of an inhaler. When should the client administer the inhaler-delivered medication to demonstrate correct use of the inhaler? A. Immediately after exhalation. B. During the inhalation. C. At the end of three inhalations. D. Immediately after inhalation.

The client should be instructed to deliver the medication during the last part of inhalation (B). After the medication is delivered, the client should remove the mouthpiece, keeping his/her lips closed and breath held for several seconds to allow for distribution of the medication. The client should not deliver the dose as stated in (A or D), and should deliver no more than two inhalations at a time (C). Correct Answer: B

An elderly male client who is unresponsive following a cerebral vascular accident (CVA) is receiving bolus enteral feedings though a gastrostomy tube. What is the best client position for administration of the bolus tube feedings? A. Prone. B. Fowler's. C. Sims'. D. Supine.

The client should be positioned in a semi-sitting (Fowler's) (B) position during feeding to decrease the occurrence of aspiration. A gastrostomy tube, known as a PEG tube, due to placement by a percutaneous endoscopic gastrostomy procedure, is inserted directly into the stomach through an incision in the abdomen for long-term administration of nutrition and hydration in the debilitated client. In (A and/or C), the client is placed on the abdomen, an unsafe position for feeding. Placing the client in (D) increases the risk of aspiration. Correct Answer: B

An elderly male client who suffered a cerebral vascular accident is receiving tube feedings via a gastrostomy tube. The nurse knows that the best position for this client during administration of the feedings is A. prone. B. Fowler's. C. Sims'. D. supine.

The client should be positioned in a semi-sitting or Fowler's (B) position during feeding, in order to decrease the chance of aspiration. A gastrostomy tube, often referred to as a PEG tube, is inserted directly into the stomach through an incision in the abdomen and is used when long-term tube feedings are needed. In (A and/or C) positions, the client would be lying on his abdomen and on the tubing. In (D), the client would be lying flat on his back which would increase the chance of aspiration. Correct Answer: B

A 73-year-old female client had a hemiarthroplasty of the left hip yesterday due to a fracture resulting from a fall. In reviewing hip precautions with the client, which instruction should the nurse include in this client's teaching plan? A. In 8 weeks you will be able to bend at the waist to reach items on the floor. B. Place a pillow between your knees while lying in bed to prevent hip dislocation. C. It is safe to use a walker to get out of bed, but you need assistance when walking. D. Take pain medication 30 minutes after your physical therapy sessions.

The client's affected hip joint following a hemiarthroplasty (partial hip replacement) is at risk of dislocation for 6 months to a year following the procedure. Hip precautions to prevent dislocation include placing a pillow between the knees to maintain abduction of the hips (B). Clients should be instructed to avoid bending at the waist (A), to seek assistance for both standing and walking until they are stable on a walker or cane (C), and to take pain medication 20 to 30 minutes prior to physical therapy sessions, rather than waiting until the pain level is high after their therapy. Correct Answer: B

60. The nurse is providing care to a client receiving high doses of chemotherapy. Which situation will cause the nurse to intervene for this client?

The florist delivers an arrangement of fresh flowers.

An elderly client who requires frequent monitoring fell and fractured a hip. Which nurse is at greatest risk for a malpractice judgment? A. A nurse who worked the 7 to 3 shift at the hospital and wrote poor nursing notes. B. The nurse assigned to care for the client who was at lunch at the time of the fall. C. The nurse who transferred the client to the chair when the fall occurred. D. The charge nurse who completed rounds 30 minutes before the fall occurred.

The four elements of malpractice are: breach of duty owed, failure to adhere to the recognized standard of care, direct causation of injury, and evidence of actual injury. The hip fracture is the actual injury and the standard of care was "frequent monitoring." (C) implies that duty was owed and the injury occurred while the nurse was in charge of the client's care. There is no evidence of negligence in (A, B, and D). Correct Answer: C

The nurse is completing a mental assessment for a client who is demonstrating slow thought processes, personality changes, and emotional lability. Which area of the brain controls these neuro-cognitive functions? A. Thalamus. B. Hypothalamus. C. Frontal lobe. D. Parietal lobe.

The frontal lobe (C) of the cerebrum controls higher mental activities, such as memory, intellect, language, emotions, and personality. (A) is an afferent relay center in the brain that directs impulses to the cerebral cortex. (B) regulates body temperature, appetite, maintains a wakeful state, and links higher centers with the autonomic nervous and endocrine systems, such as the pituitary. (D) is the location of sensory and motor functions. Correct Answer: C

A client is to receive cimetidine (Tagamet) 300 mg q6h IVPB. The preparation arrives from the pharmacy diluted in 50 ml of 0.9% NaCl. The nurse plans to administer the IVPB dose over 20 minutes. For how many ml/hr should the infusion pump be set to deliver the secondary infusion?

The infusion rate is calculated as a ratio proportion problem, i.e., 50 ml/ 20 min : x ml/ 60 min. Multiply extremes and means 50 × 60 /20x 1= 300/20=150 Correct Answer: 150

An unlicensed assistive personnel (UAP) places a client in a left lateral position prior to administering a soap suds enema. Which instruction should the nurse provide the UAP? A. Position the client on the right side of the bed in reverse Trendelenburg. B. Fill the enema container with 1000 ml of warm water and 5 ml of castile soap. C. Reposition in a Sim's position with the client's weight on the anterior ilium. D. Raise the side rails on both sides of the bed and elevate the bed to waist level.

The left sided Sims' position allows the enema solution to follow the anatomical course of the intestines and allows the best overall results, so the UAP should reposition the client in the Sims' position, which distributes the client's weight to the anterior ilium (C). (A) is inaccurate. (B and D) should be implemented once the client is positioned. Correct Answer: C

An elderly resident of a long-term care facility is no longer able to perform self-care and is becoming progressively weaker. The resident previously requested that no resuscitative efforts be performed, and the family requests hospice care. What action should the nurse implement first? A. Reaffirm the client's desire for no resuscitative efforts. B. Transfer the client to a hospice inpatient facility. C. Prepare the family for the client's impending death. D. Notify the healthcare provider of the family's request.

The nurse should first communicate with the healthcare provider (D). Hospice care is provided for clients with a limited life expectancy, which must be identified by the healthcare provider. (A) is not necessary at this time. Once the healthcare provider supports the transfer to hospice care, the nurse can collaborate with the hospice staff and healthcare provider to determine when (B and C) should be implemented. Correct Answer: D

90. The health care provider has changed a client's prescription from the PO to the IV route of administration. The nurse should anticipate which change in the pharmacokinetic properties of the medication?

The onset of action of the drug will occur more rapidly, resulting in a more rapid effect.

When assessing a client with wrist restraints, the nurse observes that the fingers on the right hand are blue. What action should the nurse implement first? A. Loosen the right wrist restraint. B. Apply a pulse oximeter to the right hand. C. Compare hand color bilaterally. D. Palpate the right radial pulse.

The priority nursing action is to restore circulation by loosening the restraint (A), because blue fingers (cyanosis) indicates decreased circulation. (C and D) are also important nursing interventions, but do not have the priority of (A). Pulse oximetry (B) measures the saturation of hemoglobin with oxygen and is not indicated in situations where the cyanosis is related to mechanical compression (the restraints). Correct Answer: A

An adult male client with a history of hypertension tells the nurse that he is tired of taking antihypertensive medications and is going to try spiritual meditation instead. What should be the nurse's first response? A. It is important that you continue your medication while learning to meditate. B. Spiritual meditation requires a time commitment of 15 to 20 minutes daily. C. Obtain your healthcare provider's permission before starting meditation. D. Complementary therapy and western medicine can be effective for you.

The prolonged practice of meditation may lead to a reduced need for antihypertensive medications. However, the medications must be continued (A) while the physiologic response to meditation is monitored. (B) is not as important as continuing the medication. The healthcare provider should be informed, but permission is not required to meditate (C). Although it is true that this complimentary therapy might be effective (D), it is essential that the client continue with antihypertensive medications until the effect of meditation can be measured. Correct Answer: A

32. The nurse comes upon an automobile accident involving many cars. Which victim should the nurse see first?

The victim who is heavily bleeding bright red blood from a thigh wound

While instructing a male client's wife in the performance of passive range-of-motion exercises to his contracted shoulder, the nurse observes that she is holding his arm above and below the elbow. What nursing action should the nurse implement? A. Acknowledge that she is supporting the arm correctly. B. Encourage her to keep the joint covered to maintain warmth. C. Reinforce the need to grip directly under the joint for better support. D. Instruct her to grip directly over the joint for better motion.

The wife is performing the passive ROM correctly, therefore the nurse should acknowledge this fact (A). The joint that is being exercised should be uncovered (B) while the rest of the body should remain covered for warmth and privacy. (C and D) do not provide adequate support to the joint while still allowing for joint movement. Correct Answer: A

A male client being discharged with a prescription for the bronchodilator theophylline tells the nurse that he understands he is to take three doses of the medication each day. Since, at the time of discharge, timed-release capsules are not available, which dosing schedule should the nurse advise the client to follow? A. 9 a.m., 1 p.m., and 5 p.m. B. 8 a.m., 4 p.m., and midnight. C. Before breakfast, before lunch and before dinner. D. With breakfast, with lunch, and with dinner.

Theophylline should be administered on a regular around-the-clock schedule (B) to provide the best bronchodilating effect and reduce the potential for adverse effects. (A, C, and D) do not provide around-the-clock dosing. Food may alter absorption of the medication (D). Correct Answer: B

99. A nurse stops at a motor vehicle collision site to render aid until the emergency personnel arrive and applies pressure to a groin wound that is bleeding profusely. Later the client has to have the leg amputated and sues the nurse for malpractice. Which statement reflects the likely outcome for the nurse?

There will be no judgment against the nurse, whose actions are protected under the Good Samaritan Act.

A client is in the radiology department at 0900 when the prescription levofloxacin (Levaquin) 500 mg IV q24h is scheduled to be administered. The client returns to the unit at 1300. What is the best intervention for the nurse to implement? A. Contact the healthcare provider and complete a medication variance form. B. Administer the Levaquin at 1300 and resume the 0900 schedule in the morning. C. Notify the charge nurse and complete an incident report to explain the missed dose. D. Give the missed dose at 1300 and change the schedule to administer daily at 1300.

To ensure that a therapeutic level of medication is maintained, the nurse should administer the missed dose as soon as possible, and revise the administration schedule accordingly to prevent dangerously increasing the level of the medication in the bloodstream (D). The nurse should document the reason for the late dose, but (A and C) are not warranted. (B) could result in increased blood levels of the drug. Correct Answer: D

76. A client has a nasogastric tube connected to low intermittent suction. When administering medications through the nasogastric tube, which action should the nurse do first?

Turn off the intermittent suction device.

Which nutritional assessment data should the nurse collect to best reflect total muscle mass in an adolescent? A. Height in inches or centimeters. B. Weight in kilograms or pounds. C. Triceps skin fold thickness. D. Upper arm circumference.

Upper arm circumference (D) is an indirect measure of muscle mass. (A and B) do not distinguish between fat (adipose) and muscularity. (C) is a measure of body fat. Correct Answer: D

Which intervention is most important for the nurse to implement for a male client who is experiencing urinary retention? A. Apply a condom catheter. B. Apply a skin protectant. C. Encourage increased fluid intake. D. Assess for bladder distention.

Urinary retention is the inability to void all urine collected in the bladder, which leads to uncomfortable bladder distention (D). (A and B) are useful actions to protect the skin of a client with urinary incontinence. (C) may worsen the bladder distention. Correct Answer: D

23. The nurse identifies a potential for infection in a client with partial-thickness (second-degree) and full-thickness (third-degree) burns. What action has the highest priority in decreasing the client's risk of infection?

Use of careful handwashing technique

The healthcare provider prescribes morphine sulfate 4mg IM STAT. Morphine comes in 8 mg per ml. How many ml should the nurse administer? A. 0.5 ml. B. 1 ml. C. 1.5 ml. D. 2 ml.

Using ratio and proportion: 8mg: 1ml :: 4mg:Xml 8X=4 X=0.5 Correct Answer: A

What is the most important reason for starting intravenous infusions in the upper extremities rather than the lower extremities of adults? A. It is more difficult to find a superficial vein in the feet and ankles. B. A decreased flow rate could result in the formation of a thrombosis. C. A cannulated extremity is more difficult to move when the leg or foot is used. D. Veins are located deep in the feet and ankles, resulting in a more painful procedure.

Venous return is usually better in the upper extremities. Cannulation of the veins in the lower extremities increases the risk of thrombus formation (B) which, if dislodged, could be life-threatening. Superficial veins are often very easy (A) to find in the feet and legs. Handling a leg or foot with an IV (C) is probably not any more difficult than handling an arm or hand. Even if the nurse did believe moving a cannulated leg was more difficult, this is not the most important reason for using the upper extremities. Pain (D) is not a consideration. Correct Answer: B

A client who is 5' 5" tall and weighs 200 pounds is scheduled for surgery the next day. What question is most important for the nurse to include during the preoperative assessment? A. What is your daily calorie consumption? B. What vitamin and mineral supplements do you take? C. Do you feel that you are overweight? D. Will a clear liquid diet be okay after surgery?

Vitamin and mineral supplements (B) may impact medications used during the operative period. (A and C) are appropriate questions for long-term dietary counseling. The nature of the surgery and anesthesia will determine the need for a clear liquid diet (D), rather than the client's preference. Correct Answer: B


Set pelajaran terkait

The Cell: Anatomy and Division Exercise 4 Pre lab Quiz

View Set

ICND1-105 Part 9 (ch33-36) Network Device Management

View Set

Money & Banking Assignment Sec 4

View Set

Corporate Boards in Action Quizzes

View Set

Chapter 14 Gysbers- Assessments that Focus on Strengths and Positive Psychology:

View Set

Week 12 - Prejudice and Discrimination

View Set